Sunteți pe pagina 1din 168

Brown, Eriksen, Jones, Heffernan, Kanjanavaikoon, Leo, Mishkin, Stern, Verlander &Wasserman

Ed. Mishkin 2006


Page 1 of 168
Block I: Homeostasis & Excitable Cells
Body Fluids and Compartments I: Functional Organization of the Body and Homeostasis

1. Explain homeostasis:
a. The set of coordinate physical processes that maintain steady state
b. includes adjustments to stress and environmental change
2. Define health:
a. How well we can cope with the environment and maintain optimum function
b. lack of health is reflected by an inability to maintain homeostasis during
environmental change
3. Levels of organization & importance:
a. Cell:
i. signal transduction, metabolism, membrane transport, channels
ii. pathology example: cardiac myopathy
b. Tissue:
i. single type of cell maintaining proper function
ii. pathology – cancer
c. Organ
i. groups of organs operating in a coordinated manner
ii. pathology – congestive heart failure
d. Organism
i. groups of cell systems communicating with each other
ii. pathology – high bp or obesity
4. Basic elements of physiological regulation and feedback control
a. Disturbance Æ sensor has a receptor mechanism Æ transducer receives
information and processes it Æ decoder Æ effector Æ response Æ return to
normal Æ feedback tells sensosr whether desired result has been achieved
b. Feedback:
i. positive: accentuates disturbance –
1. childbirth
2. orgasm
ii. negative: returns critical variable to normal (very common)
iii. feedback controller:
1. sends forcing functions to controlled system
iv. adaptive controller
1. slower and evaluates measures of performance
v. feedforward
1. A Æ B Æ C Æ D and A Æ k, which is the enzyme that catalyzes
[CÆD]
5. Not in the LOs, but useful:
a. Steady state error is the amount that the final is off after compensation
b. Gain = Amount of compensation/steady state error
c. More gain Æ mark of efficiency
6. More information from the study questions:
a. Claude Bernard – Father of modern physiology, first distinguished between the
external environment and the internal milieu
b. Walter Cannon – Emphasized role of maintaining homeostasis in order to maintain
a favorable internal environment

Body Fluid Components and Fat


Brown, Eriksen, Jones, Heffernan, Kanjanavaikoon, Leo, Mishkin, Stern, Verlander &Wasserman
Ed. Mishkin 2006
Page 2 of 168

1. Basic composition of the body (lean mass, total body water & its distribution,
adipose)
a. Ideally
kg % weight % volume
Total body weight 70 100
Lean body mass 55 79
Total body water 40 57 100
Inter-cellular fluid 25 36 63
Extra-cellular fluid 15 21 37
Interstitial fluid 11.5 16 28
Plasma fluid 3.5 5 9
b. Water distribution:
i. Total body water, TBW = 73% of LBM
ii. Extracellular water, ECFV = 1/3 TBW
1. includes interstitium and plasma
iii. Intracellular water, ICFV = 2/3 TBW
2. Calculating obesity based on body water
a. TBW = 73% LBM
i. use TBW to calculate LBM
b. LBM is ideally 79% of total weight
i. use LBM to calculate ideal weight
c. Compare ideal weight to actual weight
d. Actual weight – LBM = pounds of fat
e. pounds of fat / total weight = % fat
f. If the % fat is higher than 21 – degree of obesity is how far over 21% it is
3. Normal ion concentrations in fluid compartments (be able to convert between molar
and mass values )
Cations Plasma mMol/L Cell Water mMol/kg
Na+ 135-145 10-14
K+ 3.6-5.2 120-145
Anions
Cl- 96-107 5-10
HCO3 22-28 <10
Total osmolality of both plasma and cell water is 290 mMol/kg.
To convert between mols and mass, divide by molecular weight.
4. Tonicity and predicting fluid exchange
a. Hypertonic is anything >290mMol/kg
b. Hypotonic is anything < 290 mMol/kg
c. Isotonic = 290 mMol/kg
d. Water will move in the direction of higher osmolality. Fluid exchange across
a membrane is based on total solute concentration (osmolality) with only the
total amount of impermeable solutes being taken into consideration.
e. Therefore hypertonic solutions gain water, hypotonic solutions lose water, and
isotonic solutions have no net change in water
f. Ways of looking at particles in solution:
i. Valence – Charge on particles Æ mEq
ii. Osmolarity = total # particles in solution
iii. Osmolality = total # particles in water per weight
iv. molarity = moles/L
v. osmolarity = osmols/L
Brown, Eriksen, Jones, Heffernan, Kanjanavaikoon, Leo, Mishkin, Stern, Verlander &Wasserman
Ed. Mishkin 2006
Page 3 of 168
vi. osmolality = osmols/kg
vii. osmolairty = osmolality for body fluids because 1 L of water is 1 kg
viii. osmoles are the number of particles in solution regardless of charge or MW
1. multiply molarity by the # of particles that each molecule
dissociates into in solution
ix. equivalents: one electrochemical equivalent is the amount of an ion that will
combine with 1 M of a univalent ion of the opposite persuasion (cation or
anion)
x. The concentration of a solute in mEQ/L is the concentration of the solute
in (mmol/L) X valence
g. Infusions:
i. Isotonic infusion:
1. ↑ECFV by amount infused
2. no change in ICFV
ii. Hypertonic infusion
1. ↑ ECFV by amount infused + the amount of water that comes out
to dilute the solute
2. ↓ ICFV
iii. Hypotonic infusion:
1. ↑ ECFV by amount infused - the amount of water that goes into
the cells to equilibrate the ECFV
2. ↑ ICFV
5. Indicator dilution principle to determine plasma volume, ECFV, TBW
a. Principle: an unknown volume can be determined by administering an indicator
substance which distributes itself in a known component of body fluid and waiting
for equilibration, then measuring the concentration of the indicator
b. Indicators and body compartments
i. TBW -- must be a substance that will diffuse freely throughout
ii. ECFV -- must be a substance that can’t get into cells
iii. Plasma volume -- must be a substance that can’t get into RBCs or out of
the capillary once injected
Indicator Substances to measure various compartments
TBW ECV BV PV
Heavy water (D2O) Inulin Labeled Fe RISA
HTO Mannitol Cr51 T-1824 (Evan’s Blue)
Labeled Na
Labeled SO4
c. Math:
i. Volume = amount of indicator/indicator concentration.
ii. You get an initial volume of indicator and concentration, and a final
concentration. It’s just a ratio; the total amount of indicator didn’t change,
the total volume did, so you can set up a proportion and do the math for
TBW, ECFV, and/or plasma volume
iii. ICFV, blood volume and interstitial fluid can be calculated from these
values
1. Blood volume = plasma volume/(1-Hc)
2. ICV = TBW – ECV
3. Interstitial volume = ECV – PV

Body Fluid Volumes (ideal)


Brown, Eriksen, Jones, Heffernan, Kanjanavaikoon, Leo, Mishkin, Stern, Verlander &Wasserman
Ed. Mishkin 2006
Page 4 of 168
Kg % weight % volume
Total body weight 70 100
Lean body mass 55 79
Total body water 40 57 100
Inter-cellular fluid 25 36 63
Extra-cellular fluid 15 21 37
Interstitial fluid 11.5 16 28
Plasma fluid 3.5 5 9

Membranes and Transport I: Diffusion and Osmosis

1. Fick: Electro-chemical gradient, diffusion coefficient, surface area, temperature


a. Fick’s first law: J= -DAΔC/Δx
b. Translation:
i. J is flux (net rate of diffusion over time)
ii. D is diffusion coefficient (particular to a solute)
iii. A is the surface area of the membrane
iv. ΔC is the concentration gradient
v. ΔX is membrane thickness
c. For uncharged particles only
2. Time, diffusion, distance
a. 1 µm = 0.5 msec
b. 10 µm = 50 msec
c. 100 µm = 5 sec
d. 1000 µm = 8.3 min – kinda stops being worth it after that
3. Lipid solubility and membrane permeability
a. Lipid bilayer is the major barrier to permeation by solutes
b. Lipids can traverse membranes
c. The amount of lipid solubility in a solute is the oil-water partition coefficient, β. To
find this, they create a half water, half lipid solvent and see how much of the solute
winds up in the lipid portion – this amount is the β, which is directly proportional
to Kp, the permeability constant of that solute for that membrane
i. higher Kp, higher β, more diffusion
ii. Kp = βDm/Δx
1. Dm is the diffusion coefficient of the membrane
d. Electrical gradient and molecular size also effect diffusion
4. Molecular size and diffusion
a. molecular size plays a role in membrane diffusion
i. smaller molecules diffuse more easily
b. in aqueous solution, diffusion is affected by
i. the solubility in water
ii. charge
iii. size was listed but it has less effect in solution than through a membrane
5. Permeability coefficient and flux
a. J = Kp (Co-Ci)
b. Flux = permeability constant (concentration gradient)
6. Van’t Hoff equation: physical and chemical influences on osmotic pressure
a. Van’t Hoff is for osmotically active solute particles
b. Π = RT (Φ i c)
Brown, Eriksen, Jones, Heffernan, Kanjanavaikoon, Leo, Mishkin, Stern, Verlander &Wasserman
Ed. Mishkin 2006
Page 5 of 168
c. Translation: osmotic pressure = gas constant X absolute temperature X osmotic
coefficient X the number of ions formed by dissociation of the solute X
concentration of the solute
i. R is the gas constant
ii. T is the absolute temperature
iii. Φ is the osmotic coefficient
iv. i is the number of ions formed by dissociation of solute molecule
v. c is the concentration of solute (mols/L)
d. Increasing concentration of solute and an increasing number of ions on dissociation
result in increased osmotic pressure.
7. Relationship between movements of osmotically active particles and water
a. Osmotically active particles cannot diffuse through membranes
b. Water will move from lower to higher concentrations of these particles until
compartments are equilibrated
8. Molarity, osmolarity, tonicity – and effects on cells in solution
a. Molarity = moles/liter
b. osmolarity = ions/liter
c. tonicity = impermeable ions/liter
i. Tonicity is the only one that causes net water movement
9. Reflection coefficient
a. Reflection coefficient, σ, is a measure of relative permeability
b. σ= 1 – (Cfiltrate/Csolution)
i. Cf = [solute] in filtrate after pushing all the water through the membrane
with a piston.
ii. Cs = [solute] in original solution at T=0
c. 0 < σ < 1
i. 0 is completely reflected, not permeable at all
ii. 1 is completely permeable, not reflected at all
d. This is important for understanding and predicting effecitve osmotic pressure
difference across a membrane, taking into account potential movements of water
and solute
10. Filtration coefficient – hydraulic conductivity
a. Filtration coefficient measures relative permeability of water through a membrane
b. Relates J of water across a membrane to the hydrostatic pressure and effective
osmolarity (oncotic pressure) which determine water diffusion
c. Formulas, values, constants
i. Vw = Kf (∆P)
ii. Vw = Kf(∆πeff)
iii. Vw = Kf (∆πeff - ∆P)
iv. ∆πeff = σRT(Cin – Cout)
v. Vw = Volume of water crossing membrane
vi. Kf = filtration coefficient
vii. ∆πeff = effective osmotic pressure difference across membrane
viii. ∆P = hydrostatic pressure difference across membrane
1. = reflection coefficient
ix. R = gas constant
x. T = Absolute temperature
xi. C = concentration of solute (mol/L)
Brown, Eriksen, Jones, Heffernan, Kanjanavaikoon, Leo, Mishkin, Stern, Verlander &Wasserman
Ed. Mishkin 2006
Page 6 of 168
Membranes and Transport II: Carrier-Mediated Transport

1. Carrier-mediated transport systems


a. Facilitated diffusion:
i. carrier-mediated
ii. only goes high concentration to low concentration
b. Active transport
i. carrier-mediated
ii. can go against concentration gradient
c. subtypes
i. primary: directly dependent on ATP
ii. secondary: coupled to another transport event, where a molecule is going
down its concentration gradient
1. cotransport (both go the same way)
2. antiport (swap ions)
d. All carrier mediated transport is faster than diffusion
e. Carrier proteins are highly specific (chemically and sterically)
f. Transport can be inhibited compeititvely or non-competitively
2. Examples of diffusion mechanisms
a. Diffusion
i. oxygen, carbon dioxide, water, small polar molecules
b. Facilitated diffusion
i. glucose
c. Primary Active
i. Na/K pump
d. Secondary Active
i. Ca2+ and H+ trasnported by Na+
3. Na+/K+ pump – actions, regulation, relevance
a. Uses an ATPase to transport 3 Na+ out of the cell and 2 K+ in
b. Rate of transport varies with both concentration gradients
c. high K+ is maintained inside cell
d. high Na+ is maintained outside cell
e. maintains membrane potential
f. powers other pumps by providing gradient
i. can regulate water concentrations
4. Na+/K+ pump maintaing cell volume under normal conditions
a. Na+/K+ pump counteracts (rectifies) the passive diffusion of each ion across the
membrane down its electrochemical gradient
b. Na+/K+ pump can help regulate water concentrations
5. Role of membrane transport and channels in regulatory volume response to hypo and
hyperosmotic extracellular solutions
a. Water moves in or out Æ stretches or shrinks cell Æ activates specific channels and
active transport
b. Hypoosmotic Æ cell pumps out solute (esp K+Cl-) Æ water goes towards solute
Æ water won’t flood the cell
i. Transport mechanisms:
1. K+ channels open, Cl- follows K+ through its own channel
2. cotransport of K+ and Cl –
3. K+H+ exchange and Cl-/HCO3- exchange
c. Hyperosomotic Æ cell increases concentration of solute via Na/K/2Cl cotransport
and organic osmolytes Æ water follows ions Æ water won’t all leave cell
Brown, Eriksen, Jones, Heffernan, Kanjanavaikoon, Leo, Mishkin, Stern, Verlander &Wasserman
Ed. Mishkin 2006
Page 7 of 168
6. Structural properties of epithelial cells and specialized function
a. microvilli
i. increased surface area for transport (absorption and secretion)
b. tight junctions
i. join epithelial cells on their sides to prevent mixing of apical/basolateral
membrane proteins and extracellular compartments
ii. create higher selectivity of what ions can pass through
c. functional syncytium
i. free communication – like in cardiac muscle
d. apical membranes and basolateral membranes
i. asymmetric distribution of membrane channels and transporters between
apical (absorption) and basolateral (secretion) membranes is essential for
directionality of transport (aka vectorial movement)
e. paracellular pathways
i. exist between leaky epithelial cells
ii. allow transport in between so that ions don’t have to actually cross into the
cell to get through
7. Characteristics of epithelia
a. leaky
i. allow paracellular pathways
ii. prevent cellular swelling
iii. examples:
1. proximal renal tubules
2. small intestine
b. tight epithelia
i. no paracellular transport
ii. examples
1. renal collecting duct
2. salivary gland duct
c. secretory
i. moves materials from blood to lumen
d. absorptive
i. move materials from lumen to blood
8. Specific and directional transport of solutes
a. There is usually some kind of balance of apical and basolateral channels and pumps,
so that ions are able to go from the lumen, through the cell and into the blood – or
in the other direction.
b. Na+/K+ pumps work faster than any of the other symports or antiports, so the
concentrations of these ions can almost always be used to run other channels
c. example: sugars
i. SGLT1 cotransports glucose and fructose through the apical membrane
1. 2 Na+ come in per 1 glucose
ii. GLUT 2 transports glucose through the basolateral membrane
iii. Na/K pumps on basolateral membrane Æ Na from SGLT1 is pumped out
of the cell Æ continued directional transport of glucose from the lumen,
through the cell and into the blood

Physiology of Excitable Cells I


Brown, Eriksen, Jones, Heffernan, Kanjanavaikoon, Leo, Mishkin, Stern, Verlander &Wasserman
Ed. Mishkin 2006
Page 8 of 168

1. Factors underlying electrical current flow


a. Current is thought of from the point of view of ions in solution
b. At rest, all cells have a membrane potential that is negative, meaning that the inside
of the cell is more negative than the outside
c. concentration gradients for different ions vary considerably inside and outside the
cell
i. outside – mostly sodium and chloride
ii. inside -- mostly potassium and anionic proteins (Pr-)
iii. membrane is somewhat permeable to ions, not permeable to Pr-
2. Nernst equation
a. Eion = ( RT ) * ln( [I+]o ) = -60log( [I+]o )
ZF [I+]i [I+]i

b. Translation:
i. Eion = electrochemical transmembrane potential difference of the ion
ii. Z = charge of the ion
iii. F = Faraday’s #
iv. the last item shows the natural log of the ratio of the concentration of the
ion outside (o) and inside (i) the cell
c. Nernst describes the diffusion potential generated by a single ion species – it
represents the transmembrane potential at which the electrical force will exactly
balance with the concentration difference
d. Nernst only applies to a single ion species – the GHK equation (below) calculates
the Em, for the entire membrane, based on the contributions of various ions
e. Nernst does not take into account relative permeabilities of the cell membrane to
various ions – this limitation is also addressed by GHK
3. GHK’s important determinants of transmembrane potential
a. inside/outside concentration of each species
b. relative permeability to each species
4. How various cells can have different potentials despite identical gradients
a. each ion has a different permeability in different membranes
b. amount of permeability determines how much that ion’s Eion contributes to the
overall Em
5. Estimating quantity of ions that must move across the cell membrane to establish a
diffusion potential (Nernst potential)
a. very few ions have to diffuse to produce a separation of charge change in the
membrane
b. no discernable concentration change has to occur for there to be an appreciable
voltage across the membrane
6. Na/K pump and transmembrane potentials
a. K+ leaks out of cells and Na+ leaks in
b. Over time, without the pump, ICFV and ECFV concentrations of these ions would
change
7. Depolarization and hyperpolarization
a. A polarized membrane is one where a separation of charge exists across it
b. We look at polarity from the cell’s point of view
c. At rest, membranes are polarized
d. Resting Em is negative
e. Ion movements that make the Em less negative are depolarizations
i. i.e., less polarized
ii. still called depolarization if it hits 0 and becomes positive
Brown, Eriksen, Jones, Heffernan, Kanjanavaikoon, Leo, Mishkin, Stern, Verlander &Wasserman
Ed. Mishkin 2006
Page 9 of 168
f. Ion movements that make the Em more negative are hyperpolarizations
i. i.e., even more polarized
8. Effects of passing current across the cell membrane
a. Place microelectrode in cell Æ pass current
b. In one direction, positive charges are injected into the cell – cell will become less
negative – depolarization
c. In the opposite direction, negative charges are injected into the cell –
hyperpolarization
d. enough stimulation can help the cell reach its threshold – producing an action
potential

Physiology of Excitable Cells II


1. Passive electrical responses of cell membranes
a. Currents applied to the membrane Æ response of graded, proportional amplitude in
the same direction as the current
b. Exponential decay as the potentials travel away from the site of stimulation
2. Excitable cells
a. Cells that exhibit large, rapid changes in transmembrane potential in response to
sufficient depolarization -- Triggered changes = “action potentials”
i. Neurons
ii. Skeletal muscle
iii. cardiac muscle
iv. smooth muscle
b. Main requirement is sufficient voltage-gated sodium and/or calcium channels in the
membrane
3. Threshold
a. threshold is the membrane potential at which depolarization is sufficient to trigger a
Hodgkin cycle (i.e., which allows an action potential), a regenerative increase in
membrane conductance to sodium ions
b. NOT static in a given cell – conditions can change the threshold
4. Passive electrical responses, local responses, and action potentials
a. Passive:
i. initiated by subthreshold depolarization/hyperpolarization
ii. graded
iii. degrade
b. Local:
i. slightly greater depolarization, resulting from a local increase in the
membrane conduction to sodium
c. Action potential
i. binary
ii. elicited by depolarization to at least threshold
iii. depends on voltage-gated channels (not conduction of the membrane itself)
iv. wave of permeability changes to sodium and then potassium
v. amplitude is constant with distance
vi. velocity is slower than a passive electrical response
5. Refractory periods & afterpotentials
a. Refractory periods
i. The absolute refractory period is the period of time after an action potential
when another action potential cannot be elicited, regardless of the strength
of the stimulus
1. Na+ channels are largely inactivated
Brown, Eriksen, Jones, Heffernan, Kanjanavaikoon, Leo, Mishkin, Stern, Verlander &Wasserman
Ed. Mishkin 2006
Page 10 of 168
ii. The relative refractory period follows the absolute refractory period and is a
period of time when it is more difficult than normal to elicit an action
potential
1. some sodium channels are inactivated
2. membrane is hyperpolarized, making it harder to reach threshold
iii. refraction ensures that the action potentials being generated will move away
from each other and travel in opposite directions, rather than colliding
iv. frequency of action potentials is limited
b. afterpotentials
i. depolarizing or hyperpolarizing effects – can either increase or decrease
excitability
ii. either effect can be present in an excitable cell
6. Mechanisms of action potential propagation, myelination, passive properties’
contributions
a. Stimulus Æ depolarization past threshold Æ
i. increased permeability to sodium Æ sodium rushes in Æ more positive
ions inside = large depolarization
ii. slightly delayed increased permeability to potassium Æ potassium rushes
out Æ loss of positive ions = repolarization Æ
hyperpolarization/refractory period
b. Action potentials move in both directions from the point of initiation
c. Myelin: increases velocity. Myelin sheath is an insulator with regular breaks
(“nodes”) that is resistant Æ action potential jumps from node to node “saltatory
conduction”
d. Unmyelinated areas act passively – signal is degraded
7. Axon diameter and myelination and velocity
a. Myelination increases velocity
b. Larger axon diameters increase velocity

Physiology of Excitable Cells III

1. Increasing membrane permeability to an ion and changes in membrane potential


a. Membrane potential moves towards the nernst potential for the ion to which the
membrane has been made more permeable
2. Ionic currents underlying each component of the action potential
a. Depolarization Æ rapid increase in sodium permeability (gNa+) Æ moves towards
sodium nernst potential
b. Sodium in Æ depolarization Æ overshoot
c. Potassium permeability increases Æ Moves towards potassium nernst potential
ÆPotassium out Æ repolarization Æ hyperpolarization
d. Sodium channels inactivated Æ absolute refractory period
3. Hodgkin cycle
a. Sodium channel activation Æ positive feedback cycle
b. Activation of sodium channels Æ increases gNa Æ more sodium in Æ more
depolarization Æ more activationa of sodium channels
c. This cycle allows action potentials
4. Membrane potentials’ effect on activation of voltage-gated sodium channels
a. Na+ and K+ channels are activated by voltage
i. Na+ channels are also deactivated by voltage
b. During maintained depolarization, sodium channels will be activated, but then they
will wind up in the refractory state – so the number of available channels will
Brown, Eriksen, Jones, Heffernan, Kanjanavaikoon, Leo, Mishkin, Stern, Verlander &Wasserman
Ed. Mishkin 2006
Page 11 of 168
decrease and it will become more difficult to recruit enough channels to reach
threshold Æ thershold is elevated Æ if this happens even more, further action
potentials will be prevented -- “depolarizing blockade”
5. In/Activation of sodium, potassium, calcium conductances and action potentials
a. Na+ activation Æ Na+ influx Æ action potential Æ Na+ inactivation (responsible
for both absolute refractory and relative refractory periods)
b. Potassium channels are activated and unactivated but not “inactivated” in the way
sodium channels are – activated state is open for potassium to rush out in the
repolarization phase
6. Activation and inactivation of sodium channels and thrreshold
a. Maintained depolarization Æ more sodium channels in inactive/refractory state Æ
harder to recruit channels to activated state Æ threshold is increased
7. Gating properties of voltage-dependent ion channels, related to action potentials
a. Channels opening is binary (they’re either open or closed, all the way)
b. When channels are open, the channel passes a pulse of current of constant
amplitude and varying duration -- frequency and duration is a function of the degree
of depolarization and the calcium concentration
c. When membrane gates open or close, the electrical charge associated with them
redistributes across the membrane Æ changes membrane capacitance
d. Channels randomly open and close but depolarization and hyperpolarization
respectively increase and decrease the chance channels will be open
8. Ionic mechanisms of action potentials of various shapes and durations
a. Various excitable cells have action potentials of different shapes and durations,
depending on the particular pattern of permeability changes in the particular
membrane
b. This will make way more sense in the cardio section

Physiology of Excitable Cells IV

1. Changes in Em in action potentials and threshold that result from ECF ion changes
a. External sodium changes:
i. At rest, little or no change – because the cell is far from the sodium nernst
potential
ii. Decrease Æ Sodium can’t flow in Æ harder for an action potentials to be
generated
iii. Increase Æ More sodium flows in Æ action potentials should happen more
easily
b. External potassium changes:
i. Resting membrane potential changes – because at rest, K+ is close to its
nernst potential, so Em will move towards Ek
ii. Small increase Æ enhance excitability
iii. Permanent increase Æ depolarization Æ Na inactivation Æ diminishes
excitability
c. External calcium changes
i. Calcium can compete with sodium for sites in the sodium channels
ii. Influx of calcium can lead to efflux of potassium Æ can terminate bursts of
action potentials
2. Ion channels in a cell
a. Sodium
b. Potassium
c. Calcium
Brown, Eriksen, Jones, Heffernan, Kanjanavaikoon, Leo, Mishkin, Stern, Verlander &Wasserman
Ed. Mishkin 2006
Page 12 of 168
d. Chloride
3. Membrane threshold and excitability during activation or inactivation – See #1
4. Na/K pump regulation of transmembrane gradients
a. Cell has action potentials Æ sodium builds up inside, potassium is lost
b. Na/K pump rectifies
5. Glia – we’re not responsible for this.

Physiology of Excitable Cells V

1. Sizes and types of peripheral axons


a. Peripheral nerve trunks contain both afferent and motor fibers
i. myelinated axons 1-20 μm diameter
ii. unmyelinated axons 1 μm diameter (more common in a normal human
cutaneous nerve)
iii. axon diameter is inversely proportional to conduction velocity
2. Two classification schemes of axons in nerve trunks
a. myelinated vs. nonmyelinated
b. axon diameter
3. Compound action potentials, generation, components, significance, uniqueness
a. electrical measurement of whole nerve function
b. composite of many axonal action potentials occuring simultaneously within the
nerve
c. conducted along the nerve without any decrement in amplitude (like axonal ap)
d. graded in amplitude as a function of intensity of stimulus (different from axonal ap)
e. larger diameter axons have lower thresholds than smaller ones, so an increase in
stimulus recruits more axons by activating smaller ones
4. Records of compound action potentials and measurements
a. Stimuli (mild electrical shocks) are applied to the nerve through the skin with
bipolar electrodes
b. Stimulus is placed at increasing distances from the proximal recording electrode on
a hand muscle (at the thenar eminence)
c. Conduction time between stimulus and first deflection is measured
d. Distance between stimulus and recording electrode is measured
e. Difference between times at two locations and distances of the two locations is used
to estimate velocity
5. Compund action potentials’ clinical utility
a. Good for assessing general function, recovery of function after damage
b. Only the largest myelinated fibers can be assessed in this way
i. No information about small fiber diseases
c. small amplitude through skin
i. hard to measure
6. Injury and nerve conduction
a. Three main alterations
i. Mild reduction in amplitude, normal latency
ii. Normal amplitude, increased latency
iii. Blocked response
b. Nerve compession injury Æ delay
c. Inured areas can have spontaneous action potentials
7. Spatial distribution of voltage-gaed ion channels
a. Myelinated regions: K+ channels
b. Unmyelinated nodes of ranvier: Na+ channels
Brown, Eriksen, Jones, Heffernan, Kanjanavaikoon, Leo, Mishkin, Stern, Verlander &Wasserman
Ed. Mishkin 2006
Page 13 of 168
8. Mechanisms by which demyelination affects action potentials
a. Demyelination exposes regions of axonal membrane dense in K+ channels that
cannot sustain action potentials Æ slows or stops conduction of action potentials

Physiology of Excitable Cells VI: Synapses I and II

1. Impulse transmission sequence of events:


a. Action potential propagates down axon Æ depolarizes axon terminal Æ opens
calcium channels Æ influx of calcium Æ release of transmitter substance Æ
neurotransmitter crosses synaptic cleft Æ binds to receptors on postsynaptic
membrane Æ opens channels that nonselectively pass Na and K in Æ
depolarization Æ EPSP or, if it reaches threshold Æ action potential
b. NT effect is short-lived – NT is broken down or undergoes reuptake Æ channels in
postsynaptic membrane close Æ Em slowly decays back to resting level
2. mEPPs, EPSPs, IPSPs
a. mEPP: Mini end-plate potential. As you might have guessed from the name, this is
smaller than a regular EPP. They are generated at the muscle endplate by the
spontaneous release of acetylcholine (ACh) from the presynaptic motoneuron axon
terminal.
b. EPP or EPSP: Excitatory postsynaptic potential: a transmitter-induced change in
Em Æ equilibrium potential is above threshold Æ always depolarizes
i. Summation of EPPs can lead to an action potential
c. IPSP: Inhibitory postsynaptic potential: a transmitter-induced change in EM Æ
equilibrium potential is below threshold Æ almost always hyperpolarizes
3. Explanation for EPSP and IPSP action
a. See question 2
b. When the synapse becomes active, the Em approaches the equilibrium potential of
the signal – EPSP or IPSP
c. IPSPs can be depolarizing if they result in the Em being clamped down at an Em
that is below threshold – increased membrane conductance underlying the IPSP
contributes to this clamping by causing concurrent EPSPs to be attenuated.
d. IPSPs always have their Em below threshold
4. Mechanisms of presynaptic inhibition and facilitation
a. Presynaptic inhibition:
i. a configuration of synaptic contacts that leads to a reduction in amount of
NT release upon stimulation
1. Open chloride channels Æ hyperpolarization Æ can’t reach
threshold
2. NT Æ sustained partial depolarization Æ inactivates Na channels
Æ can’t reach threshold
ii. Either way, the IPSP prevents the EPSP from reaching threshold
b. Facilitation:
i. Accumulation of calcium in the nerve terminal can result in a greater NT
release
ii. This is especially relevant in Eaton Lambert syndrome, an autoimmune
disease of calcium channels that causes weakness which improves with
exercise. Calcium channels in this disease let less calcium through than in a
normal synapse, and the improvement is a result of calcium build up over
multiple action potentials, which eventually summate to produce increased
response.
5. Temporal and spatial summation
Brown, Eriksen, Jones, Heffernan, Kanjanavaikoon, Leo, Mishkin, Stern, Verlander &Wasserman
Ed. Mishkin 2006
Page 14 of 168
a. Temporal: when presynaptic action potentials (PSPs) from one stimulus occur close
together and are summed to generate a resulting Em
b. Spatial: when PSPs from multiple stimuli at slightly different points on the soma or
dendrites are summed to generate an Em
c. Summation Æ bring Em above threshold Æ action potentials
d. IPSPs can summate with EPSPs to determine Em
6. Postsynaptic membrane
a. Sensitivity of the postsynaptic membrane – unchanged by exposure to repetitive
stimulation
i. proven because mEPPs are very consistent, regardless of recent action
potentials
b. EPP amplitude declines to a plateau during a high frequency train of impulses
c. Conclusion: there is less NT being released over time because of depletion of
substores
i. one store contains immediately releasable NT
ii. second store is less mobile and replenishes the first store
d. Role of the neuron in integrating inputs
i. Synaptic potentials generated closer to the soma have the most effect
ii. Axon hillock/initial segment has the lowest threshold for action potentials
iii. Synapses on distal dendrites, though less sensitive than the soma, can
generate calcium-mediated dendritic spikes, generated at local trigger zones
in the dendrites Æ boost amplitude of remote synaptic potentials Æ get
message to soma Æ generate action potentials

Muscle Physiology I and II: Skeletal Muscle

1. Macroscopic organization of skeletal muscle


a. muscle fiber is surrounded by endomysium
b. muscle bundle is surrounded by perimysium
c. epimysium surrounds entire muscle
d. endomysium + perimysium + epimysium = tendon
2. Microscopic organization of muscle fibers that form thick and thin filaments
a. Muscle fibers are multinucleated
b. nuclei just under plasma membrane
c. light sections: thin filaments – actin
d. dark sections: thick filaments – myosin
e. organized into myofibrils with bundles of contractile proteins
i. each bundle has 1 thick filament surrounded by 6 thin
f. myofibrils make up a muscle fiber
3. Sequence of molecular events underlying sliding filaments and crossbridge cycling
a. Myosin proteins have heads (sticking out) and tails (part of thick filament).
b. During contraction: ATP binds to myosin head Æ head is released (rigor
configuration) Æ ATP hydrolyzes Æ energy released Æ head “cocks” to resting
position Æ calcium binds troponin Æ tropomyosin on thin filament moves Æ actin
binding sites exposed Æ phosphate released from head Æ head binds to globular
actin Æ ADP released Æ head uncocks back to original position “power stroke” –
it moves the filaments with it
4. Action potential Æ Calcium release
a. At neuromuscular junction (NMJ) AP starts Æ propagates down plasma membrane
and into muscle cells along T-tubules Æ where T-tubules contact the SR,
depolarization Æ configuration change in L-type calcium channels in the T-tubule
Brown, Eriksen, Jones, Heffernan, Kanjanavaikoon, Leo, Mishkin, Stern, Verlander &Wasserman
Ed. Mishkin 2006
Page 15 of 168
membrane Æ conformation change Æ SR calcium-release channels open Æ
calcium goes into muscle cell cytoplasm Æ binds troponin… see above
5. Non-contractile proteins & sarcomere length (titin) & muscle viability (dystrophin)
a. Titin extends from the M-line to the Z-line and positions thick filaments so that
they can interact with thin filaments to provide elasticity to the sarcomere so that it
expands to its starting position upon relaxation (like a spring) Æ pushes Z-lines
back to their initial positions have crossbridge cycling has stopped
b. Dystrophin helps transmit muscle force to surrounding connective tissue
i. dystrophin is part of the costamere, which links Z-discs to the basal lamina
ii. minimizes muscle damage due to eccentric muscle contraction (lengthening
contractions) in normal exercise
iii. The malformation or lack of dystrophin is the cause of muscular dystrophy
(absence is Duchenne MD).
6. Costameres in transmitting force
a. Basal lamina forms connective tissues that come together to form the tendon
b. Sarcomere shortens Æ costamere transmits force from z-discs to basal lamina
7. Sequence of events coupling an action potential to a muscle contraction
a. AP Æ presynaptic terminal Æ depolarization Æ open calcium channels Æ Ca2+ in
Æ ACh storage vesicle release Æ ACh crosses synaptic cleft Æ binds to receptors
in muscle membrane Æ Na and K channels open Æ Positive ion flux Æ EPSP Æ
depolarize membrane to threshold Æ At neuromuscular junction (NMJ) AP starts
Æ propagates down plasma membrane and into muscle cells along T-tubules Æ
where T-tubules contact the SR, depolarization Æ configuration change in L-type
calcium channels in the T-tubule membrane Æ conformation change Æ SR
calcium-release channels open Æ calcium goes into muscle cell cytoplasm Æ
calcium binds troponin Æ tropomyosin on thin filament moves Æ actin binding
sites exposed Æ phosphate released from head Æ head binds to globular actin Æ
ADP released Æ power stroke
8. Three mechanisms that remove calcium from sarcoplasm
a. Ca2+ ATPase pumps calcium against its gradient into ECF and back into SR
b. Na+/Ca2+ exchanger uses Na+ gradient to pump 1 Ca+ out and 3 Na+ in
i. powered by Na/K pump
9. Three major biochemical pathways used by muscle to generate ATP
a. Transfer phosphate from creatine phosphate to ADP
i. fastest
ii. limited by supply of creatine phosphate
b. Anaerobic glycolysis
i. can be used when there is no oxygen
ii. faster than aerobic
iii. 2ATP/glucose
c. Aerobic respiration
i. slowest
ii. most efficient: 36 ATP/glucose, 128 ATP/fatty acid
10. Isotonic and isometric muscle contractions
a. Isotonic
i. generate force to move object
ii. takes longer
b. Isometric:
i. muscle length is fixed, shortening cannot occur
ii. shorter latency period
11. Muscle length and force
Brown, Eriksen, Jones, Heffernan, Kanjanavaikoon, Leo, Mishkin, Stern, Verlander &Wasserman
Ed. Mishkin 2006
Page 16 of 168
a. Amount of overlap between thick and thin filaments determines the possible force
that can be generated by a muscle
b. When the muscle length is too short, filaments interfere with each other and thin
filaments are pulled in opposite directions, reducing the amount of force that can be
generated
c. When the muscle length is too long, there is too little overlap, and maximum force
is reduced
d. At 70-130% length, the generated force increases to its maximum. This one’s just
right, said Goldilocks.
12. Force-velocity relationship and isometric point
a. During an isotonic contraction, as the load increases, the maximum contraction
velocity decreases
b. Isometric point: When load is equal to the maximum force that the muscle can
generate – so the muscle is contracting with zero velocity
13. Three different muscle fiber types and organization of motor units
a. Slow-resistant/slow-oxidative
i. weakest, slowest contraction, use oxidative phosphorylation to generate
ATP, fatigue slowly
b. fast-resistant
i. fast contraction velocity, use oxidative phosphorylation to generate ATP,
intermediate rate of fatigue
c. fast-fatigable
i. largest, fastest contractions, use glycolysis to generate ATP, fast fatigue
d. Motor unit = motor neuron + all of the fibers it contacts
14. Major mechanisms of neuronal induction of increased force
a. Increased action potential frequency Æ summation Æ stronger contractions
b. Increase number of active motor units Æ Increased number of recruited muscle
fibers Æ increased force
c. When only a few motor units are active, muscle contraction force is lower but
muscle contraction is still generally even throughout the muscle area
15. Muscular dystrophy, myasthenia gravis, myotonia and periodic paralysis
a. Muscular dystrophy
i. No dystrophin Æ progressive muscle degeneration
ii. X-linked heritability
b. Myasthenia gravis
i. Antibody binds to Acetylcholine receptors (nAChRs) Æ weakness
c. Myotonia
i. mutations in skeletal muscle Cl- channel Æ decrease membrane
conductance
d. Periodic paralysis
i. mutations in skeletal muscle Ca2+ channel OR mutations altering
inactivation in skeletal sodium channels Æ decreases and increases in
extracellular K+ can induce temporary muscle paralysis

Muscle Physiology III: Smooth Muscle

1. Differences between skeletal muscle fibers and smooth muscle fibers


a. Smooth muscle
i. cells are small
ii. single, central nucleus
iii. less densely packed contractile fibers
iv. thick and thin filaments are not highly organized
Brown, Eriksen, Jones, Heffernan, Kanjanavaikoon, Leo, Mishkin, Stern, Verlander &Wasserman
Ed. Mishkin 2006
Page 17 of 168
v. thick and thin filaments are arranged diagonally across the cell
vi. thin filaments connect at dense bodies instead of z-discs
2. Smooth muscle fiber linkage
a. gap junctions – electrical contacts forming ion channels spanning the membranes of
two cells
b. structural junctions – can act as tissue instead of individual cells
3. Contraction of smooth muscle and phosphorylation’s role
a. Crossbridge cycling as in skeletal muscle
b. Regulated at thick and thin filament levels:
i. Thick:
1. Calmodulin binds 4 calcium ions Æ Ca2+-calmodulin complex
binds myosin light chain kinase (MLCK) Æ active MLCK
phosphorylates myosin light chain
ii. Thin:
1. At rest, Caldesmon Æ positions tropomyosin to block myosin-
binding sites on actin
2. Calcium increases Æ Ca2+-calmodulin binds caldesmon Æ
tropomyosin moves Æ thin filament binding sites are exposed
a. Tropomyosin, but no troponin
4. How an increase in intracellular Ca2+ is sensed in smooth versus striated muscle
a. Smooth muscle
i. ↑ calcium Æ binds to calmodulin Æ Ca2+-calmodulin complex
1. binds myosin light chain kinase (MLCK) Æ active MLCK
phosphorylates myosin light chain
2. binds caldesmon Æ tropomyosin moves Æ exposes thin filament
binding sites
b. Skeletal
i. Calcium binds tropomyosin Æ exposes binding sites on actin Æ
crossbridge cycling
5. Smooth muscle contraction in the absence of ATP abundance
a. Some smooth muscle can contract tonically
b. “Latched state” mechanism: maintain contact between actin and myosin
c. Dephosphorylation of myosin while it is in contact with actin
d. Actin-myosin binding lasts longer in smooth muscle Æ myosin phosphatase
dephosphorylates myosin while the head is still attached to actin
e. Low calcium Æ number of crossbridges in latched state is maintained Æ sustained
force in spite of lack of ATP and its calcium influxes
6. Similarities and differences between smooth muscle and skeletal muscle contraction
a. Smooth muscle exhibits:
i. latched state for tonic contraction
ii. Ca2+-calmodulin regulation via myosin phosphorylation
iii. Caldesmon-controlled tropomyosin positioning
iv. G-protein coupled receptors (GPCRs) for opening SR calcium release
channels
v. Many methods of calcium increase
vi. Integration of many signals indicating degree of contraction
vii. Can survive without nervous input (as in organ transplants)
b. Skeletal muscle exhibits:
i. T-tubule channel conformational change for calcium release
ii. Only SR calcium release
iii. Signalled only by the motor neuron – and requires this input
Brown, Eriksen, Jones, Heffernan, Kanjanavaikoon, Leo, Mishkin, Stern, Verlander &Wasserman
Ed. Mishkin 2006
Page 18 of 168
7. Three methods of calcium increase in smooth muscle
a. NT-linked channels (down gradient only)
b. voltage-dependent channels (down gradient only)
c. Ca2+ release from SR – chemically, not mechanically as in skeletal muscle:
i. GPCRs Æ PLC Æ IP3 Æ calcium release
8. Four mechanisms modulating smoth muscle contractions
a. autonomic nerves
b. circulating hormones
c. local signals from other cells
d. electrical signals from other smooth muscle cells (always excitatory)
9. Differences between multiunit and single unit smooth muscle
a. Multiunit vs. single unit is determined based on the level of electrical contact
between cells
b. Multiunit
i. Very few gap junctions
ii. Each cell acts individually
iii. Each cell is innervated by ANS
iv. Found in ciliary muscle of eye, iris, pilerector muscles
c. Single unit
i. Many gap junctions Æ electrical signals are passed rapidly Æ wave of
depolarization Æ cells can act as a single unit
ii. poor ANS innervation
iii. Found in intestinal tract, uterus, ureters, blood vessels

Principles of Cardiac Excitation and Contraction I and II

1. Normal sequence of events producing normal pattern of excitation and contraction


of the heart (normal sinus rhythm)
a. Spontaneous action potentials in the sinoatrial node (SAN)
i. beats are 60-100/min
ii. action potentials spread rapidly through both atria and into upper AVN
b. Conduction through AV node
i. slow – 0.1s
ii. during delay, atria contract and assist in filling the ventricles before the
ventricles are actually stimulated
c. Impulse goes from AVN through bundle of His Æ divides into left and right
branches under endocardium, spreading down through septum, around apices,
dividing into purkinje fibers covering ventricular endocardium
i. rapid conduction so that stimulation/contraction is virtually simultaneous
throughout ventricles
ii. coordinated contractions pump blood into pulmonary and systemic
circulations
2. Major components of the surface EKG; correspond with conduction & repolarization
a. P wave: initial depolarization across right and left atria Æ voltage deflection
b. PR interval: Delay in AVN
c. Q wave: depolarization of septum Æ downward voltage deflection
d. QRS complex: depolarization of ventricular myocardium
e. T wave: ventricular repolarization
f. QT interval: time required for complete ventricular repolarization
3. Gap junctions in mediating cardiac conduction & alteration by H+ and Ca2+
a. Muscle cells are connected in series by intercalated discs consisting of gap junctions
b. Gap junctions –
Brown, Eriksen, Jones, Heffernan, Kanjanavaikoon, Leo, Mishkin, Stern, Verlander &Wasserman
Ed. Mishkin 2006
Page 19 of 168
i. low-resistance electrical coupling between myocardial cells Æ
depolarization can spread
ii. can close in pathology, including myocardial ischemia produced by low pH
or high intracellular Ca2+ Æ closure helps protect healthy cells from being
damaged by ischemic neighbors
4. Ionic currents for each of the 5 phases of ventricular action
a. Phase 4 – Leaking in between cycles to produce automaticity
i. K+ flows out - inwardly rectifying K current (IK1)
ii. Na+ or Ca+ flows in (IH) in automatic cells
b. Phase 0 / Upstroke -- depolarization
i. Na+ flows in - Na current (INa)
c. Phase 1 -- Rapid repolarization
i. K+ flows in transient outward K current (Ito)
d. Phase 2 – Plateau phase
i. Ca2+ flows in - L-type Ca current (ICa); Ito is inactivated
e. Phase 3 – Rapid repolarization
i. K+ flows out - delayed rectifier K current (IK)
5. Which ionic currents drive conduction throughout heart chambers
a. Atria INa, Ito, ICa, IK, IK1
b. Ventricles INa, Ito, ICa, IK, IK1
c. AVN ICa, IK
6. AVN role in regulating ventricular rate
a. AVN acts as a filter in the current conduction by slowing transmission from atria to
ventricles and thereby regulating PR length
b. Autonomic control
i. Sympathetic influence
1. NE Æ -adrenergic receptors Æ GPCR Æ cAMP Æ Ca2+
a. ↑ ICa, heart rate (HR), contractility
b. ↓ PR
ii. Parasympathetic/Vagal stimulation
1. ACh Æ mAChR (muscarinic acetyl choline receptors) Æ Gi Æ
inhibits AC and opens K+ channels
a. IKACh antagonizes Ca current
b. ↑ PR
c. ↓ HR
7. Ionic currents modulating SAN, AVN and purkinje automaticity
a. Automaticity is the ability to spontaneously depolarize; cells that have this ability
start depolarizations of cardiac tissue and are called pacemakers. In these cells, the
transmembrane potential is never static, because in the “at rest” state there is a net
inward flux of positive ions. The diastolic (phase 4) depolarization that underlies
automaticity results from an imbalance between the net inward flux of positive ions
(Na+ or Ca2+) that act to depolarize the cell vs. the outward flux of K+ ions which
acts to hyperpolarize the cell.
b. SAN
i. T & L-type Ca2+ currents
ii. Non selective IH current
1. It is nonselective for Na and K but usually conducted by Na.
2. activates upon hyperpolarization (<-70mV)
c. AVN
i. similar, but rate is slower
d. Purkinje
i. Too negative to open calcium channels
Brown, Eriksen, Jones, Heffernan, Kanjanavaikoon, Leo, Mishkin, Stern, Verlander &Wasserman
Ed. Mishkin 2006
Page 20 of 168
ii. IH only
8. How changes in extracellular K+ alter Em, conduction velocity & automaticity
a. Resting cardiac cells’ membranes are permeable to K+ Æ Em approaches Ek
b. Change [K+] Æ Change Ek Æ Change Em
c. Normal ECF [K+] = 4mM
d. Hyperkalemia ECF [K+] > 5mM
i. Æ depolarizes Em Æ activates less IH Æ less depolarization Æ decreases
automaticity
ii. can change automaticity in ectopic pacemakers (pacemakers in places other
than the SAN)
e. Hypokalemia
i. Æ lower Ek Æ hyperpolarize Em: makes maximum diastolic potential
more negative Æ activates larger IH Æ stronger depolarization Æ steeper
phase 4 Æ increased automaticity, potentially in ectopic pacemakers
9. Mechanisms of autonomic modulation of AVN conduction via Ih, IKACh, and ICa
a. Epi Æ b-adrenergic receptors Æ ↑ L-type ICa in AVNÆ ↑ Ca+2 influx during
diastole Æ ↑ upstroke velocity Æ ↓ AVN ERP
b. Vagal stimulation Æ ACh Æ mAChR Æ ↑ IKACh Æ ↑ K+out Æ hyperpolarization
Æ ↓ conduction velocity Æ ↑ AVN ERP
10. Basic events linking cardiac excitation and contraction, and ionic modulation
a. Excitation-contraction coupling:
i. stimulation Æaction potential spreads across cells and into T-tubules Æ
Open L-type Ca channels Æ ↑ Ca Æ trigger SR to release Ca Æ ↑ ↑ Ca Æ
↑ actin-myosin interaction Æ contraction
b. Calcium entering in phase 3 of the action potential via L-type channels is insufficient
to cause contraction, rather it acts as a trigger for Ca release from the SR: “Calcium-
induced calcium release”
c. -adrenergic agonists Æ ↑ Ca release Æ ↑ force of contraction
d. Net increase in intracellular calcium during AP is regulated by Na/Ca exchange, so
Na+ gradient can influence contractility

The Normal Electrocardiogram

1. Cardiac depolarization and repolarization generating flow of a current in chest


a. Action potentials propagate along cardiac muscle cells
b. Currents flow in complete circuits across cell membranes/through EC space
c. Currents propagate through tissues and body fluids Æ to reach skin – comparatively
small voltage is measurable at skin
2. Standard limb leads, polarity, arrangement
a. Different pairs of electrodes (leads) in strategic locations facilitate cardiac
measurement:
i. I. RA-LA – negative pole at right shoulder, positive at left shoulder
ii. II. RA-LL – negative pole at right shoulder, positive at left leg
iii. III. LA-LL – negative pole at left shoulder, positive at left leg
b. Mnemonic: Lead # = number of Ls in name/description
3. Einthoven’s triangle
a. Each lead forms one side of an equilateral, “Einthoven” triangle on the chest
4. Normal ECG and relate waves to action potentials
a. P wave: initial depolarization across right and left atria Æ voltage deflection
b. PR interval: Delay in AVN
i. changes in PR duration from autonomic influence, drugs, pathology
Brown, Eriksen, Jones, Heffernan, Kanjanavaikoon, Leo, Mishkin, Stern, Verlander &Wasserman
Ed. Mishkin 2006
Page 21 of 168
c. Q wave: depolarization of septum Æ downward voltage deflection
d. QRS complex: depolarization of ventricular myocardium
i. Spreads through purkinje fibers to endocardium, then to epicardium
ii. Prolonged QRS from blocked Na+ channels, ischemia, or hyperkalemia
e. T wave: ventricular repolarization
i. AP duration shorter in epicardium than in endocardium
ii. T wave appears upright
1. During initial depolarization, endocardium is depolarized first
2. Repolarization would normally go in the opposite direction from
depolarization (i.e., down instead of up)
3. Epicardium repolarizes first, reversing the direction of the trace
4. T wave represents repolarization but appears upright as the
depolarization waves
f. QT interval: time required for complete ventricular repolarization
i. Prolonged by K+-channel blocking drugs, genetic channel anomalies (long
QT syndrome)
ii. Prolonged QT Æ proarrhthymic Æ life-threatening
5. Directionality of cardiac de/repolarization and production of EKG waves
a. Dipoles: positively and negatively charged regions of heart muscle
i. formed by vectorial movements of current
ii. reflect progressive depolarization/repolarization
iii. Net resultant dipole Æ direction of depolarization of heart
b. Depolarization takes place from the endocardium to the epicardium
c. Repolarization takes place from the epicardium to the endocardium
d. EKG recording
i. Depolarization towards the + electrode deflects the trace upward
ii. Repolarization to the + electrode deflects the trace downward
iii. See 4-e-ii
6. Illustration of EKG

a. Note: no waves are generated by activation of SN, AVN, His or Purkinje


7. Events in the heart relating to these intervals, significance, abnormalities
i. PR interval is from beginning of P wave to beginning of Q
1. .2 seconds
2. Prolonged reflects slow conduction of impulse through AVN
3. Shortened reflects impulse being conducted over shortened route
from atria to ventricles
ii. QRS is from beginning of Q spike to the end of S spike
1. includes depolarization and contraction of ventricles
2. .06-.12 seconds
3. Prolonged reflects abnormal conduction or delay of conduction
through ventricles
iii. QT is from the beginning of the Q spike to the end of the T wave
Brown, Eriksen, Jones, Heffernan, Kanjanavaikoon, Leo, Mishkin, Stern, Verlander &Wasserman
Ed. Mishkin 2006
Page 22 of 168
1. includes depolarization and repolarization of ventricles
2. .35-.4 seconds
3. Inverted, elevated or depressed reflect ischemia or infarction
iv. ST is from end of S spike to beginning of T wave
1. period between depolarization and repolarization
2. .12 seconds
3. Elevation or depression reflects ischemia or infarction
8. Mean QRS vector projected on Einthoven’s triangle
a. Mean QRS vector or mean electrical axis of the heart is the resultant dipole of all
muscle fibers; this provides a general direction for deoplarization of the heart
b. Vectorial sum of three QRS vectors from three leads
c. should be between -30° and 110° (looking in clockwise direction from patient’s Left
horizontal… see page 9-10 of lecture from packet)
9. Factors causing the electrical axis of the heart to deviate from the normal range
a. >110 is right axis deviation
i. R ventricular hypertrophy
ii. acute right heart strain
iii. left posterior fascicular block
b. <-30 is left axis deviation
i. Left ventricular hypertrophy
ii. Inferior wall MI
iii. Left articular fascicular block
c. We do not need to memorize these

Cellular Basis of Cardiac Arrhythmias

1. How myocardial ischemia causes cardiac conduction disturbance


a. Ischemia Æ
i. ↓pHin
ii. ↓ATPin Æ Opens K+ channels and loss of Na/K pump Æ ↑[K+]out Æ
depolarizes Em Æ partial inactivation of Na current Æ disturbs conduction
of cardiac cells, but spares SN and AVN
2. Reentry
a. Reentry aka reentrant excitation can occur during slow conduction
i. Unidirectional block:
1. Subendocardial infarcation Æ slower conduction in damaged
pathway Æ degradation of signal Æ loss of signal
ii. Conduction down an undamaged pathway Æ excitation
iii. Normally, two wavefronts of depolarization meet and the ERP extinguishes
them
iv. In pathology, the undamaged pathway creates a reentrant loop Æ large
stimulus in health cells jumps over damaged region
1. summation
2. elicits action potential in depressed region
3. action potential conducts upward in retrograde direction
v. retrograde impulse re-excites tissue it had previously passed through
vi. Results in a repetitive circular pattern of excitation as long as the impulse
does not run into cells within the ERP
3. Conditions of reentry
a. Two parallel pathways of conduction must be present
b. Unidirectional conduction block must occur along one pathway
Brown, Eriksen, Jones, Heffernan, Kanjanavaikoon, Leo, Mishkin, Stern, Verlander &Wasserman
Ed. Mishkin 2006
Page 23 of 168
c. Conduction time around the circuit must be longer than the ERP of any of the cells
within the circuit – typically requires abnormally slow conduction, usually
pathological or drug-induced
4. Different degrees of AVN block
a. AVN block can present as
i. abnormal slowing of conduction – long PR
ii. Complete failure of conduction – not every P wave is followed by a QRS
b. Degrees:
i. 1st degree: Long PR, > 0.23 s; followed by QRS complex
ii. 2nd degree: Intermittent failure of AVN conduction Æ QRS dropout
1. Mobitz type 1: Progressive PR lenghtening over successive cycles
followed by a QRS dropout Æ AVN recovers and next beat is
normal. This pattern repeats.
2. Mobitz type 2: Occasional QRS dropouts, no foreshadowing PR
changes. PR can be normal or long. This type is more likely to
become 3rd degree.
iii. 3 degree: Complete AVN block.
rd

5. Purkinje fiber automaticity and ectopic packemaking


a. Three mechanisms of ectopic pacemaker automaticity:
i. Hypokalemia Æ reduces background K+ conductance in purkinje fibers
more than in SAN
ii. Ischemia Æ Localized supersensativity to catecholamines
iii. Myocardial stretch Æ depressed K+ conductance or leakiness (to all ions)
b. Disturbances in K conductance or leakage Æ depolarizes Em, spontaneous
depolarizations to threshold via mechanisms other than IH
c. IH is the normal source of automaticity in Purkinje fibers; if pathology results in
enhanced automaticity, ectopic pacemaking may take place
i. Basically this occurs if Purkinje fibers start firing faster than the SAN node;
whatever is cycling fastest sets the pace.
6. Abnormal automaticity that produce arrythmias
a. Early Afterdepolarizations (EADs)
i. Secondary depolarizations occuring before repolarization is complete
(before the end of phase 3)
ii. Slowed heart rate Æ Increased number/frequency of EADs
iii. Can result from depressed K+ conductance in phases 2 & 3
iv. Etiology:
1. Tx with K+-channel blocking drugs (Class Ia or III)
2. Antiarrhytmic drugs (i.e., quinidine)
3. acidosis
4. long QT (congenital or acquired)
b. Delayed Afterdepolarizations (DADs)
i. Depolarizations occur during phase 4
ii. Increased heart rate Æ Decreased number/frequency of DADs
iii. usually during calcium overload
1. Etiology
a. Chronic heart disease
b. Exposure to excessive levels of digoxin or catecholamies
c. Hypercalcemia
Brown, Eriksen, Jones, Heffernan, Kanjanavaikoon, Leo, Mishkin, Stern, Verlander &Wasserman
Ed. Mishkin 2006
Page 24 of 168
Block II: Cardiac & Renal
The Cardiac Pump

1. Definitions:
a. Preload: A measure of myocyte stretching in the ventricles (generally in the left
ventricle) at the end of diastole/beginning of systole.
i. pratically measured as EDV
b. Afterload: The pressure in efferent cardiac vessels at the end of systole – also, can
be considered as the amount of pressure against which the ventricles must pump at
systolic commencement. (LV afterload = aortic pressure, RV afterload = pulmonary
artery pressure).
c. Stroke volume (SV): the amount of blood pumped out of the heart with each beat.
i. SV = EDV - ESV
d. Cardiac output (CO): SV * Heart Rate (HR)
i. blood volume pumped out per minute
e. Ejection Fraction: The percentage of end diastolic volume (EDV) that leaves the
heart at the end of systole (EF = SV / EDV)
i. normal =65%
f. Stroke work (not on the list, but necessary here): SV * Ventricular Ejection
Pressure, in other words, it’s the efferent vessel pressure times the amount of fluid
being pumped against that pressure. (Stroke work for LV is about 6 times that for
RV).
g. Cardiac work: Stroke work * HR
h. Cardiac efficiency: work output/energy expended, normally around 5-10% but can
increased by activity, and up to 25% in an athlete intensely excercising.
2. Frank-Starling and Ventricular Contraction
a. ↑Venous return Æ ↑EDV Æ ↑ myocyte fiber length at the end of diastole Æ ↑SV
Æ ↑CO
b. This feature is an intrinsic phenomenon
c. There is therefore a largely linear relationship between end myocardial fiber length
at the end of diastole and ventricular performance, the slope of which is increased
sympathetically and decreased parasympathetically. The linear curve does plateau,
and therefore cardiac output is not infinitely increased by increased EDV.
i. the plateau is a result of myocardial fibers being stretched past what they
can handle
d. Propanolol blocks sympathetic innervation, decreasing HR
e. Atropine blocks parasympathetic innervation, increasing HR
3. Ventricular function curve illustrates the above relationships.
4. Ventricular performance and myocardial fiber length
a. “Ventricular Performance” in 2/c can be indexed by SV, CO, Stroke work or
cardiac work, all defined above. In other words, how much blood is being ejected
from the heart.
b. “Myocardial Fiber Length” is indexed by the ventricular EDV, ventricular ED
pressure, mean atrial pressure, and ventricular circumference. Basically, these are
things that try to ascertain how distended the muscle fibers in the heart are at the
end of diastole.
5. Stroke volume and Cardiac output:
a. CO = SV * HR. CO is therefore affected in a parallel manner by everything
described below that affects SV, and is also increased by HR and therefore increased
by sympathetic stimulation and anything that increases HR.
Brown, Eriksen, Jones, Heffernan, Kanjanavaikoon, Leo, Mishkin, Stern, Verlander &Wasserman
Ed. Mishkin 2006
Page 25 of 168
i. Sedentary people increase CO primarily by increasing HR
ii. Athletic people increase CO by increasing HR (even higher than a sedentary
person could) and increasing SV significantly – even up to over 50%
iii. Therefore, the total amount by which a sedentary individual can increase
CO is approximately 3X, and for the well-trained athlete, CO can be
increased by 6X or more.
b. SV factors:
i. MOST IMPORTANT: EDV: Increased EDV increases SV up to a point
(as per Frank-Starling)
1. EDV is determined by preload
ii. HR: Increased HR decreases SV (less time for heart to fill, so increased HR
decreases EDV, thereby decreasing SV)
iii. Ventricular stretchability (influences EDV), contractility (influences
Ejection fraction)
iv. Afterload is also an important factor (increased afterload decreases cardiac
output by reducing stroke volume)
6. Heterometric Autoregulation: two hearts beating as one
a. The heart is self-regulating, and under physiological conditions, any deviation in the
balance of right ventricular output (RVO) and left ventricular output (LVO) is
rectified automatically via the following relationships.
i. We want RVO = LVO so that both pulmonary and systemic circulation are
full but not flooded (this doesn’t mean that the same amount of blood is in
each part of the system, only that the same amount moves between systems
during any one beat of the heart, keeping the total volume of each system
relatively constant).
b. An increase in RVO increases the amount of blood being pumped to the lungs and
therefore increases the amount of blood flowing back into the left atrium via the
pulmonary vein, thereby increasing the amount of blood pumped into the left
ventricle, aka EDV, thereby increasing SV or LVO. So:
i. ↑RVO Æ ↑LA pressure Æ ↑EDV Æ ↑LVO
c. An increase in LVO fixes itself:
i. An increase in LVO causes the next filling phase to involve an increased
depletion of atrial blood, thereby decreasing the left atrial volume and
pressure after the next ventricular contraction (a.k.a. the more you pump
out of your ventricles, the less remains in left ventricle and left atrium).
ii. So in beat 1, too much blood was pumped out to the systemic circulation,
and then in beat 2, after replenishing ventricular volume, the left atrium is
depleted.
iii. This decrease in LA volume pressure also decreases the amount of volume
and pressure being pumped into the ventricle in the next atrial contraction,
thereby also decreasing left ventricular EDV.
iv. The decrease in EDV decreases the next LVO, so that over the few beats,
the CO is self-corrected. Aren’t human bodies amazing?
v. ↑LVO Æ LA volume and pressure Æ Left EDV Æ LVO
7. Methodologies of CO measurement
a. Fick principle:
i. CO times (the oxygen content of oxygenated blood) = [CO times (oxygen
content of deoxygenated blood) + (the rate at which oxygen is being
consumed), so:
ii. CO = Oxygen consumption rate/(Oxygen content in oxygenated blood in
pulmonary veins – oxygen content in deoxygenated blood in pulmonary
arteries)
Brown, Eriksen, Jones, Heffernan, Kanjanavaikoon, Leo, Mishkin, Stern, Verlander &Wasserman
Ed. Mishkin 2006
Page 26 of 168
iii. Pulmonary vein flow = pulmonary artery flow = CO
iv. Very little oxygen is removed from the blood between the pulmonary veins
and the capillary beds. So if you take a blood sample from a peripheral
artery, not too much oxygen has been lost since that blood was in the lungs.
Therefore you can use the oxygen content of peripheral arterial blood as a
reasonable estimate of the oxygenation of the blood in the pulmonary veins.
v. On the same note, peripheral venous oxygenation is a fair estimate of
pulmonary artery oxygenation. (A better estimate would be right atrial
blood, see below for why we don’t use that…)
vi. These two values can be measured much more easily than actually
catheterizing the pulmonary vessels, so peripheral oxygenation
measurements are made to estimate oxygenation around the heart, allowing
the calculation of CO with very minimal invasion.
8. Standard values (not an LO, but useful, I think…)
a. Cardiac index means CO/Body surface area in square meters
b. CO is 3146ml/m2 or about 5600 ml in a 70 kg person
c. EDV is 70ml/m2 or about 125 ml in a 70 kg person
d. ESV is 24ml/m2 or about 45 ml in a 70 kg person
e. SV is 45ml/m2 or about 80 ml in a 70 kg person
f. EF = 64%

The Cardiac Cycle

1. Chronology of events in a cardiac cycle


A normal HR is 75, making a normal cardiac cycle only .8 seconds!
a. Atrial systole:
i. aortic flow is 0
ii. aortic pressure is low
iii. atrial pressure is higher than left ventricular pressure, causing mitral valve to
open
iv. left ventricular volume is high (increases slightly during this time)
v. PQR
vi. Heart sound 4 is normally not heard in adults, and is the sound of
ventricular filling due to atrial systole
b. mitral & tricuspid valves close
i. Heart sound 1 is caused by the mitral and triscuspid valves closing. It lasts
throughout the isovolumetric contraction (below). This sound can be split,
if the bicuspid closes before the tricuspid, which is a normal variation.
c. Ventricular systole
i. isovolumetric contraction: this is when ventricules are starting to contract;
all four valves are closed at this point.
1. aortic flow is 0
2. aortic pressure is low
3. ventricular pressure > atrial pressure & ventricular pressure rises
4. left ventricular volume is high and static – this volume is EDV
5. S wave
ii. semilunar valves open
1. ventricular pressure surpasses aortic pressure, opening the aortic
valve
iii. rapid ejection
1. aortic flow increases to its maximum
Brown, Eriksen, Jones, Heffernan, Kanjanavaikoon, Leo, Mishkin, Stern, Verlander &Wasserman
Ed. Mishkin 2006
Page 27 of 168
2. the fact that the aortic valve is open allows both aortic and
ventricular pressures to reach their respective maximums, both in
the neighborhood of 120 mmHg in a person with normal blood
pressure (i.e., systolic blood pressure).
3. left atrial pressure drops slightly
4. left ventricular volume drops drastically
5. in the ST segment
iv. reduced ejection
1. aortic flow falls back down to 0
2. aortic and ventricular pressures decline (and by the end of this
phase, aortic pressure will exceed ventricular pressure)
3. left atrial atrial pressure is static
4. left ventricular volume continues to fall (less drastcially than during
rapid ejection)
5. T wave
d. Semilunar valves close
i. this is due to the fact that the aortic pressure has now again exceeded the
ventricular pressure
ii. dicrotic notch or incissura occurs: this is a decrease in aortic pressure that
dips below 0, meaning that there is backflow of blood from the aorta to the
heart
iii. heart sound 2 occurs, this is the sound of the aortic valve closing
e. Ventricular Diastole
i. Isovolumetric relaxation
1. all four valves are closed at this point
2. aortic flow is 0
3. aortic pressure rises from dicrotic notch and then slowly declines
4. ventricular pressure declines drastically
5. left atrial pressure increases slightly
6. left ventricular volume remains low – its value now is the end
systolic volume or ESV
ii. mitral valve opens
1. this occurs at the moment that the left atrial volume and pressure
exceed the left ventricular volume and pressure
iii. rapid filling
1. aortic flow is 0
2. aortic pressure is slowly declining
3. ventricular pressure declines slowly,
4. atrial pressure declines as well but exceeds ventricular pressure
5. left ventricular volume increases steadily
a. this is what causes heart sound 3, which is normally
audible in children but not adults
iv. reduced filling
1. overlaps with atrial systole (reduced filling starts during atrial
diastole, and ends just before atrial systole ends)
2. the longest phase of the cycle
3. no aortic flow
4. slight decrease in aortic pressure
5. slight increases in left ventricular and atrial pressure
6. a continued rise in left ventricular volume
2. Valvular mechanisms that regulate blood flow
a. atrial systole/ventricular filling – AV valves open, semilunars closed
Brown, Eriksen, Jones, Heffernan, Kanjanavaikoon, Leo, Mishkin, Stern, Verlander &Wasserman
Ed. Mishkin 2006
Page 28 of 168
b. isovolumetric contraction – all four valves closed
c. ventricular ejection – semilunars open, AV valves closed
d. isovolumetric relaxation – all four closed
3. Pressure changes in chambers of heart and valvular action are included in
chronology above
4. Ventricular pressure-volume relationship
a. There is a pressure-volume loop for the left ventricle
i. pressure is y axis
ii. volume is x axis
b. When mitral valve is open, pressure increases mildly until mitral closes Æ increase
in pressure Æ aortic valve opens Æ volume decreases Æ aortic valve closes Æ
isometric relaxation Æ LV pressure decreases
c. Pressure ranges from about 5 mmHg to 90 mmHg
d. End systolic volume (ESV) is the lowest volume the heart ever has, or about 45 mL
e. EDV is the highest volume the heart ever has, or about 150 mL
f. SV = EDV – ESV; in our example, = 150-45 = 105 mL (80 is normal, so the loop
we’ve been shown is probably for an athletic elephant… page 5 of 2/7/06 packet).
g. If there is an increased preload, there is an increased EDV, because of increased
venous return, and this causes an increased SV
i. preload Æ venous return Æ EDV Æ SV
ii. shown as increased width of the loop
iii. increased preload can be due to IV infusion
iv. decreased preload can be due to hemmorhage
h. If there is an increased afterload, that means there is an increased aortic pressure.
The ventricle must therefore eject blood against a higher pressure, which results in a
decreased stroke volume
i. ↑afterload Æ ↑aortic pressure Æ ↓SV
ii. shown as decreased width of the loop
i. systolic dysfunction: decreased SV (aka ventricular ejection), so more blood remains
in your ventricle – moves loop to the right because ESV and EDV are both
increased because not enough blood is leaving the heart, so more blood must be
sitting in the heart – this also narrows the loop, because SV is decreased.
j. diastolic dysfunction – your ventricles can’t relax, so the total possible amount of
blood that can ever get in is reduced. So ESV stays the same but EDV decreases –
so there is a narrowing of the loop. Again, SV is reduced.
5. Heart sounds are included in the chronology (#1)
6. Murmur and thrill
a. Caused by turbulent blood flow due to valvular problems, usually either stenosis
(not open enough) or regurgitation (not closed enough)
b. Congenital heart defects can cause murmurs
i. ventricular septal defect
ii. atrial septal defect
iii. patent ductus arteriosus – connects pulmonary artery and aorta
iv. coarctation of aorta –aortic stenosis
1. coarctation means its being pushed together
c. Systolic murmur causes
i. stenosis of semilunar valves
ii. regurgitation of AV valves
iii. Ventricular septal defect
d. diastolic murmur causes
i. Mitral stenosis
ii. Regurgitation of semilunar valves
Brown, Eriksen, Jones, Heffernan, Kanjanavaikoon, Leo, Mishkin, Stern, Verlander &Wasserman
Ed. Mishkin 2006
Page 29 of 168
e. thrill, for our purposes here at least, is when a murmur is so severe that it can be felt
through the chest wall with a bare hand

Regulation of Heart Rate and Contractility

1. Cardiac autonomic innervation


a. Sympathetic
i. effects:
1. vasoconstriction of vasculature (other than capillaries)
2. increased heartrate
3. increased contractility
ii. transmitters:
1. epinephrine – hormone, released from adrenal medulla, directly
enters circulation, and is degraded by monoamine oxidase (MAO)
2. norepinephrine – neurotransmitter, significantly more potent than
epinephrine (and is made into epinephrine by phenylethanomine n-
methyltransferase). Because norepinephrine is not considered a
hormone, it should not be measurably found in the circulation.
Systemic norepinephrine implies a catecholaminergic hyperplasia.
3. in the heart, catecholamines (dopamine, epinephrine and
norepinephrine are the catecholamines) decrease potassium
conductance and increase calcium permeability.
4. cocaine inhibits norepinephrine reuptake (and it also blocks sodium
channels, but it does a lot of more interesting things we’ll learn
about later). Reuptake into the presynaptic neuron is the main way
norepinephrine gets out of the synapse, though it is also
metabolized by COMT and MAO in the liver.
iii. receptors:
1. alpha 1 – in vascular smooth muscle, elicits vasoconstriction
2. beta 1 – in cardiac tissue, elicits increased heart rate and
contractility (due to increased Calcium influx)
a. inhibited by propanolol
3. beta 2 – in vascular (primarily pulmonary) smooth muscle, elicits
vasodilation ** so asthma treatment can include sympathetic
stimulation to increase vasodilation – “sympathomimetic drugs”
b. Parasympathetic
i. effects:
1. vasodilation of vasculature – endothelium-dependent
2. decreased heartrate
3. decreased contractility
ii. transmitter:
1. acetylcholine, released by vagal termini
2. degraded by acetylcholinesterases – rapid breakdown into choline;
choline re-enters presynaptic terminal
3. in the heart, acetylcholine increases potassium conductance and
decreases calcium permeability
iii. receptors:
1. muscarinic receptors or mAChR are found in endothelium and in
cardiac tissue
a. smooth muscle: stimulation of mAChR leads to
contractions
Brown, Eriksen, Jones, Heffernan, Kanjanavaikoon, Leo, Mishkin, Stern, Verlander &Wasserman
Ed. Mishkin 2006
Page 30 of 168
b. cardiac tissue: stimulation of mAChR leads to decreased
contractions
c. atropine inhibits mAChR
c. anatomy
i. Cardiac plexus: postganglionic autonomics (both types)
ii. Sympathetic: fibers are broadly and assymetrically distributed
iii. Parasympathetic: vagal fibers; right vagus synapses onto SA node (therefore
affects pacemaking) and left vagus synapses onto AV node (therefore
affects AV conduction)
2. Reflex pathways
a. baroreceptors: ↑ pressure results Æ ↑ stretch receptors being stretched in the
carotid sinus Æ ↑ baroreceptor firing rate Æ cardiorespiratory half of nucleus
tractus solitarius in the medulla integrates input Æ parasympathetic discharge
i. impairment can lead to hypertension
ii. no sympathetic response from baroreceptor reflex
b. chemoreceptors: ganglia on the external carotids near the bifurcation, and on the
arch of the aorta are able to sense chemical changes.
i. Mechanism: A normal concentration of ATP inhibits depolarization of
ascending sensory fibers, so a decrease in ATP (aka lowered oxygen), and
the presence instead of ADP, AMP and/or adenosine results in
depolarization of ascending sensory fibers, which, similarly to the
baroreceptor reflex, go to nucleus tractus solitarius, resulting in sympathetic
discharge. There is no parasympathetic response from the chemoreceptor
reflex. The following conditions result in this pathway:
1. low oxygen
2. high carbon dioxide
3. presence of cyanide
4. high plasma osmolarity
c. bainbridge reflex: Increased venous return resulting in increased right atrial volume
results in stretch reception, carried by vagal fibers, ultimately resulting in an
autonomically increased heart rate, which is compensatory in that it decreases the
volume of blood in the right atria. Therefore, heart disease that results in right atrial
distension is likely to lead to an increase in heart rate.
d. ventricular reflex: initiated by stretch receptors in the cardiac ventricles, eliciting a
parasympathetic response (thereby maintaing cardiac output during increased stroke
volume).
3. Reflex pathways’ factors, included throughout #2. But they all control heart rate.
4. Reflex pathways’ autonomics, also throughout #2
5. Reflex pathways’ homeostatic influences are inherent to their actions, so as long as
you know what homeostasis is, the answer is still included above.
6. Intrinsic and extrinxic myocardial factors
a. intrinsic: things that can happen without neural or hormonal influence
i. heterometric autoregulation of heart (Item 6 in “The Cardiac Pump.”)
ii. Frank-Starling: there is a linear relationship between EDV and CO
iii. Treppe phenomenon: aka “staircase phenomenon.” Increased heartrate
leads to enhanced intracellular calcium, which in turns enhances contractile
force. The result is a step-wise, continual increase in heart rate and
intracellular calcium. This can be useful when suddenly increasing activity
level (like sprinting, or like me trying to get out of bed in the morning).
iv. Frequency-dependent cardiac filling: cardiac filling is directly related to the
length of diastole. As heartrate increases, diastolic duration decreases while
Brown, Eriksen, Jones, Heffernan, Kanjanavaikoon, Leo, Mishkin, Stern, Verlander &Wasserman
Ed. Mishkin 2006
Page 31 of 168
systolic duration is static. Thus an increase in heartrate directly results in a
decrease in cardiac output.
b. extrinsic: everything else we’ve learned about, so anything that includes hormonal
control, autonomic innervation, etc.
i. Epinephrine: already discussed; a hormone, increases heart rate
ii. Glucagon: The inotropic effects of glucagon are qualitatively similar to
those of catecholamine agents but are not mediated by the beta-receptors.
As a result, glucagon can work in patients undergoing beta-blockade
therapy, but catecholamines cannot gain access to the receptors. A cAMP
pathway is still activated, resulting in increased heart rate contractility.
iii. Insulin: Increases calcium levels, thereby increasing contractility.
iv. Thyroxin: causes an increase of beta-1 receptor expression as well as these
receptors’ affinity for catecholamine substrates. This promotes a
hyperdynamic state – i.e., an increase in binding activity of sympathetic
transmitters and therefore an increase in all sympathetic activities.

Hemodynamics I

1. Flow, veolicity and area


a. Velocity = Flow/cross-sectional area
b. Cross-sectional area = πr2
c. Velocity represents the speed of a particle in a stream: distance/time
d. Flow represents the number of particles that would pass in a unit of time:
volume/time
2. Diameter and lateral pressure
a. Total pressure is constant
b. Total pressure = lateral pressure + dynamic pressure
c. lateral pressure is the pressure of the fluid on the walls of the vessel
d. dynamic pressure = fluid density (ρ) X v2 /2
i. dynamic pressure is in the direction of flow
e. If area increases, velocity decreases, decreasing dynamic pressure and increasing
lateral pressure to maintain total pressure
3. Flow, pressure and resistance (Ohm)
a. V=IR is Ohm’s law
b. E (electromotive force) = IR
c. Flow (Q) = Pressure Difference (ΔP) / Resistance (R)
4. Resistance in parallel and series
a. in series, resistance summates
b. in parallel, resistance summates in inverse
i. 1/Rtotal = 1/R1 + 1/R2 + 1/R3…
c. to calculate, if there are resistances in both series and parallel, solve for parallel first,
then count those in parallel as one in the series summation
d. Total Flow (Q) = Pressure Difference (ΔP) / Total Resistance (R)
5. Factors determining flow (Poisseuille) and uniqueness of cardiovascular system
a. Poisseuille’s law: F = P r4 / 8 L

b. translation: Flow is equal to the difference in pressure down a tube, X r4, over 8 X
the length of the tube X the viscosity of the fluid
c. F = 1/R, resistance. Therefore resistance is directly related to the length of the tube
and the viscosity of the fluid, and is inversely related not only to flow, but also to
the radius of the tube and the difference in pressure.
Brown, Eriksen, Jones, Heffernan, Kanjanavaikoon, Leo, Mishkin, Stern, Verlander &Wasserman
Ed. Mishkin 2006
Page 32 of 168
d. Assumptions to make Poisseuille work, and why they don’t work in the body:
i. constant geometry of tube
1. vasculature geometry is inconsistent and dynamic
ii. rigidity and straight shape of tube
1. vasculature is curved and compliant
a. compliance is the ability to change volume in response to a
change in pressure (direct relationship). Compliance
encompasses distensibility and elasticity.
b. Veins are more compliant than arteries.
iii. laminar flow
1. in vasculature, laminar flow exists, but turbulent flow also exists
2. laminar flow is when blood flows in parallel vectors, creating
lamellar sheets, which has a low vibration level and is highly
efficient
3. turbulent flow is when blood flows in random vectors, thereby not
producing lamella, which has a high vibration level and is not
efficient
a. turbulent flow is what causes murmurs in the heart and
bruits in the vessels
4. Reynold’s number is an index of laminar versus turbulent flow. In
general, a Reynold’s number of 2000 or less reflects laminar flow,
and a Reynold’s number of 3000 or more reflects turbulent flow
(numbers in between we don’t have to worry about, but probably
mean that both types of flow are occurring simultaneously.
a. Reynold’s # = ρDv/η
b. translation: Reynold’s# = density of the fluid times
diameter of the tube times velocity over viscosity of the
fluid
i. viscosity is lowest in the capillaries due to
“skimming,” which is that capillaries come off of
arterioles, acting as funnels, and taking more
plasma than RBCs
ii. at high viscosity, “rouleaux” formation, or
pancake-stacks of RBCs, are more common
iv. newtonian/ideal fluid - meaning that it is a solution, and therefore has only
one component, itself. Water is Newtonian, as is saline.
1. blood is not a solution, it has blood cells, for example, so it is
clearly non-newtonian, non-ideal and delicious.
2. The fact that viscosity is not constant throughout circulation is the
Fahreaus-Lindquist effect (lowest viscosity is by necessity in the
capillaries, as decribed above in 5-iii-b)
6. Laplace, aneurysms
a. T = Pr/w
b. translation: Wall tension is equal to the transmural pressure (across a wall) times the
radius of the cylinder over wall thickness
c. increased pressure increases tension and diameter and decreases the thickness of a
vessel
d. In a place where an artery narrows, the point just downtstream from that narrowing
is susceptible to damage because at the narrowing, velocity is increased (because
diameter is reduced), therefore turbulent flow is encouraged, and the turbulent, fast-
flowing blood that hits the point directly downstream is likely to increase the lateral
Brown, Eriksen, Jones, Heffernan, Kanjanavaikoon, Leo, Mishkin, Stern, Verlander &Wasserman
Ed. Mishkin 2006
Page 33 of 168
pressure and wall tension (this increase is the part called the law of LaPlace), causing
an aneurysm (swelling), which eventually will grow and rupture.
7. I know there’s no LO, but I think we should know…
a. Mean arterial pressure and pulse pressure:
i. Pulse pressure =Systolic-Diastolic
ii. Mean arterial pressure = 1/3 systolic pressure + 2/3 diastolic pressure
=1/3 pulse pressure + diastolic pressure
b. blood pressure cuff sounds are audible turbulent flow produced by the cuff’s
restriction of laminar flow
c. Blood pressure determinants:
i. Resistance increases Mean arterial pressure
ii. Cardiac output increases Mean arterial pressure
iii. Heart rate increases CO, increasing MAP
iv. Stroke volume increases CO, increasing MAP
v. Arterial pressure = HR*SV*R (Total peripheral resistance)
d. Capacitance relationship: relationship between transmural pressure and total
contained volume of a vessel (Pressure-volume relationship)

Hemodynamics II

1. Changes in pressure, velocity, and area


a. At the level of the capillaries, total cross sectional area is greatest and velocity is
lowest
b. pressure drops throught system, with the greatest fall at the level of arterioles
c. smallest arteries have resistance which decreases pressure before the blood enters
the capillaries. This is good because we want the lowest pressure and the lowest
velocity in the capillaries – not only because we don’t want to explode the
capillaries, but also because slowing down the flow of blood increases the time
available for nutrient and oxygen exchange in the tissues, which is, after all, the
purpose of the circulatory system.
2. Components of circulatory system (and everything about their flow)
a. aorta:
i. primary conduit, thickest wall, most elastic, highest velocity, highest
pressure
ii. compliance in the aorta (and arteries) minimizes pulse pressure and sustains
peripheral flow:
1. during systole, the heart contracts, filling the aorta with blood
2. blood flows down aorta, but also puts pressure against aorta wall,
causing it to dilate
3. dilation of aorta creates a potential energy, the “elastic energy,”
which is stored until diastole
4. during diastole, the heart relaxes, and ceases to put pressure on the
aorta
5. the elastic energy can then be turned into its kinetic form, so the
wall of the aorta itself increases the pressure, allowing blood flow
to continue
6. in this way, aortic pressure is maintained during diastole, allowing
for flow to be relatively continuous
7. in disease, elastic recoil can be lost, resulting in phasic flow, so that
full-on flow only occurs during systole. This can lead to a cessation
of blood flow through the capillaries during diastole.
b. arteries
Brown, Eriksen, Jones, Heffernan, Kanjanavaikoon, Leo, Mishkin, Stern, Verlander &Wasserman
Ed. Mishkin 2006
Page 34 of 168
i. with age, arterial compliance decreases, decreasing the average diameter
(because you can’t have need-based increases in diameter), increasing
velocity (V=Q/A), and thereby increasing pulse pressure
c. arterioles
i. primary sites of resistance – therefore largest pressure drop
ii. large wall to lumen ratio (like a stack of donuts! not to be confused with a
stack of pancakes, which is way more like blood cells…)
iii. arteriolar constriction increases arterial pressure (everything upstream of the
constriction), and decreases capillary hydrostatic pressure (everything
downstream of the constriction), like what would happen if Elton John
stepped on a garden hose.
d. capillaries
i. aka microcirculation, capillaries are designed to be ase close to the tissue as
possible to deliver nutrients –
1. concentration of oxygen is inversely proportional to distance from
nearest capillary.
2. oxygen donation ability of the blood in the capillaries decreases
longitudinally (distally)
ii. two types of capillaries:
1. continuous – like in the blood-brain barrier, proteins can’t leak out
a. found in muscle, lung, CNS
b. spleen editing of dying RBCs is facilitated by this type of
capillary
2. fenestrated – endothelial cells connected by diaphragms, allowing
much more in and out of the capillary
a. found in sites of fluid and metabolite absorption, like renal
corpuscles (shout out to Dr.Jeter)
iii. capillary permeability: pores allow water, ions and gas to get through, but
proteins can’t get out unless it’s fenestrated
iv. transcapillary fluid movement:
v. k(Δ −Δπ), where k is the capillary filtration coefficient (static), P is the
hydrostatic pressure (pressure pushing out of capillaries) and π is the
osmotic pressure (pressure sucking into capillaries), and differences are
taken between the capillaries and interstitial fluid
1. oncotic pressure means osmotic pressure in the capillaries – created
by the plasma proteins in the blood
2. There are four types of pressure, each pulling its own way (these
are starling forces: fluid movement forces)
a. Pc – hydrostatic pressure of the capillary – pushes fluid
from the capillary into interstital fluid
b. Pi –hydrostatic pressure of the interstitium – pushes fluid
from the interstitium into the capillary
c. πc – oncotic pressure of the capillary – pulls fluid into the
capillary from the interstitium
d. πi – oncotic pressure of the interstium – pulls fluid into
the interstitium from the capillary (this value was 0 in the
examples we were given)
e. Therefore the difference between P’s determines how
much pushing there is each way and the difference
between π’s determines how much pulling there is each
way and the total result if you use the formula shows
Brown, Eriksen, Jones, Heffernan, Kanjanavaikoon, Leo, Mishkin, Stern, Verlander &Wasserman
Ed. Mishkin 2006
Page 35 of 168
which way fluid winds up going. It winds up going out in
the arterial side of the capillary bed, where the largest
driving force is Pc, and back in on the venule side, where
the driving force is πc.
f. Increased venous pressure (Pc) that rises above πc causes
venule behavior at the capillary bed to mimic arterial
behavior, resulting in enhanced filtration on the venule
side, i.e., the blood flows from the venules into the
capillaries and into the interstitium, which is backwards
and bad for you (leads to edema and decreased venous
return)
g. A significant decrease in pressure (Pc) will result in less
blood being sent out to the tissues, which could result in
local necrosis.
h. Starling equation: Jv = Kf[ΔP –Δπ]
i. Kf is a measure of hydraulic conductance called
the “filtration coefficient”
e. venules and veins
i. If arterial system is the pressure reservoir, the venous system is the actual
reservoir (it holds most of the volume).
ii. high level of compliance
iii. one-way valves in series facilitate unidirectional flow and reduce the height
of fluid column to be overcome at any given point (it divides the venous
blood into smaller compartments making it easier for the blood to scale the
body to get back up to the heart).
iv. muscle contraction facilitates venous return:
1. when muscles contract, veins are physically constricted, forcing
blood upwards
2. when muscles relax, more blood can fill the veins from below
3. blood can’t go the wrong way because of the valves
f. vena cavae
i. the inferior vena cava has to take a large amount of blood to the heart
against gravity. it has the help of the thoraco-abdominal pump:
1. during inspiration, the diaphragm contracts, moving downwards.
Thoracic pressure is decreased and abdominal pressure is increased,
forcing the venous blood upward, closer to the heart.
2. during expiration, the diaphragm relaxes and moves up, decreasing
abdominal pressure, allowing blood flow from the lower
extremities to fill the portion of the IVC that is located in the
abdomen
3. if inspiration and expiration continue phasically, as in normal
breathing, this mechanism will continue to help pump blood to the
heart against gravity
4. in the case of a pneumothorax, increased thoracic pressure
eliminates this mechanism. There will be resultant decreased
venous return, decreased SV and CO.
3. Elastic recoil of aorta, blood flow and age – see 2-a-ii
4. arterioles and resistance – see 2-c
5. solute and fluid flux across capillary endothelium – see 2-d-v
6. venous circulation and gravity – see 2-e-iv and 2-f
7. lymphatic system and edema
Brown, Eriksen, Jones, Heffernan, Kanjanavaikoon, Leo, Mishkin, Stern, Verlander &Wasserman
Ed. Mishkin 2006
Page 36 of 168
a. the lymphatic system helps remove fluid from the interstitium
b. smallest unit is a blind sac: fluid enters blind sacs from tissue, then fluid travels
through lymphatic capillaries, collecting capillaries, and lymph vessels, to nodes, and
liquid eventually returns to the systemic circulation when fluid travels through the
thoracic duct into the left subclavian vein
i. smooth muscle begins at the level of the lymph vessel
1. spontaneous contraction (limited sympathetic control)
ii. lymphatic capillaries and vessels have one-way valves

Local control of Vasculature

1. Adrenergic receptors
a. alpha 1
i. location: vascular smooth muscle
ii. elicits: vasoconstriction
b. beta 1
i. location: cardiac tissue
ii. elicits: increased heart rate and contractility
c. beta 2
i. location: pulmonary and vascular smooth muscle
ii. elicits: bronchial dilation – allows greater intake of oxygen and facilitation
of fight or flight response
d. dopaminergic receptors (not adrenergic, wait for the connection…)
i. elicit renal vasodilation
ii. dopamine also activates beta-1 receptors
iii. dopamine is converted to norepinephrine, which is converted to
epinephrine

This is biochem, but looking at


the physical similarities and
differences between these
molecules makes it feel way more
logical that they all stimulate the
same receptors but with different
affinities.

2. Epinephrine vs norepinephrine
a. Epinephrine is a hormone released by the adrenal medulla
b. Norepinephrine is a neurotransmitter released by sympathetic nerve endings
i. the affinity of norepinephrine for adrenergic receptors is *much* higher
than the affinity of epinephrine for these receptors
3. Local control of blood flow
a. instrinsic factors
i. basal vascular tone: vessels are always partially constricted: therefore you
can have change that go either way. how ingenious.
ii. myogenic response: increased transmural pressure (against the wall),
pushing out against the wall, resulting in an increase in elastic recoil. this is
a physical response of the smooth muscle. This is most significant in
tissue where systemic pressure is likely to change because it helps equalize
pressure by restricting huge increases and compensating for drops.
Brown, Eriksen, Jones, Heffernan, Kanjanavaikoon, Leo, Mishkin, Stern, Verlander &Wasserman
Ed. Mishkin 2006
Page 37 of 168
iii. metabolic regulation: tissue can have enough oxygen or not. When not,
you use up your ATP and wind up with adenosine (triply dephosphorylated
ATP). Adenosine is a vasodilator, which is protective because it allows
greater blood flow, which helps if you don’t have enough oxygen in your
blood.
1. adenosine binds to adenosine receptors (shocking) which
decreases the sensitivity of contractile proteins for calcium, which
relaxes arterioles (we don’t know how this happens, but we do
know that the calcium concentration itself does not cause this
change) increasing flow, restoring flow and oxygen
b. endothelial-derived factors: all produced by endothelial cells themselves in response
to receptors or shear stress
i. NO, nitric oxide, Adrienne’s favorite chemical
1. a shear force-mediated dilator
a. Cells are in vascular wall
b. shear force in this instance = luminal flow
2. increased flow through the vessels increases dilation – so during
increased flow, each endothelial cell is impacted, which results in
NO release, which dilates the vessels further by relaxing local
smooth muscle
3. inside the endothelial cell, L-Arginine is converted to L-citruline
and NO by the enzyme nitric oxide synthase
4. NO then stimulates soluble guanylate cyclase Æ cGMP Æ reduces
calcium flux Æ relaxing smooth muscle
5. NO is always being produced and has a short effect (half-life is
about 1 second)
6. inhibition of NO can lead to hypertension
7. ACh, Bradykinin and Histamine all promote an increase in
Calcium influx, and subsequent contraction. All 3 also result in
NO release, which counteracts the contraction
a. ACh stimulates release of NO synthase via endothelial
mAChRs
b. Bradykinin stimulates constrction of smooth muscle and
dilation of vascular tissue Æ dilation increases NO
i. graded response
c. Histamine released during anaphylactic reactions can also
promote NO release
8. more about NO and all of its amazing abilities tomorrow
ii. prostaglandins
1. actions:
a. can promote vasodilation or vasoconstriction
b. those produced by platelets promote platelet aggregation
and blood clotting (thromboxane, or TXA2, is the direct
clot forming factor)
c. promote renal dilation in heart failure
i. COX inhibitors might promote renal constriction
and renal failure
2. synthesis:
a. arachidonic acid Æ prostaglandins, via cyclooxygenase
(COX)
b. COX 1 – gastric mucosa protection
c. COX 2 –
Brown, Eriksen, Jones, Heffernan, Kanjanavaikoon, Leo, Mishkin, Stern, Verlander &Wasserman
Ed. Mishkin 2006
Page 38 of 168
i. inflammation,
ii. endothelium-dependent vasodilation in chronic
cardiovascular and renal disease
iii. vioxx, celebrex are COX-2 inhibitors but cannot
be used in patients with the above conditions
(hence the recall)
3. Non-steroid anti-inflammatory drugs (NSAIDs, i.e., aspirin) work
by inhibiting both COX forms: aspirin irreversibly inhibits COX
in platelets and endothelium Æ endothelium synthesizes new
COX within a few hours, but platelets cannot Æ decreases clot
formation. This is why aspirin reduces the chances of vascular
clotting, stroke and MI
4. COX-2 inhibitors only inhibit COX in the endothelium, not
platelet prostaglandin formation
a. COX 1 continues to make TXA2, thereby continuing to
encourage clotting
b. COX 2 inhibitors are preventing anti-inflammation and
reduction of clotting
c. encouraging clotting + not reducing clotting = sucks
iii. endothelins
1. Peptides derived from endothelium
2. cause constriction – possible role in hypertension

Regulation of Regional Vascular Beds

From page 3 of the notes, a little simplification:


Structure Smooth Muscle Sympathetic Metabolic Regulation
Arterioles yes yes minor
Metarterioles sparse very sparse major
Precapillary single fiber very sparse major
sphincter
Capillaries no no no
Venules some yes minor

Learning objectives:
1. Factors regulating coronary blood flow
a. controlled almost entirely by local metabolic factors
b. exhibits autoregulation
c. increased metabolic activity requires more blood flow Æ therefore an increase in
activity promotes vasodilation to facilitate increased blood access
d. Work = SV * Aortic pressure
e. Phasic blood flow in coronary arteries:
i. left branch: diastole lowers resistance, maximizes flow
ii. right branch: less phasic & less flow
f. Cardiac perfusion factors
i. metabolic regulation by adenosine: adenosine vasodilates (not enough
ATP, you must need more oxygen, better get more blood flowing…)
ii. sympathetic modulation (indirect; minor role)
iii. peripheral resistance (increases blood flow to heart tissue)
iv. beta-1 receptor antagonists slow heartrate, increase diastole Æ increase
perfusion and SV
Brown, Eriksen, Jones, Heffernan, Kanjanavaikoon, Leo, Mishkin, Stern, Verlander &Wasserman
Ed. Mishkin 2006
Page 39 of 168
2. Severe hypotension, partial coronary artery occlusion, severe aortic stenosis – effects
on coronary blood flow, oxygen supply and oxygen demand
a. stenosis: narrowing of the aortic valve Æ increases LV pressure Æ increases O2
consumption (without increasing work output) Æ hypoxia, angina pectoris, cardiac
failure
i. little relief from vasodilators because the vessel is already dilated, but it’s
still blocked, so angioplasty would be the more effective treatment
b. coronary atherosclerosis: limits flow to heart, leading to ischemia (no collateral
circulation exists here)
c. cardiovasospasm: weakness, angina, possible contributor to “sudden cardiac death”
i. random constriction
ii. vasodilation drug treatment helps with this (makes sense)
d. hypotension
i. most commonly caused by hypovolemia
ii. leads to low blood pressure
iii. it may sound like it wouldn’t be as bad as hypertension, but as it turns out,
less is more. Symptoms include chest pain, shortness of breath, irregular
heartbeat, fever, headache, stiff neck, back pain, productive cough,
prolonged diarrhea and/or vomiting, difficulty eating, burning and foul-
odored urine, allergies, seizures, and loss of consciousness.
3. Temperature, neural and local factors regulating cutaneous blood flow
a. temperature
i. principal function of cutaneous sympathetic nerves
ii. increased ambient temperature Æ vasodilation, allows dissipation of heat
iii. decreased ambient temperature Æ vasoconstriction, heat retention
b. neural
i. extensive sympathetic innervation mediates response to cold
ii. hypothalamus responds to heat Æ withdraws sympathetic tone Æ lets
local parasympathetic tone take over
c. local
i. cutaneous blood flow is extrinsically controlled
ii. metabolic regulation
iii. sweat glands
1. adjacent to parasympathetics – activation causes sweat glands to
release a factor which promotes bradykinin formation Æ
vasodilation
iv. anastomosis – “shunt”
1. anastomosis constricts Æ increases flow to distal portions of
peripheral appendages (a.k.a. fingertips)
2. highly responsive to circulating vasoconstrictors
3. no metabolic control, no autoregulation
4. Neural and local factors and myogenic mechanisms in regulating skeletal muscle
blood flow
a. increase activity Æ motor cortex releases sympathetic discharge to heart and
vasculature Æ vasoconstriction Æ tissue becomes hypoxic Æ local metabolic
factors cause vasodilation, allowing more blood flow, and more oxygen reaches
muscle
i. 20-fold dilation is required to keep up with maximum exertion
b. venous pumping: rhythmic motion of skeletal muscle exerts pressure on vessels,
improving venous blood return
c. neural
Brown, Eriksen, Jones, Heffernan, Kanjanavaikoon, Leo, Mishkin, Stern, Verlander &Wasserman
Ed. Mishkin 2006
Page 40 of 168
i. controlled by extrinsic sympathetic innervation of blood vessels running
through skeletal muscle (arteries more than veins)
ii. sympathetic innervates skeletal muscles at rest
iii. adrenergic receptor stimulation:
1. alpha-1 Æ vasoconstriction
2. beta-2 Æ vasodilation
iv. baseline constriction Æ peripheral resistance
d. local
i. metabolic control – lactate, adenosine and potassium Æ vasodilate
1. adenosine is most powerful
e. myogenic mechanisms
i. vascular smooth muscle contracts when stretched
ii. hypothesis is that increased pressure will cause constriction of vessels
5. Neural and local factors controlling cerebral blood flow (CO2, Pa)
a. Cranium is incompressible & BBB limits exchanges, therefore flow in = flow out
within brain
b. local regulation is the major regulatory feature
i. autoregulation is present from 60-160 mmHg (set point shifted in
hypertension)
ii. high sensitivity to O2, some to CO2
c. the most important local vasodilator for cerebral circulation is carbon dioxide
increases
d. vasoactive substances in systemic circulation have little or no effect on cerebral
circulation because of the blood brain barrier
e. neural role is small and indirect (autonomic tone affects peripheral blood flow and
blood pressure, which affect blood being pumped to the brain, but no direct
autonomic regulation exists for cerebral blood flow)
f. Cerebral ischemia
i. Cushing’s phenomeon is an example of the response to cerebral ischemia
ii. Increases in intracranial pressure and/or infarct, etc., anything that reduces
oxygen Æ compress cerebral blood vessels Æ O2 delivery decreases,
medullary O2 center chemoreceptor senses Æ sympathetic discharge to
increase peripheral pressure (so that more blood can get to the brain) Æ
systemic vasoconstriction Æ baroreceptor reflex senses high BP Æ
DECREASES heart rate
1. HR is low and BP is high (even at life-threatening levels of high)
because of baroreceptor reflex – this is the defining feature
2. so you’re losing blood into your own brain. this sucks. this
happens in the terminal stages of acute head injury.
iii. “Last breath” phenomenon – is the sympathetic response associated with
drastically decreased oxygen – lets you gasp for breath as you’re dying
1. usually freaks the family out, but it’s great if you were drowning
because you might actually live
6. No LO specifically, but we obviously have to talk about viagra.
a. parasympathetic innervation is required for erection and sympathetic innervation is
requred for ejaculation (yup, you have to be relaxed to get it up, but you have to be
excited to ejaculate... sounds right).
b. parasympathetic fibers lead to afferent arterioles Æ NO Æ cGMP Æ vasodilation
Æ turgidity of corpus cavernosum
i. cGMP is normally degraded by a phosphodiesterase
ii. Viagra is a phosphodiesterase inhibitor, so it prolongs vasodilation
Brown, Eriksen, Jones, Heffernan, Kanjanavaikoon, Leo, Mishkin, Stern, Verlander &Wasserman
Ed. Mishkin 2006
Page 41 of 168

Short-term Regulation of Arterial Pressure

Arterial pressure = CO * total peripheral resistance


“Short term” means about 24 hours
1. Baroreceptor reflexes in arterial regulation
a. baroreception is the fastest means of decreasing HR, and therefore decreasing BP
b. for more information, see “Regulation of Heart Rate and Contractility” LO #2
2. Afferent and efferent neural pathways of carotid sinus/aortic arch baroreceptors
a. see “Regulation of Heart Rate and Contractility” LO #2 for backstory
b. in response to inspiration, pulmonary stretch receptors promote systemic dilation
c. mean arterial pressure less than 100 mmHg qualifies to set off this system
d. no regulation is ever “complete” – always modulatory
3. Autonomic NT and arterial pressure
a. NE/Epi Æ vasoconstriction
b. ACh Æ slows HR, decreased contractility
4. Carotid body/aortic arch chemoreception and the medulla, regulating arterial
pressure
a. See “Regulation of Regional Vascular Beds” LO#5-f
5. renin-angiotensin, vasopressin, atrial natriuretic peptide in BP regulation
a. Renin-angiotensin-aldosterone system:
i. low BP Æ decrease in renal perfusion pressure Æ renin released Æ cleaves
angiotensinogen (from liver) Æ angiotensin I Æ circulates to lungs Æ
angiotensin converting enzyme (ACE) cleaves angiotensin I Æ angiotensin
II (a potent vasoconstrictor) Æ 1. restores BP and 2. causes release of
aldosterone from the adrenal cortex Æ aldosterone increases Na
reabsorption and therefore water reabsorption Æ increased ECF volume
also helps to restore BP
1. the aldosterone component of this is slow because it requires new
protein synthesis, so this overall mechanism works both
immediately and long term, immediately vasoconstricting and long-
term increasing water retention to maintain blood volume
ii. ACE inhibition:
1. captopril (an example): blocks ACE, therefore blocks conversion
of angiotensin I to angiotensin II, limiting vasoconstriction and
decreasing BP
2. losartan (another example): antagonizes angiotensin II receptors,
limiting vasoconstriction and decreasing BP
a. it’s apparently ok to have excess renin and angiotensin I
hanging out in your system – there are no known side
effects
iii. CHF patients have high HR, low breathing rate at night, and an increased
urge to urinate at night
1. Treatable with ACE inhibitors
b. vasopressin (anti-diuretic hormone, or ADH)
i. vasoconstriction and water reabsorption are the actions
ii. involved with longterm blood pressure regulation
iii. degrades in 3-4 minutes
iv. released from posterior pituitary in response to increased osmolarity (too
much concentration, so you need to keep more water) or reduced stretch in
cardiac tissue (too little fluid stretching, so water is retained to restore)
Brown, Eriksen, Jones, Heffernan, Kanjanavaikoon, Leo, Mishkin, Stern, Verlander &Wasserman
Ed. Mishkin 2006
Page 42 of 168
v. anything that causes fluid redistribution (changing gravity’s effects, by
escaping gravity or changing your position) will result in suppression of
vasopressin (and a subsequent need to pee)
1. astronauts are given a synthetic vasopressin replacement
c. atrial natriuretic peptide (ANF… F for factor)
i. released by atria in response to increased atrial pressure/stretch
ii. causes vascular smooth muscle relaxation Æ arterial dilation Æ decreases
total peripheral resistance
iii. causes increased excretion of sodium (“natriuresis”) and therefore water Æ
reduce blood volume Æ reduces pressure
iv. inhibits renin secretion Æ reduces pressure
v. ANF is produced after a meal to promote rapid removal of sodium
1. it’s the only hormone known to cause direct sodium excretion
vi. a major factor that enhances survival during heart failure
1. ANF is increased in CHF
2. Decreased contractility Æ decreased CO Æ increased renal
absorption (via renin-angiotensin system) Æincreases pulmonary
pressure Æ pulmonary edema Æ compression Æ pulmonary
resistance Æ increases pulmonary pressure (cyclic)
3. ANF can keep a CHF patient alive for about 5 years, but it
suddenly ceases to be adrenally recognized, resulting in rapid
decline and death within 48 hours.
6. Time line of responses:
a. Baroreceptors, Chemoreceptors, Angiotensin II within first 15 seconds
b. Aldosterone is effective around 2 hours after a shift

Role of peripheral circulation in control of cardiac output

1. Measuring CO, determining effects of central venous pressure changes


a. CO = HR X SV
b. SV = EDV – ESV
c. Central venous pressure increases EDV, increasing SV, therefore increasing CO –
up to the plateau
d. SV increases with contractility and with sympathetic tone, also increasing CO
2. Changes in heart rate, myocardial contractility, preload and afterload affect CO
a. preload = pressure of venous return
i. a certain amount of preload is necessary to fill the heart
ii. linear relationship between venous pressure and CO in normal function
1. slope increases and decreases along with contractility
a. increased contractility – sympathetic
b. decreased contractility – parasympathetic (or heart failure)
2. slope has a negative correlation with changes in afterload
a. decreased afterload – vasodilation
b. increased afterload – vasoconstriction
3. curve shifts with increases and decreases in volume
a. shift to the right (up) – increase in volume
b. shift to the left (down) – decrease in volume
iii. excessive preload Æ inability to clear vena cava blood Æ plateau in CO
b. afterload = pressure of arterial vasculature
i. work = stroke volume * afterload
3. Central venous pressure and CO (sounds strangely familiar, like question #1)
Brown, Eriksen, Jones, Heffernan, Kanjanavaikoon, Leo, Mishkin, Stern, Verlander &Wasserman
Ed. Mishkin 2006
Page 43 of 168
a. At high CO, venous pressure is lowest because the heart pumps blood out of
venous circulation and into arterial circulation. As CO declines, more blood is
pooling in the venous system, so reduction in CO results in an increase in venous
pressure and a decrease in arterial pressure.
b. Increased VP, however, increases CO: increase volume Æ increase venous pressure
Æ increase CO

Vascular function curves: Circle represents point of “theoretical equilibrium,” towards which the
system will move under given conditions. The axes are different for the vascular curve than for the
ventricular function curve in two senses. First, for the vascular curve, the Y axis is the independent
variable and the X axis is the dependent variable. Secondly, for the vascular curve, the Y axis
represents venous return (which is obviously correlated with CO, but isn’t exactly the same), and the
X axis represents Right atrial pressure.

Heart failure:
This curve shows a reduction in CO, directly resulting
from decreased contractility, and causing an increase
in venous pressure because more blood is pooling in
the venous system. This could be a result of CHF or
negative inotropic agents.

Inotropic stimulation:

This curve shows increased contractility resulting in


increased CO, and therefore a decrease in venous
pressure (because more blood is ejected from the
heart, removing it from venous circulation). This
could be a result of positive inotropic agents (eg.
digitalis), or sympathetic tone.
Brown, Eriksen, Jones, Heffernan, Kanjanavaikoon, Leo, Mishkin, Stern, Verlander &Wasserman
Ed. Mishkin 2006
Page 44 of 168
Blood loss or venous dilation:

This shows a shift in the vascular curve but not the


ventricular curve – so now the CO is decreasing and
venous pressure is decreasing, both because of decreases in
blood volume (hemorrhage) and/or increases in
compliance.

Volumetric expansion:

Here the vascular curve has shifted to the right,


increasing CO and venous pressure. Underlying causes:
water retention, transfusion, i.e., increases in blood
volume, or decreases in venous compliance.

Arterial constriction:

An increase in Arterial constriction reduces the amount


of blood reaching the venous side, thereby reducing
venous pressure. Arterial constriction separately
increases the afterload, thereby decreasing SV, thereby
decreasing CO, thereby increasing ESV, thereby
increasing venous pressure for the blood trying to get
into the heart. So CO is definitely decreasing, but VP is
both increasing and decreasing, so VP tends to break
even.

Arterial dilation:

Arterial dilation increases the flow through the arteries


to the venous system, increasing VP. At the same time,
arterial dilation decreases afterload, increasing SV and
CO, decreasing ESV, decreasing VP. So this time, CO is
increased, but VP again is increasing and decreasing and
there is ideally no net effect on VP.
Brown, Eriksen, Jones, Heffernan, Kanjanavaikoon, Leo, Mishkin, Stern, Verlander &Wasserman
Ed. Mishkin 2006
Page 45 of 168

The effects of progressive increases in volume:


initial increase improves CO, but further
increases are futile as the plateau has been
reached. Therefore extra volume will not
improve CO but will result in edema. This can
happen when a person with compromised cardiac
function is given a transfusion, because the set
point is decreased, and the heart is unable to
adapt to the new increased volume, resulting in
respiratory distress. Instead of transfusing a
patient, administering drugs that increase
contractility (dobutamine, dopamine, ionotropics,
digitalis) might buy the patient time.

Circulatory Shock

1. Hemorrhage and hypovolemic shock


a. insufficient oxygen/nutrient delivery Æ diffuse irreversible deterioration Æ
complete system failure
b. shock types are categorized based on the nature of the initiation
c. Hemorrhagic shock is the most common form of hypovolemic shock
i. blood pressure can be maintained despite blood loss, but CO will decrease
ii. patients can survive acute hypotension, only to die a few days later from
multiorgan failure (renal, etc) due to ischemia
2. Compensation for hypovolemia
a. Compensated shock is non-progressive – generally an early stage of shock
i. always some compromise of nutrient delivery
ii. can have reduced BP
iii. can have reduced CO
b. In compensated phase, negative feedback mechanisms are protective against low
arterial pressure
c. Mechanisms:
i. sympathetic activation
1. baro and chemoreceptors (seconds)
2. arterial pressure falls Æ parasymp withdrawn Æ increased HR Æ
CO & BP increased (or attempted)
3. increase HR, contractility, total peripheral resistance,
vasoconstriction, decreased compliance
a. vasoconstriction occurs in peripheral tissues but not in
coronary or cerebral vascular beds
b. sympathetic innervation increases arterial pressure, but
what is really desired is increased arterial supply or flow –
so although sympathetic activation can prolong life
somewhat, it isn’t really targeting the problem
appropriately – all tissues other than the prioritized heart
and brain wind up starving for nutrients
4. Full-blast sympathetic activation stimulates vasoconstriction but
also reduces ventricular function (because it decreases diastole and
therefore decreases EDV, decreasing SV), so you’re increasing
Brown, Eriksen, Jones, Heffernan, Kanjanavaikoon, Leo, Mishkin, Stern, Verlander &Wasserman
Ed. Mishkin 2006
Page 46 of 168
arterial pressure and decreasing cardiac output at the same time –
so not that much blood is going to get to the tissues expediently
ii. CNS ischemia (minutes)
1. decreased oxygen Æ oxygen deficit Æ accumulation of CO2 in
blood/fall in pH Æ sympathetic discharge via chemoreceptors Æ
attempts to restore cardiac output and blood pressure
2. in the brain, medulla is extremely sensitive to O2 changes,
reinforcing sympathetic tone
iii. renin-angiotensin-aldosterone (minutes to hours)
1. a fall in arterial pressure Æ fall in renal perfusion pressure Æ
renin-angiotensin system – angiotensin II promotes
vasoconstriction Æ attempt to restore blood pressure (impairs CO)
iv. vasopressin increase (minutes)
v. reabsorption of extravascular fluid (hours)
vi. renal remodeling (days)
3. Decompensatory mechanisms – acidosis, end-organ failure (heart, kidney, intestine,
liver)
a. Decompensated phase is 25% normal CO or less
b. progression involves systemic deterioration secondary to reduced perfusion
i. “irreversible” = inevitably fatal
c. Progressive phase:
i. deteriorating blood pressure
ii. deteriorating CO
iii. Rouleaux / thrombosis / sludging
1. sludging: decreased blood velocity Æ increased blood viscosity Æ
cells stick together
a. limited oxygen
b. macrophages are blocked
c. venous resistance increased
iv. capillary damage
v. increased venous resistance
vi. increased oxygen deficit
vii. lactic acidosis / loss of catecholaminergic response
1. build-up of CO2 & increase of anaerobic response Æ lactic acid
buildup Æ lowers pH Æ denatures catecholaminergic receptors
a. dopamine can bind past when NE & Epi can, so
dopaminergic drug therapy is a good last-ditch effort
viii. endotoxins
4. Hemorrhagic shock treatment
a. DON’T give a transfusion in cardiac Æ as per the last LO set, it’s only going to
cause edema, so the patient will drown in the blood you perfuse
b. dobutamine/dopamine/glycosides are the only thing that might help (see 3/c/vii/a)
– anything that will increase contractility
c. see below in chart

Fleshed out “Pocket Guide” to shock: Here these forms are separated into clean little categories we
can memorize, but in real life shock is often a combination of these things.
Type: Hypovolemic
Examples: blood or plasma loss – like as in burn victims
Primary Problems: decreased circulating volume Æ decrease in CO
Brown, Eriksen, Jones, Heffernan, Kanjanavaikoon, Leo, Mishkin, Stern, Verlander &Wasserman
Ed. Mishkin 2006
Page 47 of 168
Initial hemodynamic changes: There is a decrease in total volume, therefore obviously a decrease in
central venous pressure, because less blood is pooling in the venous system. This is what causes the
decreased CO. Sympathetic innervation also increases TPR in an attempt to compensate.
Treatment options: transfusions, plasma expanders
Prognosis: reasonable if treated early
Type: Cardiac
Examples: extensive infarct
Primary Problems: decreased contractility Æ decreased CO
Initial hemodynamic changes: There is decreased contractility, so therefore there is decreased CO.
Meanwhile, blood is waiting to get into the heart in the venous system, and it backs up, increasing
central venous pressure. Sympathetic innervation increases TPR in an attempt to make flow
constant.
Treatment options: no infusions! drugs which increase cardiac contractility
Prognosis: very poor
Type: Vasodilatory
Examples: anaphylaxis, endotoxins
Primary Problems: extensive peripheral vasodilation
Initial hemodynamic changes: Vasodilation means decreased peripheral resistance, so the CO is
increased because it is pumping against a much-decreased afterload. Since CO is increased, more
blood is being pulled out of venous reservoirs, so there is decreased central venous pressure.
Treatment options: glucocorticoids, antihistamines, vasoconstrictors
Prognosis: reasonable if treated early
Type: Neurogenic
Examples: anesthesia overdose, brain damage
Primary Problems: decreased CO, extensive peripheral vasodilation
Initial hemodynamic changes: Neurogenic shock is a result of sympathetic collapse. So all
sympathetic activities should cease, resulting in decreased peripheral resistance, but also decreased
CO, so even though afterload is reduced, the heart still is pumping out at a lower rate, so like in
cardiac shock, blood is waiting in the venous system to be pumped, and there is an increase in central
venous pressure.
Treatment options: pacemaker, vasoconstrictors
Prognosis: poor

Renal hemodynamics and regulation of glomerular filtration rate

1. Renal fraction of CO: define, calculate, factors


a. 20% of CO is in the renal fraction = 1.2 L or 4 ml/min/gram
i. this is one of the highest rates in the body, and it delivers more oxygen than
the kidney can use
b. Renal fraction = Renal blood flow (RBF) / CO
i. RBF = total renal flow / combined weight of kidneys
c. Factors:
i. arterial pressure
ii. blood composition
iii. neural imputs
iv. hormonal signalling
2. Average values for RBF, GFR and compare blood flow and O2 consumption in renal
tissue and skeletal muscle
a. RBF: 1200ml/min
b. GFR: 130ml/min
c. Blood flow is much higher in renal tissue than in skeletal tissue, and more oxygen is
consumed in renal tissue than in skeletal tissue, but proportionately (amount of
Brown, Eriksen, Jones, Heffernan, Kanjanavaikoon, Leo, Mishkin, Stern, Verlander &Wasserman
Ed. Mishkin 2006
Page 48 of 168
oxygen consumed over amount of blood perfused), skeletal muscle consumes more
oxygen than the kidneys
3. Filtration fraction: define, calculate
a. Filtration fraction = GFR / RPF
b. Translation: Glomerular filtration rate / Renal plasma flow = the amount of the
plasma coming through the kidneys are able to filter
c. usually about .17-9
4. Extrinsic/intrinsic factors regulating RBF and renal vascular resistance, and
predicting changes in RBF and GFR from changes in sympathetic tone and
increased epinephrine
a. Intrinsic
i. autoregulation:
1. maintenance of glomerular pressure and filtration rate despite
systemic changes – because of smooth muscle surrounding afferent
vessels, which adjusts for changes in incoming pressure
a. decreased systemic pressure Æ afferents dilate
b. increased systemic pressure Æ afferents constrict
ii. macula densa-tubular glomerular feedback (TGF)
1. increases in flow through macula densa Æ vasoconstriction
2. decrease in flow through macula densa Æ vasodilation
3. this is a smooth muscle response, like in section 4/a/i/1
4. TGF mechanism is responsible for long-term control of Na+/H20
balance, and thus extracellular fluid volume maintenance
5. increase in arterial pressure Æ increased in afferent renal pressure
Æ an increase in GFR Æ increased flow to distal tubule Æ
increased chloride delivery to macula densa Æ change in osmolality
causes macula densa to release hormones Æ signals for
constriction of afferent arteriole Æ GFR is brought back down
iii. myogenic mechanism
1. increase in wall tension (passive response to elevated arterial
pressure) Æ vascular smooth muscle contraction – this is exhibited
by afferent arterioles and interlobular arteries
b. Extrinsic
i. circulating vasoactive agents or renal sympathetic nerve activity
ii. Sympathetic innervation, Hypotension or decreased sodium delivery Æ
kidney releases renin Æ cleaves hepatic angiotensinogen to angiotensin I Æ
pulmonary ACE cleaves to angiotensin II Æ systemic vasoconstriction &
causes adrenal cortex to release aldosterone Æ increased electrolyte
reabsorption Æ increased water reabsorption, sensation of thirst
iii. Sympathetic tone
1. stress, trauma, hemorrhage, pain and exercise all cause sympathetic
discharge, which increases renal vascular resistance
5. Renal vascular resistance sites, hydrostatic pressure profile
a. resistance mainly occurs in afferent and efferent arterioles, drastically reducing the
mean pressure, so: mean pressure is stable in arteries Æ reduced in afferents Æ
stable in glomerular capillaries Æ reduced in efferents Æ stable in peritubular
capillaries Æ reduced slightly in veins
b. RA = PRA – PG / RBF
c. RE = PG – PC / RBF - GFR
d. In glomerular capillaries, fluid comes out of vessels into nephronic tubular space,
because of the higher hydrostatic pressure in the capillary than in bowman’s space,
Brown, Eriksen, Jones, Heffernan, Kanjanavaikoon, Leo, Mishkin, Stern, Verlander &Wasserman
Ed. Mishkin 2006
Page 49 of 168
so there needs to be reduced glomerular pressure just so that the poor teeny
capillaries won’t explode, but pressure still must exceed that of bowman’s capsule so
that liquid will go into nephronic tubular space
e. In peritubular capillaries, liquid is being reabsorbed into systemic circulation, having
been filtered, so now the pressure needs to be even lower than in the glomerular
capillaries, because it needs to be lower than the pressure in the bowman’s capsule
so that liquid will come back into the capillaries (the math is below)
f. PG=60, PB=20, PI=6 (I is the interstitium). This all works out well because we need
the pressure in the bowman’s capsule to push out less than that of the glomerular
capillaries, so that liquid will go from the capillaries into the bowman’s capsule
g. πC=37, PC=20, so the total sucking power in the peritubular capillaries is 17. If Pc
were 60 (i.e., if there were no additional resistance in the efferent arterioles, so that
Pc=PG), then no liquid would enter the peritubular capillaries, because have you
would 37 mmHg pushing into the capillaries (πc) but 60 pushing out (PC), and no
liquid would ever re-enter circulation. So Pc is reduced to a level where the total
difference in pressures would pull into the capillaries to facilitate net reabsorption,
and sure enough, a Pc=20 will allow plenty of liquid back in.
6. Hydrostatic/colloid osmotic pressure in regulating glomerular filtration rate,
filtration barriers in the glomerular membrane, protein and macromolecular
restriction
a. Pressure question – See #5
b. Filtration barriers in glomerular membrane
i. three layers
1. inner: fenestrated endothelium – walls are 90-95% complete,
fenestrations have no diaphragams, this allows small molecules
through
2. outer: basement membrane – made of mucopolysaccharides and
proteoglycans. It is negatively charged and has pores with a radius
of 50 Å.
3. sticking out from the basement membrane: podocytes, which have
negatively charged slit pores.
4. Something about the combination of the basement membrane’s
pores and the podocyte’s pores stops macromolecules (albumin,
antibodies) from going through
c. Protein/macromolecular restriction
7. Calculating net filtration for glomerular filtration, Kf
a. GFR = Kf(PG-PB-πG) = Kf * EFP (Effective filtration pressure)
b. Kf is the filtration coefficient, decreased by:
i. hypertension
ii. diabetes
iii. glomerulosclerosis decrease
c. An increase in RA decreases RPF and GFR, but GFR goes down more
i. PG is also reduced
ii. aka, if resistance increases on the plasma’s way in, it will decrease pressure
and flow, so there will be less perfusion and also less filtration going on
d. An increase in RE decreases RPF and GFR, but RPF goes down more
i. PG is increased
ii. aka, if resistance increases on the plasma’s way out, it will decrease flow but
it will increase pressure, because more liquid is trying to get through than
can – so there will still be drops in both flow and filtration, but there will be
less of a drop in filtration because the pressure is keeping the liquid
Brown, Eriksen, Jones, Heffernan, Kanjanavaikoon, Leo, Mishkin, Stern, Verlander &Wasserman
Ed. Mishkin 2006
Page 50 of 168
pumping into filtration more than in the case of increased afferent
resistance
8. Renal autoregulation, tubuloglomerular feedback, myogenic mechanism – see
#4/a/i/2
9. RBF/GFR changes from angiotensin II, increased prostaglandin E2, increased NO,
increased renal sympathetic tone
a. angiotensin II – an increase in angiotensin II will stimulate aldosterone release,
which increases total water reabsorption, and will discourage filtration. RBF and
GFR are reduced – less liquid is coming through the kidneys and less liquid is
leaving the body, to conserve water. But GFR is reduced less than RBF, so the
filtration fraction is even higher
i. proximal tubular reabsorption is increased
ii. thirst stimulated
iii. enhances sympathetic activity
iv. arterioles (A & E) constricted
1. efferents are affected way more
v. increases sensitivity of TGF mechanism
vi. decreases Kf by reducing hydraulic conductivity of glomerular capillaries
vii. inhibits renin release Æ negative feedback
viii. (See #4/b/ii for more info on angiotensin II)
b. prostaglandin E2
i. produced from arachidonic acid via cyclooxygenase (COX)
ii. promotes renal vasodilation Æ increases RBF and GFR
iii. protective effect against vasoconstrictor stimuli, hypovolemia, hypotension
iv. Non-steroidal anti-inflammatory drugs (N-SAID’s) are COX inhibitors Æ
reduce prostaglandins Æ reduces opposition to vasoconstriction Æ
decrease RBF, GFR and Na excretion
c. NO
i. cGMP Æ vasodilation Æ increases RBF and GFR
ii. preglomerular and postglomerular arterioles are both responsive to NO Æ
blockade GFR decreases less than RBF
d. renal sympathetic tone – see # 4/b/iii
i. tonic sympathetic innervation on renal hemodynamics is minimal
ii. low level -- increases renin release, increases tubular Na reabsorption
iii. moderate levels – increase afferent and efferent arteriolar resistance,
reduces both GFR and RBF – but RBF is reduced more than GFR
iv. high level
10. Changes in tubular reabsoprtion association with pressure changes in peritubular
capillaries – see #5

Clearance principle and assessment of renal function

1. Derive clearance equation, be able to determine clerances for creatinine, inulin, p-


amino hippuric acid
a. Clearance is an evaluation of nephronic handling of a given substance
i. clearance is the minimum amount of liquid that must pass through into the
bowman’s space in order to account for the amount of that substance in the
urine
b. For a substance S
i. Clearance = amount of S excreted in urine / plasma concentration of S
ii. amount of S excreted in urine = urine flow X urine concentration of S
Brown, Eriksen, Jones, Heffernan, Kanjanavaikoon, Leo, Mishkin, Stern, Verlander &Wasserman
Ed. Mishkin 2006
Page 51 of 168
c. If S is something that is completely passed (i.e. p-amino hippuric acid or PAH),
clearance = renal plasma flow
d. If S is something that is completely reabsorbed (such as glucose), clearance = 0.
e. If S is neither completely passed nor reabsorbed, some of the time we can use the
equation below – In using this equation we are making a basic assumption that the
percentage of S that passes (from the plasma into the Bowman’s capsule) will be
approximately equivalent to the percentage of plasma that comes through into the
Bowman’s capsule. This is perfectly true for S= creatinine, inulin, and some other
compounds, but the formula can also be used as an estimate for the passage of
many other substances that have a similar filtration pattern (i.e., fit the criteria in
1/e/ii).
i. Equation: GFR = UF*SU/SP
1. Translation: Glomerular filtration rate = Urine flow *
Concentration in the Urine / Concentration in the plasma
ii. Criteria to use this equation (these are characteristics of creatinine and
inulin):
1. excreted via filtration
2. filtered without restriction
3. no net reabsorption or secretion
4. not metabolized anywhere in the tubules
5. Æ amount excreted = amount filtered
iii. Derivation:
1. Amount filtered = GFR X SBS
2. Amount excreted = UF X SU
3. GFR X SBS = UF X SU
4. SBS = SP
5. GFR X SP = UF X SU
6. GFR = UF*SU/SP
7. QED ;)
2. Explain which clearances represent glomerular filtration rate (GFR), and renal
plasma flow (RPF) and explain criteria
a. GFR is represented by creatinine and inulin
i. because they fit all of the criteria in 1/e/ii, which basically means that the
amount being filtered and the amount being excreted in urine are the same
b. RPF is represented by PAH
i. This is because all of the PAH that comes through the kidney comes out of
the body. So the total amount of plasma flowing through the kidney can be
estimated by the amount of PAH that comes out of the body.
3. Given plasma and urine concentrations and urine flow, calculate filtered load,
tubular transport, excretion rate and clearance for a substance S
a. Filtered load, FL = GFR*SP
b. Tubular transport = FL – excretion rate
i. we think this is the same thing as “net reabsorption,” or the amount
reabsorbed across the tubules
c. Excretion rate = Urine concentration * Urine formation rate
d. Clearance = tell your mom to see 1/b
e. Explain how changes in filtration, reabsorption and secretion will affect excretion
i. Increased filtration Æ increase excretion
ii. Increased reabsorption Æ decrease excretion
iii. Increased secretion (i.e., from efferent capillaries back into tubules) Æ
increase excretion
4. Determine net tubular reabsorption vs. secretion by comparing clearance to GFR
Brown, Eriksen, Jones, Heffernan, Kanjanavaikoon, Leo, Mishkin, Stern, Verlander &Wasserman
Ed. Mishkin 2006
Page 52 of 168
a. if clearance < GFR – that means you have net reabsorption (for a given S)
b. if clearance > GFR – that means you have net secretion (for a given S)
c. transport maximum aka tubular maximum: the maximal rate of secretion or
reabsorption of a substance by the renal tubules. Glucose, for example, has a
tubular maximum for reabsorption – 375mg/min. Glucose will be excreted into the
urine (glucosurea) if blood glucose is above 300mg/dl.

Proximal tubule Transport mechanisms

1. Basic principles of epithelial transport, characteristics of active transport, facilitated


diffusion, passive diffusion with regard to energy source and carrier protein
involvement
a. Reabsorption is achieved via both active and passive transport
b. active transport uses energy and goes against the concentration gradient
i. primary needs ATP directly
1. in tubular cells, occurs at basolateral membrane
ii. secondary uses concentration gradient produced by primary
1. in tubular cells, occurs at apical membrane
2. in this particular case, efflux (from tubular cell to peritubular fluid)
of sodium in primary transport is actually facilitating influx of
sodium (from tubular lumen into tubular cell) in secondary
transport by reducing the concentration inside the cell and
therefore changing the concentration difference between the inside
of the cell and the lumen of the tubule
iii. active is subject to limitations:
1. metabolic inhibition – ATP is required, therefore a depletion in
ATP will lead to decreased ability to pump against a gradient
2. competitive inhibition – amino acids, for example, can compete for
the same protein carrier
3. transport maximum (Tmax) – the rate of reabsorption for any
substance whose transport is active may have a limit. For example,
reabsorption of glucose is by secondary active transport via a
sodium glucose cotransporter. The number of sodium-glucose
carriers is limited, so at plasma glucose concentrations > 350 mg%,
carriers are saturated – making this amount the Tmax.
c. facilitated diffusion works down a concentration gradient but requires a carrier
protein
Brown, Eriksen, Jones, Heffernan, Kanjanavaikoon, Leo, Mishkin, Stern, Verlander &Wasserman
Ed. Mishkin 2006
Page 53 of 168
d. passive diffusion does not require either energy nor a carrier protein
2. Contribution of proximal tubule and other major nephron segments to the
reabsorption of the filtered load of water and solute
a. proximal tubule is responsible for most of the reabsorption – hence it is reffered to
as the “bulk reabsorber.” It is the location of the reabsorption of:
i. 67% NaCl reabsorbed
ii. 100% glucose
iii. 80-85% HCO3
iv. lots of phosphate, sulfate, lactate, citrate, succinate (the organic anions)
b. descending limb of henle:
i. no active NaCl transport
1. but there is passive transport
ii. high permeability to water
iii. some permeability to urea and NaCl
c. thin ascending limb of henle:
i. completely impermeable to water
ii. some permeability to urea and NaCl
d. thick ascending limb of henle
i. completely impermeable to water
ii. Has active transport of NaCl out of the tubule Æ creates the driving force
for Na/K/2Cl cotransporters (which bring 1 Na, 1 K and 2 Cl into the
tubular cell from the lumen of the tubule)
1. this provides K inside the cell, which will leave down its own
gradient, creating an electrochemical difference (negative inside the
cell), creating a positive charge outside the cell, which will push
luminal sodium away (positive charge repelling) and into the blood
(it travels between the cells of the tubule in order to get from
lumen to blood). This is called “paracellular diffusion,” being that
it is diffusion occuring next to a cell.
3. Major transport functions of proximal tubule with regard to sodium, chloride,
potassium, bicarbonate, hydrogen phosphate, glucose, amino acids and urea
a. Sodium – See #2
b. Chloride – See #2
i. Chloride is not absorbed early on in the proximal tubule, which means that
as fluid is reabsorbed (as you go distally) chloride concentration increases,
therefore in the distal portion of the proximal tubule, chloride diffuses into
the tubular cells, down its concentration gradient. The negative charge that
the chloride brings with it creates a potential difference between the lumen
and the tubular cells, which is used in passive distal reabsorption of cations
c. Potassium
i. Passively reabsorbed in late proximal tubule
d. Bicarbonate
i. ATPase pumps Na out and K in
ii. Na/H antiporter pumps Na in and H out
iii. H in the tubular lumen combines with HCO3 Æ H2CO3 (carbonic acid)
iv. H2CO3 Æ CO2 + H20 via carbonic anhydrase
v. water passively diffuses across tubular cells and back into peritubular fluid
vi. the carbon dioxide diffuses into tubular cells, where it is hydroxylated back
into HCO3 by cytosolic carbonic anhydrase (again)
vii. Now bicarbonate can be cotransported with sodium out of the tubular cells
into the peritubular space
e. Hydrogen phosphate
Brown, Eriksen, Jones, Heffernan, Kanjanavaikoon, Leo, Mishkin, Stern, Verlander &Wasserman
Ed. Mishkin 2006
Page 54 of 168
i. 85% of filtered phosphate is reabsorbed in the proximal tubule by a sodium
phosphate cotransporter
f. glucose
i. 100% of reabsorption is in the proximal tubule by a sodium cotransporter
g. amino acids
i. 100% of reabsorption is in the proximal tubule by a sodium cotransporter
h. Urea
i. 100% of reabsorption is in the proximal tubule
ii. 50% of filtered urea is reabsorbed passively in the proximal tubule
4. Generation of tubular osmotic forces
a. As sodium is reabsorbed from the tubular lumen, across the tubular cells and into
the renal interstitium, the concentration in the renal interstitium becomes higher
than that in the tubular lumen, which pulls water out of the lumen and into the
peritubular space
b. luminal hypotonicity: In the proximal tubule, osmolality is 275-280mOsm/kg. The
proximal tubule is very permeable to water, therefore water will flow out to meet
higher ion concentration. Originally, it was thought that the lumen was isotonic,
and though this has turned out not to be quite true, this flow is still referred to
erroneously as “isoosmotic reabsorption”
c. Axial Anion Assymetry: As you travel distally down the proximal tubule, the
concentration of biocarbonate decreases and the concentration of chloride
increases. In the late proximal tubule, the reflection coefficient (σ) is high for
bicarbonate and low for chloride. The reflection coefficient is the degree to which
something cannot traverse a membrane, so the high σ of HCO3 means that it cannot
diffuse back into the lumen freely, and the low σ of Cl means that it can diffuse back
into the lumen freely. Therefore there will effectively be a significant concentration
gradient for HCO3 but not for Cl.
i. HCO3 gradient that occurs accounts for 90% of total water
reabsorption
5. Flow-dependent regulation of proximal tubular reabsorption underlying
glomerulotubular balance
a. Glomerulotubular balance is based on starling forces in peritubular capillaries, which
alter the reabsorption of sodium and water in the proximal tubule. The route of
isoosmotic fluid reabsorption is from the lumen, through the tubular cells, to the
renal interstitum and into the peritubular capillaries.
b. Starling forces in the peritubular capillary blood govern the amount of reabsorption
of isoosmotic fluid
c. Starling forces for those of you just joining us (like us) are the summation of sucking
in and pushing out pressures across a membrane (P’s and π’s)
d. See “Renal hemodynamics and regulation of glomerular filtration rate” part 5 g
e. The more fluid we send into the tubule, the more concentrated the efferent blood
will be, and therefore the more fluid will have to re-enter the peritubular capillaries
from the tubules in order to achieve physiological osmolality in the blood

Loop of Henle and countercurrent system

1. Describe and contrast the mechanisms of solute and water transport by the
descending segment of the loop of Henle
a. NaCl is passively transported but not actively
b. high permeability to water
c. some permeability to urea and NaCl
Brown, Eriksen, Jones, Heffernan, Kanjanavaikoon, Leo, Mishkin, Stern, Verlander &Wasserman
Ed. Mishkin 2006
Page 55 of 168
2. Explain how transport and permeability characteristics of the descending and
ascending segments of the loop of Henle enable the kidney to produce a
concentrated urine
a. See Proximal tubule transport mechanisms #2/c & d
b. Descending limb Æ
i. water out of lumen Æ increases concentration
ii. urea into lumen
iii. NaCl – only slight permeability
c. Ascending thin limb Æ
i. Sodium and chloride ions diffuse out into interstitium
d. Thick ascending limb
i. pumping chloride into interstitium
ii. passive follow of sodium into interstitium
iii. no water permeability Æ no diffusion out
iv. solute concentration in interstitium has increased
e. distal convoluted tubule
i. now permeable to water again, water goes out to meet ions
ii. urea is left inside distal convoluted tubule with fewer ions and less water Æ
urine has a concentration of urea that is much higher than the blood
concentration of urea, but urine does not have as much water or as many
ions as does the blood
3. How renal tubular handling of urea contributes to concentrated urine production
a. See #2
4. Process responsible for dilute urine production, and effects of changes in vasopressin
(aka anti-diuretic hormone or ADH) levels
a. Water deprivation Æ increased plasma osmolarity Æ stimulates osmoreceptors in
anterior hypothalamus Æ ADH secretion from posterior pituitary Æ increased
water permeability of distal tubule and collecting duct Æ increased water
reabsorption Æ increased urine osmolarity & decreased urine volume & decreasing
plasma osmolarity
b. Water abundance Æ decreased plasma osmolarity Æ inhibits osmoreceptors in
anterior hypothalamus Æ suppression of ADH Æ decreased water permeability of
distal tubule and collecting duct Æ decrease water reabsorption Æ decreased urine
osmolarity, increased urine volume, increased plasma osmolarity
i. High-ceiling diuretics such as furosemide (Lasix) bind to the Na/K/2Cl
transporter and inhibit ionic influx. If you can’t pump ions out of the loop
of Henle, water won’t follow them, and urine output will increase and have
a lower concentration of urea, as well as a higher concentration of water
and electrolytes
5. No LO, but “Counter currents” seemed pretty important to address, considering
their place in the title of the lecture:
a. Countercurrent just means that there is a semipermeable membrane with fluid on
either side and that there is exchange of properties (in this case, concentrations)
between the two fluids
b. Loop of henle acts as a countercurrent mulitplier: this is the phrase for the long-
described above phenomenon that concentrates urine
i. Countercurrent mulitplication depends on NaCl reabsorption in the thick
ascending limb and countercurrent flow in both descending and ascending
limbs of Henle
Brown, Eriksen, Jones, Heffernan, Kanjanavaikoon, Leo, Mishkin, Stern, Verlander &Wasserman
Ed. Mishkin 2006
Page 56 of 168
c. Vasa recta – countercurrent exchanger (these are the blood vessels right by the loop
of Henle, so they’re obviously absorbing some of the fluid and ions coming into the
cells of the walls from the lumen)
d. Countercurrent arrangement – I think this is just how everything is sitting together
in it’s little limbed, hair-pinned way that lets countercurrent stuff occur.

Distal tubule and collecting duct

1. Mechanisms of solute and water transport by distal tubule and collecting duct
segments
a. 25% of filtered NaCl is reabsorbed in the loop of Henle
b. 8-9% of filtered NaCl is reabsorbed in the distal tubule and collecting duct
i. this amount is smaller, but this is the part that’s most regulated, so it’s
actually the part that determines how much reabsorption will take place
c. Early distal tubule
i. Distal convoluted tubule cells
ii. impermeable to water
iii. NaCl reabsorption via cotransporter but no reabsorption of water Æ
dilution of tubular fluid
iv. “cortical diluting segment”
d. Late distal tubule (aka connecting tubule)
i. Two cell types:
1. connecting tubule cells –
a. reabsorb NaCl and water
b. secrete potassium
c. Aldosterone increases sodium reabsorption and potassium
secretion
d. ADH increases water permeability (reabsorb more water)
2. intercalated cells –
a. secrete H+ by a H+-ATPase, which is stimulated by
aldosterone
b. reabsorbed K+ by a H+/K+-ATPase (from BRS; not
stressed in lecture)
e. Collecting duct
i. distal tubules of several nephrons Æ confluence into 1 collecting duct
ii. water reabsorption occurs in presence of ADH
iii. two types of cells:
1. NaCl is transported by principal cells
2. intercalated cells
iv. Collecting duct goes through cortical and medullary renal layers just as the
loop of henle does
v. cortical collecting tubule
1. water leaves
2. main function is to raise both fractional and absolute luminal
concentration of urea
vi. outer medullary collecting tubule
1. water leaves
2. raises absolute concentration of urea
vii. inner medullary collecting tubule
1. water leaves
2. urea concentration is now high so some urea also leaves
Brown, Eriksen, Jones, Heffernan, Kanjanavaikoon, Leo, Mishkin, Stern, Verlander &Wasserman
Ed. Mishkin 2006
Page 57 of 168
a. this helps maintain the high concentration of urea in the
inner medulla Æ urea recycling: when urea is reabsorbed
from the collecting duct, it enters the inner medullary
portion of the loop of Henle. This is necessary in order
to maintain the high urea concentration of the inner renal
medulla, because if the urea there were not being
replenished, it would soon be depleted by the inner
medullary portion of the loop of Henle, into which urea
diffuses. A high concentration in the inner medulla is
important because it makes water diffuse out of the
lumen, concentrating the urine in the loop of Henle.
b. ADH increases permeability of the inner medullary
collecting tubule to urea, allowing more urea to be
reabsorbed into inner medullary interestitium
3. maximum concentration is achieved here
2. Factors modulating distal tubule and collecting duct transport function
a. Thiazide diuretics (like hydrochlorothiazide) inhibit the transporter that symports
sodium and chloride from the lumen into the cells of the early distal tubule
b. amiloride inhibits sodium reabsorption and therefore NaCl reabsorption in the
apical membrane of principal cells of late distal tubules
i. therefore also inhibits K+ secretion – therefore “potassium-sparing”
1. other potassium-sparings are spironolactone & triamterene
c. If furosemide is given upstream of the DCT it’s a potassium-wasting diuretic
i. it inhibits NaCl reabsorption in the thick ascending limb of henle Æ
absorbing Na+ makes you secrete K+
ii. If you’re on furosemide, you take a separate K+ supplement
d. Sodium reabsorption in late distal tubule and collecting duct:
i. tubular flow causes sodium to go from the lumen into the cells Æ
potassium goes from cells into lumen
1. an increase in sodium flow increases sodium, chlorine and water
reabsorption and potassium secretion
ii. an increase in plasma potassium concentration increases sodium
reabsorption
iii. aldosterone – a mineralocorticoid secreted by adrenal cortex Æ mRNA Æ
aldosterone-induced proteins Æ
a. Na+/K+ pumps
b. increase in krebs enzymes Æ feed ATP into Na+/K+
pump
2. increases sodium reabsorption and potassium secretion
a. induces apical Na channels
b. induces basolateral Na+/K+ pumps
c. therefore sodium gets pulled from lumen into cell and
then out into interstitium
3. Tubular segments and cellular mechanism by which ADH increases permeability to
water and urea
a. ADH increases water permeability by directing the insertion of water channels into
the luminal membrane. In the absence of ADH, principal cells in the Late distal
tubule and collecting duct are virtually impermeable to water
b. V1 receptors are on vascular smooth muscle cells and arterioles
i. ADH binding causes vasoconstriction, decreases in RBF and GFR
ii. Mechanism is IP3/Ca-mediated
Brown, Eriksen, Jones, Heffernan, Kanjanavaikoon, Leo, Mishkin, Stern, Verlander &Wasserman
Ed. Mishkin 2006
Page 58 of 168
c. V2 receptors are on the basolateral membranes of Late distal tubule and collecting
duct
i. cAMP Æ PKA Æ phosphorylation results in the insertion of aquaporins
in the apical membranes (aquaporin II, specifically)
4. Two major mechanisms controlling ADH release and negative feedback
a. ADH originates primarily in the supraoptic nuclei of the hypothalamus
b. Factors that increase ADH secretion:
i. an increase in serum osmolarity (main mechanism)
1. Water deprivation Æ increases serum osmolarity Æ stimulates
osmoreceptors in anterior hypothalamus Æ increases ADH
secretion from posterior pituitary Æ increases water permeability
of late distal tubule and collecting duct Æ increases water
reabsorption Æ increases urine osmolarity and decreases urine
volume, decreases plasma osmolarity toward normal
ii. volume contraction = decrease in blood volume
1. decreases inhibition of osmoreceptors, increasing their activity,
increases the excitability of the osmolarity mechanism
iii. pain
iv. nausea
v. hypoglycemia
vi. nicotine, opiates, antineoplastic drugs
c. Factors that decrease ADH secretion:
i. a decrease in serum osmolarity
1. Water intake Æ decreases plasma osmolarity Æ inhibits
osmoreceptors in anterior hypothalamus Æ decreases ADH
secretion Æ decreases DCT/CD permeability to water Æ
decreases water reabsorption Æ decreases urine osmolarity and
increases urine volume and increases plasma osmolarity toward
normal
ii. ethanol
iii. alpha-agonists
iv. ANP
d. It takes a 15% change in blood volume to result in a change in vasopressin release
(in either direction), but only a 2% change in plasma osmolarity. Despite the fact
that the system is more sensitive to plasma osmolarity, ultimately a change in blood
volume above the 15% threshold will override changes in plasma osmolarity if the
two have opposing effects. In other words, your body won’t care that you’re not
sweating if you just hemorrhaged.

Renal Hormones

1. Identify the Renal Hormone Systems Regulating Renal Function


a. The major renal hormone system is the Renin-Angiotensin-Aldosterone System,
which is essential for Na+ and H2O balance
i. Decreases in blood volume cause a decrease in renal perfusion pressure,
which in turn increases renin secretion.
ii. The enzyme renin is produced by the Juxtaglomerular (JG) cells of the
afferent arteriole
iii. Renin catalyzes the conversion of systemically circulating angiotensinogen
to angiotensin I, which is then converted to angiotensin II (ANG II) by
angiotensin converting enzyme (ACE) which is present in the highest
Brown, Eriksen, Jones, Heffernan, Kanjanavaikoon, Leo, Mishkin, Stern, Verlander &Wasserman
Ed. Mishkin 2006
Page 59 of 168
quantities in the pulmonary endothelial cells, and then in the kidneys, but it
present on endothelial cells throughout the body. In the kidneys ACE is
located in the interstitium and the basolateral membrane of the proximal
tubule (PT) cells
iv. ANG II is the biologically active octapeptide which then acts on various
membrane receptors, resulting in vasoconstriction, and increasing the total
peripheral resistance and elevating the blood pressure back to normal
v. ANG II also elicits Aldosterone release from the zona glomerulosa of the
adrenal cortex.
vi. Aldosterone release causes increased renal Na+ reabsorption, thereby
restoring extracellular fluid (ECF) volume and blood volume to normal
vii. The actions of Aldosterone include:
1. ↑ Renal Na+ reabsorption (action on the principal cells of the late
distal tubule and collecting duct)
2. ↑ Renal K+ secretion (action on the principal cells of the late distal
tubule and collecting duct)
3. ↑ Renal H+ secretion (action on the intercalated cells of the late
distal tubule and collecting duct)
b. Renal prostaglandins (PGs), also play a critical role in regulating renal function
i. Cyclooxygenase (COX) converts arachidonic acid into different classes of
prostaglandins (or autocoids, since they act at site of production)
ii. COX 1 and COX 2, both eventually convert arachidonic acid to PGI2 and
PGE2 (as well as Thromboxane 2, TXA2)
iii. PGE2 and PGI2 are potent vasodilators. Both ↑ RPF and ↓H2O
reabsorption by the collecting duct
1. PGE2 acts on glomeruli, thick ascending limb, and collecting duct
2. PGI2 acts on thick ascending limb, and collecting duct
3. PGE2 and PGI2 inhibit the action of ADH on LDT and CD,
thereby ↓ water reabsorption by the LDT/CD
4. Under normal volume conditions, there is no substantial PG effect.
However, When ECFV is compromised and vasoconstricting
mechanisms are activated, then PGs play a crucial role in Na+ and
H2O balance
iv. ECFV depletion resulting from ↓ Na+ intake, ↓ Blood Pressure, or intense
dehydration causes ↑ PGE2 and PGI2 production
1. PGs counteract the vasoconstriction of ADH and ANG II
2. COX-Inhibitors (e.g. Vioxx), prevent the formation of PGs,
leading to increased unopposed vasoconstriction, which can result
in renal failure with chronic use or in a patient whose ECF is
already depleted extensively
c. Bradykinin, is another potent hormone that promotes vasodilation
i. Bradykinin is produced from a cascade of transformations, taking Renal
Prekallikrein to Bradykinin; all of which occurs in the LDT
ii. Bradykinin opposes the actions of ANGII, and ↑ NaCl excretion and ↑
H2O excretion
iii. Bradykinin also stimulates PG production (PGE2 and PGI2)
iv. Renal PGs and Bradykinin are potent renal vasodilators, leading to ↓ Na+
reabsorption and ↓ H2O reabsorption, which antagonizes the
vasoconstriction by ADH and ANG II
Brown, Eriksen, Jones, Heffernan, Kanjanavaikoon, Leo, Mishkin, Stern, Verlander &Wasserman
Ed. Mishkin 2006
Page 60 of 168
d. Erythropoietin (EP) is important in the generation of RBCs and ultimate
sustenance for the kidneys themselves
i. 90% of EP is synthesized in the kidneys
ii. EP stimulates Hematopoietic stem cells to become proerythroblasts (that
will ultimately become RBCs)
iii. ↓ O2 delivery to the kidney, results in ↑ EP synthesis, leading to ↑ RBC
synthesis, ↑ Hematocrit, ↑ability to deliver O2 to kidneys
iv. Chronic ↓ O2 delivery to the kidney resulting from ↓ blood volume or
chronic anemia, leads to chronic renal vasoconstriction, and eventual renal
failure
1. can also result from constriction of the afferent renal arteriole
v. Renal failure/Dialysis patients cannot produce their own EP, so they
receive injections
2. The major stimuli that influence regulation of these renal hormone systems:
The most important regulatory factors are ECF balance and blood volume. These renal hormone
systems have the ability to preserve normal ECFV homeostasis and adequate arterial pressure. There
are 3 mechanisms that act to regulate cAMP levels and intracellular [Ca2+] of the JG cells of
the afferent arterioles in response to changes in ECF balance and blood volume.
a. Baroreceptor mechanism: JG cells of the afferent arteriole act as baroreceptors
and respond to changes in transmural pressure gradient between the afferent
arteriole and interstitium
i. Afferent arteriole acts as a stretch receptor: Increased stretch leads to
decreased renin secretion
ii. Increased renal arterial pressure, increased stretch of afferent arteriole,
which increases JG cells’ membrane stretch, which causes stretch-activated
Ca2+ channels in the JG membrane to allow Ca2+ to rush down its
concentration gradient into the JG cell cytoplasm.
iii. Increased intracellular [Ca2+], decreases renin secretion – since renin
promotes constriction, a decrease in renin allows dilation, lowering blood
pressure and bringing flow back down towards normal
iv. Conversely, if renal arterial pressure is decreased, there is decreased stretch
on the afferent arteriole and decreased JG membrane stretch, leading to
decreased intracellular [Ca2+] and increased in renin secretion
b. Sympathetic Innervation
i. Proximal tubule (PT) and distal tubule (DT) cells are innervated by
sympathetic efferent fibers.
ii. JG cells are also innervated by sympathetic efferents that release
Norepinephrine (NE), that acts on β1-adrenergic receptors of the JG cells,
leading to increased intracellular [cAMP] and a consequent increase in renin
secretion
1. For example, a decrease in renal arterial pressure Æ activates
increased sympathetic tone Æ increased NE release Æ increased
cAMP Æ increased renin secretion Æ vasoconstriction Æ return
in pressure towards normal
c. Macula Densa (MD) mechanism involves increasing or decreasing Renin secretion
(distinct from the Tubuloglomerular feedback mechanism in which MD cells
communicate with the afferent arteriole to elicit vasoconstriction)
i. Macula densa cells are located between the thick ascending loop of Henle
and the distal convoluted tubule
ii. Macula densa cells function as chemoreceptors and are stimulated by
decreased NaCl load
Brown, Eriksen, Jones, Heffernan, Kanjanavaikoon, Leo, Mishkin, Stern, Verlander &Wasserman
Ed. Mishkin 2006
Page 61 of 168
iii. Any stimulus that increases flow through the thick ascending limb of the
loop of Henle, leads to the increased delivery of tubular fluid Na+ to the
macula densa cells of the JG apparatus
iv. Increased Na+ delivery (from increased GFR, resulting from increased
blood pressure), results in decreased renin secretion to release
vasoconstriction of systemic vessels causing decreased blood pressure,
decreasing GFR
1. ECFV expansion
a. Increases blood volume Æ increasing blood pressure Æ
increasing renal blood flow Æ increasing GFR Æ
increasing the Na+ delivery to the MD cells.
i. Na+ reabsorption requires ATP, thus more Na+
reabsorption (from increased GFR) leads to a
depletion of ATP, increasing the concentration of
Adenosine.
ii. Adenosine acts on JG cells increasing intracellular
[Ca2+] and decreasing Renin secretion
2. ECFV depletion
a. Decreases BV, decreasing BP, decreasing RBF, decreasing
GFR. Decreases Na+ delivery to MD cells, leading to an
increase in renin secretion
3 & 4 Summary of major effects on renal function and interaction of hormone systems
d. Renin-Angiotensin System
i. Potent vasoconstrictor: increases cardiac contractility, increases ADH
release, increases sympathetic tone, increases vasoconstriction, increases
NE release, increases Aldosterone release
ii. ANG II has direct vascular and transport effects
1. ANG II constricts both afferent and efferent renal arterioles,
leading to decreased RPF and an increased Filtration Fraction (FF),
leading to increased proximal tubule fluid reabsorption
2. Constricts the renal mesangial cells, leading to a decrease in the
filtration coefficient (Kf), which if severe enough can decrease GFR
(GFR = Kf ⋅ EFP)
3. Direct stimulation of proximal tubule reabsorption of NaCl, HCO3-
and H2O
e. Aldosterone, release stimulated by ANG II, causes increased NaCl and H2O
reabsorption to increase BV and hence increase BP
f. Prostaglandins, promote vasodilation to offset the vasoconstriction of ADH and
ANGII
g. Bradykinin also promotes vasodilation to antagonize ADH and ANGII
vasoconstriction
h. Erythropoietin enables increased O2 delivery, elicited by decreased renal perfusion
and O2 delivery

The Kidney in Blood Pressure Regulation and Pathophysiology of Hypertension

1. Major neurohumoral and cardiovascular mechanisms responsible for long-term


regulation of arterial blood pressure:
To maintain blood volume (and hence blood pressure) within a normal range, the kidneys regulate
the amount of water and sodium lost via the urine. There are several mechanisms, which maintain
sodium balance, extracellular fluid volume, and blood volume. This is important because
hypertension can lead to strokes, MI, renal dysfunction, vision loss, and endocrine disease.
Brown, Eriksen, Jones, Heffernan, Kanjanavaikoon, Leo, Mishkin, Stern, Verlander &Wasserman
Ed. Mishkin 2006
Page 62 of 168
a. Neurohumoral Systems (this is basically all review)
i. Arterial Reflexes
1. Myogenic autoregulation; vasoconstriction in response to increased
pressure, vasodilation in response to decreased pressure
2. Tubuloglomerular feedback mechanism; increased blood pressure Æ
increased RBF Æ increased NaCl delivery to macula densa Æ
vasoconstriction of afferent arteriole to decrease RBF
ii. Atrial Reflexes
1. Increased atrial stretch due to increased blood volume, leads to
vasodilation
iii. Renin-Angiotensin-Aldosterone System
1. Vasoconstriction and increased Na+ and H2O reabsorption
iv. Adrenal Catecholamines
1. Vasoconstriction
v. Vasopressin (Anti-diuretic Hormone, ADH)
1. Increased Na+ and H2O reabsorption
vi. Atrial Natriuretic Peptide/Factor (ANP/ANF)
1. increased blood volume Æ increased blood pressure Æ atrial stretch Æ
Vasodilation
2. Increased Na+ excretion by inhibiting Na+ reabsorption in collecting
tubule cells
vii. Endothelial Factors (Nitric Oxide, Endothelins)
1. Vasodilation in response to increased pressure and flow
viii. Kallikrein-Kinin System
1. Leads to Bradykinin production; vasodilation
ix. Prostaglandins & Other Eicosanoids
1. Vasodilation to offset the vasoconstriction of ADH/ANG II
b. Cardiovascular mechanisms
i. The two major determinants of arterial pressure: 1) cardiac output and 2)
peripheral resistance
ii. On a long-term basis
1. mean circulatory pressure and blood volume depend on sodium
balance which is regulated by the kidneys
2. arterial pressure is linked to the ability of the kidneys to excrete
sufficient salt to maintain normal sodium balance, extracellular fluid
volume and blood volume
2. Normal range of dietary sodium intake and the relationships between sodium balance,
plasma volume and CV hemodynamics: Salt concentration is regulated, so an increase in
salt results in an increase in water reabsorption.
a. Normal Range of Dietary Sodium Intake
i. Many people consume 1,150- 5,750 mg/day -- that is termed the "hygienic
safety range" of sodium intake1
b. Plasma Volume changes with increases in salt intake
i. Increased salt intake increased plasma volume Æ increasing blood pressure
ii. The maintenance of blood pressure depends on the ability to get rid of the extra
salt by a variety of neurohumoral mechanisms
a. Decreases in sodium excretory capability Æ chronic
increases in ECFV and BV which result in hypertension
c. The integration of sodium exchange Æ sodium ECF level

1 “Salt and Health” http://www.saltinstitute.org/28.html


Brown, Eriksen, Jones, Heffernan, Kanjanavaikoon, Leo, Mishkin, Stern, Verlander &Wasserman
Ed. Mishkin 2006
Page 63 of 168
i. Quantity of NaCl in ECF / Extracellular Fluid Volume =
Na+ and Cl- concentrations in ECF
ii. Increased NaCl intake Æ Increased NaCl in ECFV Æ ADH release Æ
Concentrated urine (increased free water reabsorption) and thirst (increased
water intake)
iii. Decreased NaCl intake Æ Decreased NaCl in ECFV Æ ADH inhibition Æ
Dilute urine (increased solute-free water excretion)
d. Osmolality is rapidly regulated by adjusting the ECF volume to the total solute present
i. [NaCl] in ECF is regulated by an indirect measurement of volume:
1. Increased NaCl intake Æ increase ECFV Æ stretch receptors in CV Æ
Therefore volume and not concentration is the effective measured
variable.
3. NaCl regulation along the nephron
a. Proximal Tubule: reabsorbs 2/3 of filtered Na and H2O
i. Early proximal tubule, basolateral Na+-K+ ATPase, decreases cellular [Na+]Æ
for Na+ reabsorption
1. Na+ is reabsorbed by cotransport with glucose, amino acids, phosphate,
and lactate
2. Na is also reabsorbed by countertransport via Na+ - H+ exchange,
which is linked directly to the reabsorption of filtered HCO3-
ii. Middle and late proximal tubules
1. Na+ is reabsorbed with Cl-
iii. Stimulation of Na+ reabsorption
1. ANG II via Aldosterone
2. Adrenergic agents of increased renal nerve activity
3. Increased luminal flow or solute delivery
iv. Inhibition of Na+ reabsorption
1. Volume expansion
2. Atrial Natriuretic peptide
3. Dopamine
4. Prostaglandins
b. Thick Ascending limb of the loop of Henle: reabsorbs 25% of filtered Na+
i. Contains a Na+-K+-2Cl- cotransporter in the luminal membrane
ii. Is the site of action of the loop diuretics (eg. furosemide), which inhibit the
Na+-K+-2Cl- cotransporter
iii. Is impermeable to water Æ NaCl is reabsorbed without water Æ tubular fluid
[Na+] and tubular fluid osmolarity decrease to less than their plasma
concentrations. This segment is called the diluting segment
iv. Na+-K+ ATPase Æ increases intracellular [K+] Æ lumen-positive potential
difference, which raises driving force of K+ secretion Æ driving force for the
Na+-K+-2Cl- cotransporter and paracellular diffusion of cations from the tubule
lumen to the pericapillary fluid
v. Stimulation of Na+ reabsorption
1. ADH
2. β-Adrenergic agents
3. Mineralocorticoids (aldosterone)
vi. Inhibition of Na+ reabsorption
1. Hypertonicity
2. Prostaglandin E2
3. Acidosis
c. Distal Tubule and Collecting Duct: together reabsorb 8% of filtered Na+
Brown, Eriksen, Jones, Heffernan, Kanjanavaikoon, Leo, Mishkin, Stern, Verlander &Wasserman
Ed. Mishkin 2006
Page 64 of 168
i. Early distal tubule
1. Reabsorbs NaCl by a Na+-Cl- cotransporter
2. Is the site of action of thiazide diuretics which block the Na+-Cl-
cotransporter – on apical membrane
3. Is impermeable to water Æ reabsorption of NaCl occurs without water,
which further dilutes the tubular fluid
4. Is called the cortical diluting segment
ii. Late distal tubule and collecting duct: have 2 cell types
1. Principal Cells
a. Reabsorb Na+ and H2O
b. Secrete K+
c. Aldosterone increases Na+ reabsorption and K+ secretion.
i. Like other steroid hormones, the action of aldosterone
takes hours to develop because new protein synthesis
is required
d. ADH increases H2O permeability by directing the insertion of
H2O channels in the luminal membrane. In the absence of
ADH, the principal cells are virtually impermeable to water
e. K+-sparing diuretics (spironolactone, triamterene, amiloride)
decrease K+-secretion
i. Is the site of action of amiloride diuretics which block
the apical Na+ channel Æ blocking sodium influx
results in decreased Cl influx and K efflux
1. because K efflux (i.e., secretion) is decreased,
amiloride is “potassium-sparing”
2. α-Intercalated cells
a. Secrete H+ by a H+-ATPase, which is stimulated by
Aldosterone
b. Reabsorb K+ by a H+,K+-ATPase (not emphasized in class)
3. Stimulation of Reabsorption
a. Aldosterone
b. ADH
4. Inhibition of Reabsorption
a. Prostaglandins
b. Nitric Oxide
c. Atrial Natriuretic Peptide
d. Bradykinin
4. The Renin-Angiotensin-Aldosterone System – Key in regulating blood pressure and sodium
excretion
a. Decrease in blood volume Æ decrease in renal perfusion pressure Æ increase renin
secretion: Renin, catalyzes [angiotensinogen Æ angiotensin I], angiotensin I is converted
to angiotensin II by angiotensin-converting enzyme (ACE)
b. ANG II
i. Constricts resistance vessels (predominately via AT1 receptors in the adult)
thereby increasing systemic vascular resistance and arterial pressure
ii. Activates adrenal cortical release of aldosterone
1. Aldosterone causes kidney to increase sodium and fluid retention
2. Aldosterone activates mineralocorticoid receptors in collecting duct
cells to increase production of Na+/K+ ATPase on the basolateral
membrane, and also sodium and potassium channels on the luminal
membrane (increased production of these proteins ultimately results in
more sodium retention)
Brown, Eriksen, Jones, Heffernan, Kanjanavaikoon, Leo, Mishkin, Stern, Verlander &Wasserman
Ed. Mishkin 2006
Page 65 of 168
iii. Stimulates the release of vasopressin (ADH) from the posterior pituitary which
acts upon the kidneys to increase fluid retention
iv. Stimulates thirst centers within the brain
v. Facilitates norepinephrine release and inhibits norepinephrine reuptake
vi. Stimulates cardiac and vascular hypertrophy
c. Angiotensin II Receptor Subtypes and Renal Actions
i. Subtype 1A and 1B
1. Increase arterial pressure
2. Increase aldosterone release Æ increase Na+ reabsorption
3. Afferent and efferent vasoconstriction
4. Mesangial cell (support cells in the glomerulus) contraction Æ
decreases Kf (i.e., makes cells less permeable, allowing less liquid to get
into the glomerulus, decreasing water loss)
5. Increased sensitivity of TGF mechanism
a. Increased sensitivity of afferent arteriole to signals from
macula densa cells
6. Increase Na+ reabsorption
a. In the proximal tubule: Stimulate Na+/H+ exchanger activity
(apical, amiloride sensitive) and Na+/HCO3- co-transporter
(basolateral)
b. In the distal tubule: Stimulate Na+/H+ exchanger activity
7. Decrease Renin secretion (negative feedback)
8. Increase Endothelin, TXA2, Reactive Oxygen Species
ii. AT2 type receptor (fetal/embryonic)
1. Vasodilator effect
2. Inhibit cell proliferation
3. Stimulate Bradykinin
4. Stimulate nitric oxide synthase
5. Major tubular transport mechanisms and commonly used diuretics that block transport
mechanisms
a. Thick Ascending Limb of the Loop of Henle
i. Furosemide (a diuretic indicated for edema) blocks the luminal Na+-K+-2Cl-
cotransporter
1. Increases NaCl excretion
2. Increases K+ excretion
3. Increases Ca2+ excretion
4. Decreases ability to concentrate urine, (because of decreased
corticopapillary gradient)
5. Decreases ability to dilute urine (because of inhibition of diluting
segment)
b. Distal Tubule
i. Thiazide blocks the luminal Na+/Cl- co-transporter in the early distal tubule.
Amiloride blocks the luminal Na+/H+ exchanger in the early distal tubule.
1. Increase NaCl excretion
2. Decrease K+ excretion
3. Decrease Ca2+ excretion
4. Decrease ability to dilute urine (because of inhibition of cortical diluting
segment – aka early distal tubule)
5. No effect on ability to concentrate urine
c. Collecting Duct Cells
i. Amiloride blocks the luminal Na+ channel (in the principal cells) and Na+/H+
exchanger (in intercalated cells)
Brown, Eriksen, Jones, Heffernan, Kanjanavaikoon, Leo, Mishkin, Stern, Verlander &Wasserman
Ed. Mishkin 2006
Page 66 of 168
1. Increase Na+ excretion
2. Decrease K+ excretion (used in combination with loop or thiazide
diuretics)
3. Decrease H+ excretion
6. Pressure Natriuresis
Increased blood volume increases arterial pressure, renal perfusion, and glomerular filtration rate.
This leads to an increase in renal excretion of H2O and Na+ that is termed pressure natriuresis.
Pressure natriuresis is modulated and regulated by other systems…
a. Low Blood volume Æ Renin-Angiotensin-Aldosterone system Æ increased sodium
retention Æ increased water retention
i. Both angiotensin and aldosterone, although by different mechanisms, stimulate
distal tubular sodium reabsorption and decrease renal sodium and water loss
b. Low Blood Volume Æ vasopressin (ADH) release Æ water reabsorption in the
collecting duct Æ decreasing water loss & increasing blood volume
7. Hypertension from renal impairments
Hypertension results from an underlying defect in the ability of the kidneys to adequately handle
sodium. Increased sodium retention could then account for the increase in blood volume.
a. Chronic increase in “effective” blood volume (i.e. increased cardiac output)
i. Hypervolemia
1. Renal artery stenosis
2. Hyperaldosteronism
3. Hypersecretion of ADH
4. Aortic Coarctation
5. Pregnancy (preeclampsia)
b. Inappropriate activation of one of the many vasoconstrictor systems (i.e. increased
systemic vascular resistance)
i. Stress (sympathetic activation)
ii. Atherosclerosis
iii. Renal artery disease (increase ANG II)
iv. Pheochromocytoma (increased catecholamines)
v. Thyroid Dysfunction
vi. Diabetes
vii. Cerebral Ischemia (Cushing’s phenomenon)
c. ANG II-Dependent Hypertension can cause:
i. Increased arterial pressure and pulse pressure
ii. Increased aldosterone secretion
iii. Activation of Endothelins, Thromboxane, and reactive oxygen species
iv. Activation of cytokines and growth factors
d. When hypertension is sustained, possible results:
i. Vascular injury
ii. Stroke
iii. Coronary Arterial disease
iv. Kidney damage and fibrosis in many other tissues

Mechanisms of Acid Base Balance I & II: HCO3 Reabsorption & H+ Secretion; Tritatable
Acids & NH4 Excretion

Whole body acid-base balance and the role of the kidney


1. The normal rates of daily acid production and the sites of elimination.
a. We produce acid at a rate of 1mEq/kg/day, so for an average 70 kg person, 70
mEq/day is produced on a typical diet.
b. The acid must all excreted by the kidneys.
Brown, Eriksen, Jones, Heffernan, Kanjanavaikoon, Leo, Mishkin, Stern, Verlander &Wasserman
Ed. Mishkin 2006
Page 67 of 168
2. The importance of bicarb/CO2 buffer system and the use of the Henderson-
Hasselbach equation in interpreting acid-base homeostasis.
a. The bicarb/CO2 buffer system is the highest capacity buffering system in the body.
b. CO2 + H2O ↔ H2CO3 ↔ H+ + HCO3
c. This equation can be written as the Henderson-Hasselbach equation:
i. pH=6.1 + log (HCO3/.03 x p CO2)
ii. We can calculate any of the three variables provided by this equation (pH,
bicarb, or CO2) from any two other variables. (i.e. if we have two, we can
get the third… isn’t that the point of algebra?)
3. Four “Simple” types of acid-base disorders (metabolic/respiratory
acidosis/alkalosis) and the direction of changes in HCO3, CO2, and pH in each
disorder with measurement of arterial “blood gases.” Understand the concept of the
“anion gap” in diagnosis of metabolic acidosis.
a. Acid Loads
i. Respiratory acidosis- an increase in CO2 levels (possibly from respiratory
depression, emphysema, anything that makes you not breathe out as well)
1. Alveolar ventilation should increase to maintain constant CO2
levels. Alveolar ventilation is controlled by chemoreceptors in the
medulla which are sensitive to pH.
a. Decrease in cerebral pHÆ increases ventilation Æ lowers
pCO2 levels
2. The chemoreceptors respond more rapidly to increased CO2 levels
than increased non-volatile acid levels
3. Simply by mass action, an increase in CO2 will lead to an
increase in H+ and HCO3
a. In chronic respiratory acidosis, renal compensation occurs:
HCO3 reabsorption is increased, which helps bring pH
higher, towards normal.
ii. Metabolic acidosis- caused by an increase in non-volatile acids. These can
include phosphoric and sulfuric acid which are dietary byproducts. Non-
volatile acid is what makes up the 70 mEq/day that our kidneys excrete.
When the non-volatile acid load exceeds what the kidney can excrete,
metabolic acidosis ensues.
1. In an arterial blood gas test HCO3 plasma levels are lowered. The
loss of the alkali component is equivalent to the addition of acid, so
CO2 and pH levels decrease. (See Henderson-Hasselbach for
clarification)
b. Alkali Loads
i. Respiratory alkalosis- excess CO2 removal (resulting from
hyperventilation)
1. CO2 loss Æ H+ and HCO3- loss Æ pH increases
a. Renal compensation:
i. decreased excretion of H+
ii. decreased reabsorption of new bicarb
ii. Metabolic alkalosis- caused by an increase in non-volatile alkali
(exogenous from the diet or a decrease in acid excretion)
1. Not usually caused by an increase in alkali from a normal diet
2. Reflected in arterial blood gas by increased HCO3 plasma levels,
constant CO2 levels, and increased pH.
c. Anion gap- depleted plasma anions (Cl or HCO3)
i. Normally when you add up all the ions in the blood (Na, K, Cl, and bicarb)
the charge will be about +12. If the Cl or bicarb is depleted, the charge
Brown, Eriksen, Jones, Heffernan, Kanjanavaikoon, Leo, Mishkin, Stern, Verlander &Wasserman
Ed. Mishkin 2006
Page 68 of 168
difference between anions and cations will be even greater (so the charge
will be greater than 12). This indicates the presence of metabolic acidosis.
ii. This is the measure most frequently used to determine if acidosis is the
result of increased H+ ions or decreased bicarb.
4. Compensation for acid-base disorders
a. Two cases and a brief explanation from the old notes
b. A primary change in one system (respiratory (lungs) or metabolic (kidneys)) results
in a secondary change in the other system to compensate for the pH change.
c. Case #1- respiratory alkalosis
i. Moving to an area with increased altitude causes a great demand for O2, so
you breathe faster.
ii. When you breathe faster (hyperventilate) there is a drop in pCO2 leading to
respiratory alkalosis
iii. Over the next few days your kidneys will excrete more HCO3 into the urine
to lower the pH back to a normal value.
iv. Thus, the primary change in the respiratory system causes a secondary
metabolic change.
d. Case #2- metabolic acidosis
i. A patient with uncontrolled type I diabetes develops ketoacidosis (a form of
metabolic acidosis)
ii. As his pH falls, the respiratory rate will increase to lower pCO2 levels,
which will raise the pH to normal in a matter of minutes.
iii. Thus, a primary change in metabolism causes a secondary change in the
respiratory system to compensate.
5. Two major tasks of the kidney in acid-base balance: bicarb reabsorption and
generation of “new” bicarb (by urinary acid excretion)
a. The kidneys play a major role in regulating systemic bicarb levels by regulating two
processes. Urine is generally free of bicarb, so almost all that is filtered (4.5 moles
per day) will need to be reabsorbed along the course of the nephron (mostly in the
proximal tubules).
b. In addition, the kidney is responsible for generating additional bicarb. The kidneys
produce bicarb by the excretion of acid in the urine (secretion of acid is equal to
production of alkali). Bicarb is produced at the same rate that acid is secreted: 1
mEq/kg body weight/day, replacing the amount of bicarb occupied in acid
buffering.
6. Components of net acid excretion (NAE).
a. Acid cannot be secreted in the urine as protons (urine pH is about 4.5, but that does
not account for all the acid we must secrete).
b. There are two main processes by which acid is excreted by the kidneys: excretion of
titratable acid and the excretion of ammonia. About half the acid is excreted by
each method.
i. Titratable acid refers to excretion of a proton coupled to a urinary buffer.
The chief urinary buffer is phosphate.
ii. Ammonia (NH4+) helps in the formation of bicarb. Thus if it is excreted it
represents the formation of new bicarb (or acid secretion). More details in
next section.
c. NAE= titratable acid + ammonium – urinary bicarbonate
i. The loss of other organic ions is not quantitatively important in calculating
whole body acid-base balance

Specific Renal Mechanisms of Acid-base transport


1. General characteristics of proximal and distal tubules in acid-base transport
Brown, Eriksen, Jones, Heffernan, Kanjanavaikoon, Leo, Mishkin, Stern, Verlander &Wasserman
Ed. Mishkin 2006
Page 69 of 168
a. Proximal tubule reabsorbs about 75-80% of total filtered bicarb.
i. Most bicarb reabsorption is driven by the Na/H exchanger located on the
apical surface of the tubule cells: H+ is secreted Æ combines with bicarb
Æ forms carbonic acid Æ broken down into CO2 and water.
1. Some H+ is also secreted via an H-ATPase
ii. CO2 Æ diffuses across cell membrane Æ combines with OH- (via the help
of carbonic anhydrase) Æ Bicarb Æ pumped back into the blood (via a Na-
3 HCO3 exchanger on the basolateral surface).
b. Distal tubule – intercalated cells are responsible for acid/base exchange
i. The intercalated cells secrete H+ via an H+/ATPase pump located on the
apical membrane. Also, the distal tubule wall is fairly impermeable to ions.
ii. A transepithelial pH gradient is generated (this is not possible in the
proximal tubule since it is permeable to ions) Æ urine pH < 5 because of
the combined acid secretion and ion impermeability.
iii. The proton secretion will result in bicarb reabsorption if bicarb is present in
the distal tubule (probably only if there is some pathology of bicarb
absorption in the proximal tubule).
2. Apical and basolateral membrane transport characteristics of the proximal tubule.
Know the role of carbonic anhydrase.
a. See above, question 1.a. i. and ii.
3. Factors that regulate proximal tubule bicarb reabsorption.
a. Factors that increase bicarb reabsorption:
i. Decreased cellular pH
ii. Increased systemic pCO2
iii. Decreased peritubular bicarb levels
iv. Chronic potassium depletion
v. Chronic acidosis (increases apical Na/H exchangers and basolateral
Na/bicarb transporters)
vi. Decreased ECF volume (in general in low volume states you reabsorb more
solute so that you can reabsorb more water from the tubules)
vii. Angiotensin II
viii. Hypercalcemia
b. Factors that inhibit bicarb reabsorption
i. Parathyroid hormone
ii. Increased peritubular bicarb
iii. Increased cellular pH
4. General features of the collecting duct that mediate acid-base transport.
a. Collecting duct, like the distal tubule, is capable of generating a large transepithelial
pH gradient (urine <5, blood ~7.4)
b. H+ secretion (accompanied by Na reabsorption) is stimulated by mineralocorticoids
c. Otherwise, the collecting duct functions much like the distal tubule.
5. General features of ammonium production and excretion by the kidneys.
a. Ammonia is excreted in the proximal tubule.
b. The predominant precursor of ammonia is glutamine (each glutamine forms one
bicarb and one ammonia molecule). Most of the ammonia ends up in the urine.
c. Glutamine Æ NH4+ + [alpha-ketogluterateÆCO2Æ HCO3 ]
d. If NH4+ is reabsorbed, the HCO3 from glutamine will combine with it to form urea
which will be excreted, so generation of HCO3 causes no change in acid-base
balance
e. If NH4+ is not reabsorbed, it will be excreted in the urine and represent the true
formation of a bicarb ion.
Brown, Eriksen, Jones, Heffernan, Kanjanavaikoon, Leo, Mishkin, Stern, Verlander &Wasserman
Ed. Mishkin 2006
Page 70 of 168
i. Thus NH4+ in the urine represents acid excretion (see 6.c.i. for NAE
equation)

Integrative function of the kidneys

1. How sodium balance determines ECF and response to ECF depletion


a. As we learned in block 1, changes in ionic concentration lead to changes in fluid
retention and compartmentalization in order to maintain ideal osmolarity. Sodium
balance is what ultimately determines ECFV and therefore controls all pathological
states of volume change.
b. Sodium depletion
i. caused by: vomiting, diarrhea, severe burns, intestinal obstruction, diuretics
c. Sodium excess
i. caused by:
1. CHF Æ extracellular volume depletion Æ Na reabsorptive
mechanisms Æ increased ECV Æ edema, JVD, etc.
2. nephrotic syndrome, cirrhosis, renal failure
2. General distribution of potassium in ECF and ICF and the role of the kidneys in
potassium regulation
a. Potassium Standards:
i. ECF 3.5-4.5 mEq/liter
ii. ICF 140-150 mEq/liter
iii. Total body stores: 3500 mEq
iv. Daily intake 100mEq
b. Renal potassium
i. Excretion of potassium is predominantly via the kidneys
1. determined by secretion in distal convoluted tubule
ii. reabsorbed mostly in the proximal tubule
3. Mechanisms regulating potassium uptake from ECF into cells
a. maintained by Na/K pump
b. Factors driving potassium into the cells:
i. Insulin (by stimulating basolateral Na/K ATPase pump)
ii. beta-adrenergic agonists
iii. mineralocorticoids
iv. glucocorticoids
v. alkalemia
vi. high HCO3
vii. hyposmolarity (K flows into cell along with water)
viii. Alkalosis – exchange of intracellular H+ for extracellular K+
4. Mechanisms regulating potassium excretion in the urine
a. Factors increasing renal excretion of potassium:
i. aldosterone
ii. increased potassium intake
1. also causes aldosterone release
iii. alkalemia
iv. increased Sodium delivery to distal tubule
v. increased urine flow rate
vi. insulin deficiency
vii. beta-adrenergic antagonists
viii. acidosis – exchange of extracellular H+ for intracellular K+
ix. exercise
x. cell lysis
Brown, Eriksen, Jones, Heffernan, Kanjanavaikoon, Leo, Mishkin, Stern, Verlander &Wasserman
Ed. Mishkin 2006
Page 71 of 168
xi. inhibition of Na+/K+ pump
b. Luminal voltage is generally negative, which helps to drive potassium secretion
i. aldosterone increases the negative voltage
5. Not an LO, but since most of the packet section is about it…
a. Hyperkalemia
i. Signs and Symptoms:
1. ventricular arrythmias
2. cardiac arrest
3. EKG: peaked T waves, increased PR intervals, widened QRS
complexes, sine wave pattern
4. hypotension – because of peripheral vasodilation and decreased
CO
5. weakness and paralysis
ii. compensatory response
1. release of insulin, epinephrine and aldosterone
iii. etiologies:
1. pseudohyperkalemia:
a. faulty blood drawing technique – prolonged tourniquet
application or excess activity of arm below tourniquet
b. hemolysis of drawn blood
c. lysis of white cells or platelets
2. exogenous potassium load
a. from diet, transfusion, or potassium penicillin
b. has to be quite a bit for the body to not be able to cope
3. endogenous potassium load or shift
a. tissue damage Æ potassium released from intracellular
stores
i. tissue damage can be from crush injuries,
rhabdomyolysis (muscle necrosis), ischemia, or
intrvascular hemolysis
b. chemotherapy Æ massive cell lysis
c. acidemia
d. drugs – massive digitalis overdose
e. acute hyperosmolality
f. hyperkalemic period paralysis (rare)
4. decreased renal potassium excretion from:
a. potassium sparing diuretics
b. mineralocorticoid deficiency
i. Addison’s Disease
ii. Hyporeninemic hypoaldosteronism (type IV
tubular acidosis)
iii. Tubular defects of potassium excretion (seen in
sickle cell disease)
b. Hypokalemia
i. Signs and Symptoms:
1. muscle weakness, paralysis, necrosis
2. “ileus,” - decreased gut motility
a. because of decreased neuromuscular activity
3. arrythmias
a. especially in digitalized patients
4. glucose intolerance
a. modified insulin secretion
Brown, Eriksen, Jones, Heffernan, Kanjanavaikoon, Leo, Mishkin, Stern, Verlander &Wasserman
Ed. Mishkin 2006
Page 72 of 168
5. impaired renal concentration
a. acquired nephrogenic diabetes insipidus
6. chronic hypokalemia Æ tubular structural abnormalities
ii. Etiology
1. cellular shifts without potassium deficiency
a. alkalosis or hypokalemic periodic paralysis
i. unknown origin
2. inadequate intake
a. starvation, undernourishment, alcoholism
b. kidneys can compensate quite a bit
3. excessive potassium loss
a. vomiting, diarrhea
b. standard use of diuretics and osmotic diuretics (glucose)
i. this is the most common cause
c. primary mineralocorticoid excess (Cushing’s syndrome)
d. primary aldosteronism
e. Bartter’s syndrome
i. hypokalemia in association with
hyperaldosteronism but without hypertension
f. excessive licorice ingestion (umn, what?)
g. adrenogenital syndrome
h. Liddle’s syndrome
i. stimulation of aldosterone-independent K+
secretion
i. secondary mineralocorticoid excess
j. hyperrenimia
i. examples: edematous disorders, malignant
hypertension, renovascular hypertension
k. renal tubular acidosis
l. antibiotics
i. amphotericin B, gentamicin, carbenicillin
m. acute leukemias
n. Magnesium deficiency
6. Since the LOs had nothing to do with the lecture, here’s the lecture, just for the hell
of it:
a. Case 1: Vomiting patient
i. Vomiting Æ loss of fluid Æ renin-angiotensin system is activated to bring
BP back up Æ renin-angiotensin system activates aldosterone Æ promotes
Na+ reabsorption, K+ excretion, H+ excretion
1. K+ excretion change is indirect; it is due to the voltage change
resulting from Na+ reabsorption: Na+ comes in Æ inside cell is
more +, lumen of tubule is more negative Æ lumen attracts
cations, therefore K+ and H+ are both pulled out and excreted
ii. H+ excretion results in an increase in bicarb levels (mass action)
b. Case 2: Addison’s disease
i. adrenal failure Æ no aldosterone Æ compromised sodium reabsorption
1. Reverse of vomiting situation: Na+ is leaving too much Æ luminal
charge should be too high Æ less K+ and H+ go into lumen Æ
K+ and H+ levels in body increase
2. increased H+ decreases bicarb (mass action)
c. Case 3: Lung cancer
Brown, Eriksen, Jones, Heffernan, Kanjanavaikoon, Leo, Mishkin, Stern, Verlander &Wasserman
Ed. Mishkin 2006
Page 73 of 168
i. The tumor is inappropriately secreting ADH (SIADH = Syndrome of
inappropriate ADH recretion)
1. Increased ADH Æ increased reabsorption of water Æ osmolality
decreases in spite of normal absolute ionic levels
d. Case 4: Diabetes
i. Not shockingly, very high glucose levels are found in the diabetic patient
not taking insulin.
ii. other electrolyte/ion levels are all of due to diabetic nephropathy, which is
basically the loss of renal function in chronic uncontrolled diabetes.

Block III: Pulm & GI


Introduction to respiration and respiratory mechanics I
March 20th
Dr.Levitzky
Overview of the respiratory system, generation of a pressure gradient, elastic properties of
the lung and chest wall
1. Exchange of oxygen and carbon dioxide between the body and the atmosphere –
related to the metabolism of body tissues
a. Air is breathed in (21% oxygen) through conductive airways Æ into lungs Æ into
venous blood that has returned from body tissues (mixed venous blood has high
carbon dioxide and low oxygen content –when blood leaves the lungs –arterial
blood-- the oxygen content is high and the carbon dioxide is low) Æ into tissue Æ
release oxygen into tissue & pick up carbon dioxide in blood Æ back to lungs Æ
breathed out
2. Generation of the pressure gradient between the atmosphere and the alveoli
a. Air moves from high pressure to low pressure – just like fluid
b. Inspiratory muscles contract Æ Thorax volume increases Æ increase transmural
pressure gradient Æ increase alveoli volumeÆ pv is static so increased V decreases
pressure
c. Transmural pressure is inside-outside, or, here, alveolar-atmospheric
d. Δp=VR
i. Translation: change in pressure = volume X resistance
3. Structure of the respiratory system – anatomical and respiratory gases
a. Air goes through nose or mouth Æ pharynx Æ larynx Æ tracheobronchial tree:
Trachea Æ Bronchi Æ Bronchioles Æ Terminal bronchioles Æ Respiratory
bronchioles Æ Alveolar ducts Æ Alveolar sacs
i. Trachea to terminal bronchioles are “conducting zone”
ii. Respiratory bronchioles and distal are “respiratory zone”
b. The conducting zone is anatomical dead space – i.e., no air exchange
c. The respiratory zone has gas exchange across the alveoli
4. Elastic properties of lung and chest wall
a. Lung distends easily at decreased transpulmonary pressures and not as easily at
increased pressures
b. Hysteresis: The fact that the lung distends differently during inspiration and
expiration
c. Compliance = Δv/Δp
i. Compliance is the inverse of elastic recoil
ii. Compliance is added directly in parallel
iii. Compliance is added inversely in series
Brown, Eriksen, Jones, Heffernan, Kanjanavaikoon, Leo, Mishkin, Stern, Verlander &Wasserman
Ed. Mishkin 2006
Page 74 of 168
d. Lungs are in series with chest wall
e. Lungs are in parallel with each other
f. Elastic recoil of lung: parenchyma has elastic properties, surface tension also helps
(surface tension occurs at any gas-liquid-interface)
5. Passive expansion and recoil of alveoli
a. Alveoli only expand passively in response to contraction of inspiratory muscles,
which increases the transmural pressure gradient Æ opens alveoli Æ lowers alveolar
pressure
b. Alveolar recoil initiates expiration Æ lung decreases its volume
6. Mechanical interaction of lung and chest wall – related to negative intrapleural
pressure
a. Lung decreases its volume due to alveolar elastic recoil
b. Chest wall increases its volume from its own outward elastic recoil
c. Chest wall holds alveoli open in opposition to their elastic recoil
d. Æ Intrapleural pressure becomes more negative because pv is constant, so when the
chest wall pulls out and the alveoli pull in, the intrapleural space increases, so
volume has increased and therefore pressure must decrease
7. Pressure-volume characteristics of lung and chest wall – predict changes in lung and
chest wall compliances and explain how the combination of the two changes in
different physiological and pathological conditions
a. This has really been addressed already.
b. Volume is correlated indirectly with pressure (see 8)
c. Obstructive diseases increase compliance Æ volume/pressure curve – slope
increased
d. Restrictive diseases decrease compliance
8. Draw a normal pulmonary pressure volume curve – label inflation and deflation
limbs – cause and significance of hysteresis in the curves
a. Surfactant causes hysteresis

9. Lung compliance calculation; 2 clinical conditions that change compliance


a. Compliance = Δv/Δp
b. 1/Total compliance = 1/compliance of the lung + 1/compliance of the chest wall
c. Obstructive and restrictive diseases have already been covered.
Brown, Eriksen, Jones, Heffernan, Kanjanavaikoon, Leo, Mishkin, Stern, Verlander &Wasserman
Ed. Mishkin 2006
Page 75 of 168
10. Roles of surface tension, pulmonary surfactant, alveolar interdependence in recoil
and expansion of lung
a. If surface tension were constant, small alveoli would have greater pressure because
radius is lower so in order for tension to be maintained, pressure would have to be
higher as per LaPlace’s Law:
i. Tension = pressure X radius /2
b. If there were greater tension in the alveoli, you would expect that the alveoli would
collapse
c. Luckily for us, surfactant and water have different properties: Surfactant does not
have a constant surface tension
i. Surfactant has low surface tension in small areas and high surface tension in
bigger areas Æ equalizes pressure between alveoli of varying sizes
ii. Surfactant decreases surface tension Æ decreases inspiratory work Æ
decreases elastic recoil & increases compliance
iii. Premature babies sometimes cannot produce surfactant Æ increased
respiratory work Æ collapsed alveoli Æ “Infant respiratory distress
syndrome”
iv. In adults – decreased surfactant production and increased surfactant
production – “Adult respiratory distress syndrome” aka “Shock-lung
syndrome”
d. In addition, we have structural interdependence which helps maintain alveoli in the
beautiful honeycomb shape
i. Alveoli help keep each other open via mechanical force
ii. Pneumothorax Æ interrupts interdependence Æ lose interdependence Æ
lung collapses
11. FRC – define, and predict changes in FRC in different physiological and
pathological conditions
a. FRC is functional residual capacity:
i. =Residual Volume + Expiratory reserve
ii. the balance between outward chest and inward lung elastic recoils
determines FRC
b. Supine Æ reduces FRC
i. Because lung compliance remains the same but the chest wall compliance
decreases substantially in the absence of gravitational force
ii. While standing, gravity pulls abdominal contents farther away Æ allows
diaphragm to move farther down. While lying down, abdominal contents,
no longer being pulled away from the diaphragm, smush into the
diaphragm, preventing the diaphragm from fulfilling its true manifest
destiny during inspiration. It makes sense then that the chest wall
compliance would go down because theoretically, if your chest wall
compliance did not reduce to go along with the diaphragm’s reduced ability
to move away from the thorax, then you could just inhale infinitely and rip
your chest wall straight off of your diaphragm. Better to just be reined in
by the diaphragm’s limitations, n’est-ce pas?
c. Being obese, pregnant, out of the earth’s field of gravity, or having a restrictive
disease Æ reduces FRC
i. Think of it as losing your gravity, and see 11-b-ii
ii. It restricts your ability to breathe in
d. Emphysema, COPD, chronic bronchitis, asthma (all obstructive diseases) Æ
increases FRC
i. You’re having trouble breathing out, so more air gets stuck inside
Brown, Eriksen, Jones, Heffernan, Kanjanavaikoon, Leo, Mishkin, Stern, Verlander &Wasserman
Ed. Mishkin 2006
Page 76 of 168

Respiratory Mechanics II:


Respiratory muscles, resistance to air flow, work of breathing
March 20th
Dr.Levitzky

1. Diagram lung volume, tracheal pressure, alveolar pressure, pleural pressure – during

normal quiet breathing cycle. Identify onset and cessation of inspiration, and
cessation of expiration. Relate the pleural and airway pressure values to the
movement of air.

2. Definition of airway resistance; factors contributing to or altering air-flow resistance.


a. Resistance = Difference in Pressures / Flow
b. 80% of Resistance encountered is airway resistance,
c. 20% is pulmonary tissue resistance
i. caused by friction encountered as pulmonary tissues move against each
other during expansion
d. Most air flow in the lungs is turbulent or transitional because of extensive branching
e. Smaller airways Æ increased total cross-sectional area Æ decreased resistance
i. Cross sectional area is inversely proportional to resistance
ii. Poiseille’s Law: (Shout out to Dr.Johnson)
1. R=8X η X length/πr4
Brown, Eriksen, Jones, Heffernan, Kanjanavaikoon, Leo, Mishkin, Stern, Verlander &Wasserman
Ed. Mishkin 2006
Page 77 of 168
3. Dynamic compression of airways during forced expiration, identifying relevant
pressure that drives expiratory flow; describe conditions necessary for dynamic
compression to exist. How and where dynamic compression might occur during
inspiration.
a. During forced expiration Æ increase intrapleural pressure
b. Dynamic compression: compression of the airway during expiration
c. Equal pressure point hypothesis: There is a point along the airway where the
transmural pressure = 0 (because the pressure inside = pressure outside) and that
when the pressure outside is higher than the equal pressure point Æ collapse of
airway in the absence of cartilage
i. During passive expiration, EPP is in cartilage – no collapse
ii. During forced expiration, contraction of expiratory muscles Æ increased
alveolar pressure and significantly increased intrapleural pressure
(superatsmopheric) causes EPP to be driven farther down into bronchial
tree Æ collapse
d. This should not happen during inspiration – we don’t know what disease that would
be, but it would suck.
4. Areas of pressure-dependent and –independent portions of flow-volume diagram and
explain the shift in the shape of flow-volume curves, which occur with asthma.
a. Flow-volume curve:
i. At high lung volumes, the air-flow rate is effort-dependent
ii. Effort-independent flow: During forced expiration, at low lung volumes
(less than 80% of TLC), and intrapleural pressure is positive, heterogenous
expiration efforts ultimately merge into the same flow curve – it is therefore
effort independent
iii. It makes sense that a change in effort will change expiration more when
there is greater lung volume to be expelled. At low volumes, you are
expiring in vain.
iv. At high enough intrapleural pressure, dynamic compression occurs, which
traps gas Æ harder expirations can’t expell gas that’s trapped (Fig 2-22).
b. In restrictive diseases, reduced lung volumes Æ Should decrease peak expiratory
flow, because TLC is reduced
i. Effort-independent portion of the curve is normal
1. FEV1/FVC ratio is normal or high because both of these values
are decreased
c. In obstructive diseases such as asthma Æ TLC is increased Æ Residual volume may
be greatly increased
i. Effort-independent portion of the curve is depressed inward; flow rates are
low for any relative volume
ii. Increased resistance Æ small peripheral airway narrowing and reduced gas
flow
5. Relate changes in the dynamic compliance of the lung to alterations in airway
resistance
a. Dynamic compliance is the change of volume in the lungs divided by the change in
alveolar distending pressure over the course of a breath
b. Dynamic compliance = static compliance at 15 or fewer breaths/minute or even
while a healthy person is breathing faster
c. Dynamic compliance/Static compliance ratio decreases in obstructive disease (i.e.,
small airways disease) Æ changes in dynamic compliance therefore reflect changes
in airway resistance and changes in the compliance of the alveoli
Brown, Eriksen, Jones, Heffernan, Kanjanavaikoon, Leo, Mishkin, Stern, Verlander &Wasserman
Ed. Mishkin 2006
Page 78 of 168
d. An increase in airway resistance means that the portion of the lung that airway
serves will fill more slowly than it otherwise would
i. If the person is breathing quickly, the inability to fill rapidly enough will
reduce the number of alveoli that can be filled during inspiration.
ii. Resistance therefore decreases dynamic compliance and decreases oxygen
intake during rapid breathing
6. Factors contributing to the work of breathing
a. Work done in breathing is proportional to ΔpΔv
b. Δv= Tidal volume
c. Δp= Change in transpulmonary pressure necessary to overcome the elastic and
resistive work of breathing
d. The main components of the work of breathing:
i. Elastic recoil of lungs
ii. Elastic recoil of chest wall
iii. Resistance to air flow
7. Predict alterations in the work of breathing in different physiological and
pathological states
a. Obesity
i. Increased chest wall elastic recoil Æ Increased work of breathing
b. Restrictive diseases, pulmonary fibrosis or Lack of surfactant
i. Increased alveolar recoil Æ increased work
c. Obstructive diseases, asthma, emphysema, bronchitis
i. Increased airway resistance
ii. Dynamic compression of the alveoli (unopposed) Æ Decreases alveolar
elastic recoil Æ decreased expiratory pressure gradient

Alveolar Ventilation:
Dead Space, Regional Distribution of Ventilation
3/21/06, Levitzky
1. Define partial pressure and fractional concentration as they apply to gases in air.
a. Dalton’s Law: partial pressure of a particular gas is equal to its fractional
concentration times the total pressure of all the gases in the mixture.
i. Pgas= % total gas X Ptot
2. List normal atmospheric, inspired, alveolar, and expired values for O2, CO2, and N2
(all listed in mmHg, at standard barometric pressure). Explain why the
concentrations change as the gases pass through the respiratory system.
a. Atmospheric: O2- 158; CO2- 0.3; N2- 600.6; =760 torr
b. Inspired: O2- 149; CO2- 0.3; N2- 564.0; H2O – 47.0 torr =760 torr
c. Alveolar: O2- 104; CO2- 40; N2- 569; H2O – 47.0 torr =760 torr
d. Expired: O2- 120; CO2- 27; N2- 566; H2O – 47.0 torr =760 torr
e. Air is humidified during inspiration Ægas concentrations are dilutedÆpartial
pressures are lower in inspired than atmospheric air
i. Apply the formula to atmospheric gases: PIGas= FIGas (PB- PH20)
ii. Translation: partial pressure of gas in inspired air is equal to the fractional
concentration of inspired gas times the difference between the barometric
pressure and water vapor pressure
f. Alveolar pressures determined by
i. Alveolar ventilation
ii. Pulmonary capillary perfusion
iii. Oxygen consumption
Brown, Eriksen, Jones, Heffernan, Kanjanavaikoon, Leo, Mishkin, Stern, Verlander &Wasserman
Ed. Mishkin 2006
Page 79 of 168
1. 300 ml/min O2 constantly diffused from alveoli Æ pulmonary
capillaries
iv. Carbon Dioxide production
1. 250 ml/min CO2 constantly diffused from pulmonary capillariesÆ
alveoli
g. Expired air = 350 ml alveolar + 150 ml dead space air
i. Æ O2 pressure higher in mixed expired air than alveolar but lower than in
inspired PO2 ~ 120 torr
ii. Æ CO2 pressure lower in mixed expired air than alveolar but higher than
inspired PCO2 ~ 27 torr
3. Draw a normal spirogram labeling the four lung volumes and four capacities. List
the volumes which comprise each of the four capacities. Identify which volume and
capacities cannot be measured by spirometry
a. Four lung volumes
i. Tidal Volume(VT)- 500 ml; a normal breath
ii. Residual Volume(RV)- 1.5 L; what is always left in your lungs so they don’t
collapse
iii. Inspiratory Reserve Volume (IRV)- 2.5 L; the extra air you can breathe in
with forced inspiration, beginning after tidal volume
iv. Expiratory Reserve Volume (ERV)- 1.5 L; the extra air you can breathe out
with forced expiration, beginning after tidal volume
b. Four lung capacities
i. Functional Residual Capacity(FRC)= RV+ERV; volume of gas remaining
in the lung after normal expiration
ii. Inspiratory Capacity (IC)= VT + IRV; the total inhaled into lungs with
maximal inspiratory effort, beginning at the end of normal tidal expiration
iii. Total Lung Capacity (TLC)= VT + IRV + ERV + RV; the total volume of
air in the lungs after a maximal inspiration
iv. Vital Capacity (VC)= VT + IRV + ERV; total volume of air that can be
expelled from the lungs in forced expiration following a forced inspiration
c. Any volume that includes the residual volume cannot be measured using spirometry.
Therefore, in addition to RV, FRC and TLC cannot be measured directly by
spirometry. Vital capacity does not include RV and therefore can be measured by
spirometry.
4. Predict the effects of alterations in lung and chest wall mechanics, due to normal or
pathological processes, on the lung volumes.
a. Normal Physiological Processes
i. Gravity-
1. [StandingÆSupine] Æ ↓ FRC; abdominal contents no longer being
pulled away from diaphragm
2. ↓ FRC Å↓ ERV and ↑IRV
3. RV, VC, and TLC may decrease slightly b/c increased blood flow
to the thoracic cavity
b. Pathological Processes
i. Restrictive Disease
1. Reduced compliance of lungsÆ compressed lung volumes
2. ↓FRC, TLC, VC, IRV, ERV, and possibly even RV
3. ↓ Tidal volume, VT with corresponding ↑ respiratory rate
4. Example: alveolar fibrosis
ii. Obstructive Disease
1. ↑ resistance to airflow
2. Mucous obstructs airways
Brown, Eriksen, Jones, Heffernan, Kanjanavaikoon, Leo, Mishkin, Stern, Verlander &Wasserman
Ed. Mishkin 2006
Page 80 of 168
3. high intrapleural pressure to overcome airway resistance during
forced expiration
4. big ↑ RV, FRC, TLC
5. ↓VC and ERV
6. ↓ respiratory rate with corresponding ↑VT
7. Examples: emphysema, chronic bronchitis, asthma
5. Define minute ventilation and alveolar ventilation.
a. Minute ventilation- amount of air entering and leaving the nose or mouth per minute
b. Alveolar ventilation- amount of air entering and leaving the alveoli per minute; less
than the minute ventilation because some of each inspiration remains in the
conducting airways and never reaches the alveoli
6. Define the anatomic dead space and relate anatomic dead space and the tidal
volume to alveolar ventilation.
a. Anatomic dead space- conducting airways; airways that are too thick for gas
diffusion to take place; airways where blood does not come into contact with air
b. On average, 150 ml per tidal volume remains in the anatomic dead space
i. VT= VD + VA
ii. Tidal volume is equal to dead space volume plus volume entering and
leaving the alveoli per breath
c. When we rearrange the equation in 6.b.i we get the equation for alveolar ventilation:
i. A= E- D

ii. represents minute ventilation


iii. Translation: Alveolar ventilation is equal to the expired minute volume
(equivalent to tidal volume) minus the volume wasted ventilating the dead
space per minute
7. Understand the measurement of the anatomic dead space from the single breath test
diagram (See Figure 3-7)
a. Subject breathes in 500 ml test gas (ex. Helium) from a balloon
b. Some of that gas goes to alveoli (350 ml), some remains in conducting airways (150
ml)
c. The first portion of gas breathed back into the balloon on expiration is undiluted
test gas
d. The remainder of the expired gas is diluted test gas, so the total concentration of gas
in the balloon is lower than it was before inspiration
e. The last 150 ml that is expired from the alveoli remains in the anatomic dead space
after expiration
8. Determine alveolar ventilation. Describe how alveolar ventilation can remain
constant when minute ventilation changes.
a. Alveolar ventilation determined from VT, breathing frequency, and dead space
ventilation
b. Equation from 6.c.i. ( A= E - D) provides the relationship between minute
ventilation and alveolar ventilation
i. ↑ Æ↑Dead spaceÆAlveolar ventilation remains constant
ii. ↓ Æ↓Dead spaceÆ Alveolar ventilation remains constant
9. Define and determine physiologic and alveolar dead spaces.
a. Physiologic dead space= anatomic dead space + alveolar dead space
b. Alveolar dead space is volume of gas that enters unperfused alveoliÆ no gas
exchange occurs
i. Not usually present in healthy individuals
Brown, Eriksen, Jones, Heffernan, Kanjanavaikoon, Leo, Mishkin, Stern, Verlander &Wasserman
Ed. Mishkin 2006
Page 81 of 168
c. Any expired CO2 must have come from alveoli that are ventilated and perfused. In a
healthy individual, this CO2 level should be the same as that in perfused alveoli,
alveolar capillaries, and arterial blood. However, if the expired PCO2 (which
represents the mixed alveolar PCO2) is much less than the arterial PCO2, this indicates
the presence of significant alveolar dead space.
i. Translation: a lot of air is going to alveoli that are not getting perfused, so
the total volume of air you breathe out does not have the same CO2 level as
the blood perfusing the alveoli
ii. This difference is not surprisingly referred to as the arterial-alveolar CO2
difference. If it is large, it signifies a lot of alveolar dead space.
10. Describe in quantitative terms the effect of ventilation on Pco2 according to the
alveolar ventilation equation.
a. PAO2= FIO2 (PB-PH20) - PACO2/R
i. FIO2 (PB-PH20)= PIO2 (inspired partial pressure of oxygen)
ii. R= respiratory exchange ratio, CO2/ O2, AKA alveolar ventilation
iii. As alveolar ventilation increases, ↓PACO2 (alveolar CO2 pressure)Æ brings
PAO2 (alveolar partial pressure of oxygen) closer to PIO2 (inspired partial
pressure of oxygen)
11. Be able to estimate the alveolar oxygen partial pressure, PAO2, using the simplified
form of the alveolar gas equation.
a. See above, question 10, for formula… and know how to plug and chug
12. Predict the effects of alterations of alveolar ventilation on alveolar carbon dioxide and
oxygen levels.
a. See above, question 10
b. ↑R (alveolar ventilation)Æ ↓ PACO2 Æ brings PAO2 closer to PIO2
c. ↓RÆ↑PACO2 Æ brings PAO2 further from PIO2
13. Describe the regional differences in alveolar ventilation found in the normal lung and
explain these differences.
a. Regional differences in the lung influenced by gravity
i. “lower” regions (with respect to gravity) considered “dependent”Æbetter
perfusion and better ventilation – therefore more gas exchange.
ii. “non-dependent” regionsÆless ventilated and less perfused – therefore less
gas exchange
1. Perfusion is more reduced than ventilation is, therefore the V/Q
ratio is higher in the upper part of the lung
b. Dependent (lower) region
i. Less negative (higher) intrapleural pressure
ii. Lower transpulmonary pressure (alveolar – intrapleural) at FRC
iii. Alveoli have smaller volumeÆ this leads to difference in ventilation!
c. Move up lung to non-dependent regionÆ more negative intrapleural pressure,
higher transpulmonary pressure, larger volume alveoli
14. Predict the effects of changes in the lung volumes, aging, and disease processes on
the regional distribution of ventilation.
a. Aging ÆIncreased static lung compliance, decreased chest wall compliance
i. Caused by:
1. loss of alveolar elastic recoil
2. calcification of costal cartilages
3. decreased spaces between vertebrae
4. greater spinal curvature
ii. Æ ↑ FRC, constant TLC
Brown, Eriksen, Jones, Heffernan, Kanjanavaikoon, Leo, Mishkin, Stern, Verlander &Wasserman
Ed. Mishkin 2006
Page 82 of 168
b. ↓alveolar elastic recoilÆ ↓ opposition to dynamic compressionÆ airways close at
higher volumesÆ higher closing capacity
i. airways in dependent regions close at lung volumes above FRCÆ more
ventilation in non-dependent regions
c. ↓ strength of expiratory muscles
i. ↑ RV
ii. ↓ maximal expiratory rates (i.e. FEV1)
d. ↓ arterial oxygen tension
i. ventilation of poorly perfused areas
ii. loss of alveolar surface area, decreased diffusion capacity
e. More mismatch, lower PaO2
15. Define the closing volume and explain how it can be demonstrated.
a. Closing volume: lung volume at which airway closure begins to occur
b. Use Fowler method to measure closing volume
i. Beginning at RV take a deep inhale (to TLC) of 100% oxygen
ii. Subject then exhales to RV, measure nitrogen levels
1. Phase I- subject exhales purely from anatomic dead space; pure
oxygen, no nitrogen
2. Phase II- mixture of dead space and alveolar gas, increasing
nitrogen levels (nitrogen was in the alveoli from previous breath)
3. Phase III- mixed alveolar gas from upper and lower regions,
nitrogen level “plateaus”
4. Phase IV- airways begin to close; lower region, where there is less
nitrogen, closes first; nitrogen concentration rises rapidly as more
air comes from upper alveoli which have more nitrogen.
16. Predict the effects of changes in pulmonary mechanics on closing volume.
a. Airway resistance problemsÆ phase III of Fowler test not horizontal
i. alveoli supplied by high-resistance airways fill more slowly than those
supplied by normal airwaysÆ those with higher resistance will get less
100% oxygen during inspirationÆ higher resistance airways have higher
nitrogen concentration
ii. Higher resistance airways empty slower on expirationÆ rise in nitrogen
comes later in expiration (during phase III)

Pulmonary Blood Flow:


Bronchial Circulation, Pulmonary Vascular Resistance, Regional Distribution of Pulmonary
Circulation
3/21/06, Levitzky
1. Compare and contrast the bronchial circulation and the pulmonary circulation
a. Bronchial circulation, which comes from aorta and intercostal arteries, supplies the
tracheobronchial tree and other structures of the lung down to the level of the
terminal bronchioles
i. Distal to bronchioles, supplied by pulmonary circulation (respiratory
bronchioles, alveolar ducts, alveolar sacs, and alveoli)
ii. Bronchial blood pressure is equivalent to systemic blood pressure because
bronchial circulation is part of the systemic circulation
1. pulmonary blood pressure is lower than these two
2. Describe the anatomy of the pulmonary circulation and explain its physiological
consequences
a. Thin-walled vessels, little smooth muscleÆ less resistance to flowÆ lower
pressureÆ better gas exchange
Brown, Eriksen, Jones, Heffernan, Kanjanavaikoon, Leo, Mishkin, Stern, Verlander &Wasserman
Ed. Mishkin 2006
Page 83 of 168
3. Compare and contrast the pulmonary circulation and the systemic circulation
a. Pulmonary arteries:
i. Thinner walls, less smooth muscleÆ less resistance to flow
ii. lower intravascular pressuresÆ more distensible and compressible
iii. More susceptible to surrounding pressure changes
4. Describe and explain the effects of lung volume on pulmonary vascular resistance
a. ↑ lung volumeÆ
i. ↑ alveolar volume Æ pulmonary capillaries elongate and diameter
decreasesÆ ↑ resistance in alveolar vessels
ii. intrapleural pressure more negativeÆ ↑transmural pressure gradientÆ
↑distention of vesselÆ ↑ radial tractionÆ ↓ resistance in extraalveolar
vessels
iii. REMEMBER: resistance is inversely related to radius to the 4th power
b. ↓ lung volumeÆ
i. intrapleural pressure more positiveÆ extraalveolar vessels compressedÆ ↓
radial tractionÆ↑ resistance in extraalveolar vessels
1. Traction: elastic recoil of alveoli has a secondary effect of pulling
the adjacent airways open, preventing airway collapse
c. Alveolar and extraalveolar vessels can be visualized to be in series, so their
resistances are additiveÆ changes in both types of vessels gives total PVR (see Fig.
4-4)
i. At low and high lung volumesÆ ↑PVR
5. Describe and explain the effects of elevated intravascular pressures on pulmonary
vascular resistance (PVR)
a. ↑ intravascular pressureÆ ↓ PVR
i. ↑ COÆ pulmonary artery pressure is fairly constant due to ↓ PVR
ii. this decrease appears to be passive (no neural nor humoral influence)
iii. due to recruitment and distention
b. Recruitment
i. ↑ pulmonary pressureÆ critical opening pressure met for previously
unopened vesselsÆ new parallel pathways openÆ ↓ PVR
c. Distention
i. ↑ pulmonary pressureÆ ↑ transmural pressure gradient of pulmonary
vesselsÆ distention of vesselsÆ ↑ vessel radiusÆ↓ PVR
6. List the neural and humoral factors that influence pulmonary vascular resistance
a. Increase PVR: (mnemonic = SANTAH LAP, do you like santah’s lap?)
i. Sympathetic innervation (S)
ii. α-Adrenergic agonists (A)
iii. Norepi, epi (N)
iv. Thromboxane (T)
v. Angiotensin (A)
vi. Histamine (H)
vii. Low pH if mixed venous blood, blood returning from the heart (L)
viii. Alveolar hypoxia or Hypercapnia (A)
ix. PGF2α, PGE2 (P)
b. Decrease PVR:
i. Parasympathetic innervation
ii. Acetylcholine
iii. β-Adrenergic agonists
iv. PGE1
v. Prostacyclin (PGI2)
Brown, Eriksen, Jones, Heffernan, Kanjanavaikoon, Leo, Mishkin, Stern, Verlander &Wasserman
Ed. Mishkin 2006
Page 84 of 168
vi. Nitric oxide
vii. Bradykinin
7. Describe the effect of gravity on pulmonary blood flow
a. Lower parts of the lung (gravity dependent) better perfused (more pulmonary blood
flow) than upper parts of lung
8. Describe the interrelationships of alveolar pressure, pulmonary arterial pressure, and
pulmonary venous pressure and their effects on the regional distribution of
pulmonary blood flow.
a. When alveolar pressure (PA) ≥ pulmonary arterial pressure (Pa), perfusion of the
lung stops. The lung is divided into zones based upon this relationship and their
relationship to pulmonary venous pressure (PV)
b. Zone 1
i. The highest (gravity-independent) portion of the lung
ii. PA> Pa> PVÆ Pressure in the alveoli exceeds pressure in the arterioles, so
there is no blood flow into the alveoli
iii. Alveolar dead space (i.e. ventilated but not perfused)
iv. In healthy individuals not present, even at rest
c. Zone 2
i. The middle portion of the lung
ii. Pa> PA> PVÆNow pressure in the arterioles exceeds pressure in the alveoli,
and the effective driving pressure into the alveoli is Pa- PA
iii. Driving pressure ↑ in lower lung
iv. Ventilated and perfused
d. Zone 3
i. The bottom portion of lung
ii. Pa> PV> PAÆPressure in the arterioles still exceeds that of the alveoli, but
now pressure in the venules also exceeds that in the alveoli. Blood flow
continues, but now the effective driving pressure is Pa- PV
iii. Driving pressure remains fairly constant as move down the lung b/c
hydrostatic pressure of arteries= that of veins
iv. Ventilated and perfused
9. Predict the effects of alteration in alveolar pressure, pulmonary arterial and venous
pressure, and body position on the regional distribution of pulmonary blood flow.
a. ↑ Zone 1 area
i. Alveolar pressure changes with each breath, even more so with speech,
exercise, etc.
ii. A positive end expiratory pressure (PEEP) ventilatorÆ ↑alveolar pressure
iii. Hemmorrhage Æ ↓ pulmonary blood flow and pressure
iv. General anesthesiaÆ ↓ pulmonary blood flow and pressure
b. ↑ Zone 2 areaÆ recruit Zone 1 to Zone 2
i. ↑CO
ii. ↑ pulmonary artery pressure
c. Changes in body positionÆ alter orientation of zones
i. Zones retain same relationship with respect to gravity
10. Describe hypoxic pulmonary vasoconstriction and discuss its role in localized and
widespread alveolar hypoxia.
a. Hypoxic pulmonary vasoconstriction (that’s right, HPV) diverts venous blood flow
away from poorly ventilated areas by locally increasing vasoconstriction.
b. Localized atelectasis or obstruction of an airwayÆ localized HPV (just where the
obstruction is)
Brown, Eriksen, Jones, Heffernan, Kanjanavaikoon, Leo, Mishkin, Stern, Verlander &Wasserman
Ed. Mishkin 2006
Page 85 of 168
c. Hypoxia of entire lungÆ HPV throughout entire lungÆ ↑pulmonary artery
pressureÆ recruit unperfused capillariesÆ ↑ gas exchange
d. HPV Mechanism: Hypoxia decreases outward potassium current Æ pulmonary
vascular smooth muscle depolarizes Æ calcium enters Æ contraction
11. Describe the causes and consequences of pulmonary edema.
a. Causes
i. ↑ permeability of pulmonary capillary endothelium
ii. ↑ capillary hydrostatic pressure
iii. ↓ interstitial hydrostatic pressure
iv. ↑ reflection coefficient
v. ↓ capillary colloid osmotic pressure
vi. ↑ interstitial colloid osmotic pressure
vii. lymphatic insufficiency
viii. other conditions: head injury, heroin overdose, high altitude
b. Consequences
i. Æ diffusion of gases (particularly oxygen) decreases

Ventilation-Perfusion Relationships
The VA/Q concept; shunt; Physiological Dead Space; Regional Distribution of VA/Q
March 22nd, 2006
Levitzky
I. Predict the consequences of mismatched ventilation and perfusion
a. Understanding Ventilation-Perfusion relationships (VA/QC)
i. Va/Qc and V/Q are interchangeable. Don’t stress.
i. For optimal gas transfer to occur in the lung, ventilation and perfusion
must match
ii. VA = alveolar ventilation, QC = perfusion (flow through alveolar capillary)
iii. Alveolar ventilation is about 4-6 L/min and pulmonary blood flow (which
is equal to cardiac output) has a similar range Æ V/Q for whole lung = 0.8
to 1.2
1. However, V and Q must be matched on the alveolar-capillary level
and the V/Q for the whole lung is really of interest only as an
approximation of the situation in all the alveolar-capillary units of
the lung
2. If V/Q = 1.0, gas exchange is optimal
3. V/Q = 0 – no gas exchange because you’re not breathing
4. V/Q = ∞ – no gas exchange because you’re not bleeding… er, not
perfusing. In fact, maybe you are bleeding, profusely.
iv. Local airway responses and hypoxic pulmonary vasoconstriction (HPV)
help match ventilation and perfusion so that gas exchange can occur
b. Consequences of high and low V/Q
i. Normal V/Q = .8
1. Inspired air entering alveoli: PO2 = 150 mmHg; PCO2=0 mmHg
2. Mixed venous blood entering pulmonary capillaries: PO2 = 40
mmHg; PCO2 = 45mmHg
3. When inspired air mixes with the air already in the lungs, the partial
pressure gradients of oxygen and carbon dioxide change Æ PO2
becomes 100, PCO2 becomes 40
4. The partial pressure gradient for O2 and CO2 diffusion from
alveolus to pulmonary capillary = the difference between the partial
pressure in the alveoli and in the mixed venous blood:
Brown, Eriksen, Jones, Heffernan, Kanjanavaikoon, Leo, Mishkin, Stern, Verlander &Wasserman
Ed. Mishkin 2006
Page 86 of 168
a. For oxygen coming in, about 100-40 = 60 mmHg; for CO2
going out about 45-40 = 5 mmHg
5. Result Æ Alveolar and pulmonary capillary blood leaving lungs
have equal partial pressures of gas; for both: PO2 = 100 mmHg;
Alveolar PCO2 = 40mmHg
ii. Low V/Q Æ If V/Q in an alveolar-capillary unit decreases, gas exchange
will become more efficient Æ Alveolar PO2 will therefore fall, and alveolar
PCO2 will rise, (i.e., instead of oxygen falling in the blood, it falls in the
alveoli at first)
1. Extreme case Æ perfusion, but no ventilation as in completely
occluded airway Æ V/Q = 0 Æ air trapped in alveolus equilibrates
by diffusion with gas dissolved in mixed venous blood entering
capillary-alveolar unit Æ no gas exchange occurs, any blood
perfusing this alveolus will leave it exactly as it entered it Æ leads
to intrapulmonary shunt
a. Result Æ PAO2 = 40 mmHg; PACO2 = 45 mmHg
iii. High V/Q Æ If V/Q in alveolar-capillary unit increases, gas exchange will
become less efficient, as the ventilation is now perfusion limited Æ
Alveolar PO2 will rise, and alveolar PCO2 will fall
1. Extreme case Æ ventilation, but no perfusion as in blocked blood
flow by pulmonary embolism Æ V/Q = ∞ Æ No gas exchange
into or out of blood can occur because blood is not perfusing the
alveoli Æ Gas composition of unperfused alveolus is the same as
that of inspired air Æ Alveolar dead space
a. Result Æ Alveolar PO2 = 150 mmHg; Alveolar PCO2 = 0
mm Hg
b. If this alveolar-capillary unit were unperfused because
alveolar pressure exceeded capillary pressure (rather than
because of an embolus), Æ Zone 1
iv. There is a continuum of V/Qs ranging from 0 to ∞, resulting in a range of
PO2s and PCO2s, the ventilation-perfusion ratio line, as shown on the O2-CO2
diagram (p. 116, Figure 5-2)
1. LOW V/Q Æ relatively LOW PO2s and HIGH PCO2s
2. HIGH V/Q Æ relatively HIGH PO2s and LOW PCO2s
II. Describe the methods used to assess the matching of ventilation and perfusion
a. Testing for Mismatched Ventilation & Perfusion
i. Includes calculations of
1. the physiologic shunt
2. physiologic dead space
3. alveolar-arterial oxygen difference
4. single breath carbon dioxide test
5. lung scans after inhaled and I.V. administered marked Xe and Tc
6. Multiple inert gas elimination technique
ii. R to L shunt is the mixing of venous blood that has not been fully
oxygenated into the arterial blood
1. Dead space is ventilation without perfusion. Shunts are perfusion
without ventilation.
2. The intrapulmonary shunts can be absolute shunts or they can be
“shunt-like” states, that is, areas of low V/Q ratios in which alveoli
are underventilated and/or overperfused
3. Physiologic Shunt = Anatomic Shunt + Intrapulmonary Shunt
Brown, Eriksen, Jones, Heffernan, Kanjanavaikoon, Leo, Mishkin, Stern, Verlander &Wasserman
Ed. Mishkin 2006
Page 87 of 168
iii. Physiologic Shunts Æ present when the V/Q is less than 1 Æ There is low
alveolar PO2, which causes a low PO2 and Hb saturation in the blood leaving
this area of the lungs. When this blood mixes with blood from better
ventilated areas, it produces a decrease in the overall arterial PO2 and
content
1. Anatomic Shunts Æ consists of systemic venous blood entering
the L ventricle without having entered the pulmonary vasculature.
In a healthy person, about 2-5% of the cardiac output enters the
left side of the circulation directly without passing through the
pulmonary capillaries (includes venous blood from bronchial veins,
thebesian veins, which supply the myocardium, and pleural veins)
a. Pathologic anatomic shunts include R to L intracardiac
shunts, as in tetralogy of Fallot
2. Absolute Intrapulmonary Shunts Æ Mixed venous blood perfusing
pulmonary capillaries associated with, like, totally unventilated or
collapsed alveoli constitutes an absolute shunt Æ no gas exchange
occurs as the blood passes through these parts of the lung Æ
referred to as true shunts
a. V/Q = 0 in a true shunt
3. Shunt-like States Æ alveolar-capillary units with low V/Q ratios,
act to lower the arterial PO2 because blood draining these units has
a lower PO2 than blood from units with well-matched V and Q
4. The Shunt equation Æ conceptually divides all alveolar-capillary
units into those that have well-matched V/Q and those with V/Qs
approaching 0 Æ Shunt equation combines areas of absolute shunt
and shunt-like areas Æ The resulting ratio of shunt flow to the
cardiac output is referred to as the venous admixture
3. Venous admixture Æ the part of cardiac output that would
have to be perfusing absolutely unventilated alveoli to cause the
systemic arterial O2 content obtained from a patient
4. Pulmonary venous admixture (SHUNT EQUATION)

Qs/Qt = ( CcO2 - CaO2)/( CcO2 - CvO2)

Qs = amount of blood flow per minute entering the systemic


arterial blood without receiving any O2 (shunt flow)
Qt = total pulmonary blood flow per minute (CO)
Cc′O2 = End capillary O2 content
CaO2 = arterial O2 content
C⎯⎯v O2 = O2 content of the mixed venous blood
• The shunt equation is multiplied by 100 so that shunt flow
is expressed as % of cardiac output
5. CaO2v &C⎯⎯v O2 can be measured from blood samples
6. O2 content of blood at end of pulmonary capillaries with well-
matched V/Q ratios (Cc′O2) must be calculated using the
alveolar air equation and the patient’s Hb concentration
7. At LOW inspired O2 concentrations, Qs/Qt will include both
true shunts and alveolar-capillary units with low V/Q
1. After patient has inspired 100% O2 for 30 minutes,
even alveoli with low V/Qs will have high enough
Brown, Eriksen, Jones, Heffernan, Kanjanavaikoon, Leo, Mishkin, Stern, Verlander &Wasserman
Ed. Mishkin 2006
Page 88 of 168
alveolar PO2s to completely saturate the Hb in the
blood perfusing them
2. These units will no longer contribute to the calculated
Qs/Qt and the new calculated shunt should include
only areas of absolute (true) shunts
8. At HIGH inspired O2 concentrations, atelectasis may occur in
otherwise very poorly ventilated alveoli that remain perfused,
and so by asking a patient with restrictive disease to inspire
high O2, in order to assess how much blood is being
oxygenated vs. physiologically shunted, you may in fact be
altering what you were trying to measure because the FIO2 is so
high (results may be skewed towards the shunt seeming larger
than it really is)
b. Physiologic dead space = anatomic dead space + alveolar dead space
i. Anatomic dead space (VD) Æ volume of gas that occupies “conducting
zone” i.e., the airways, which do not participate in gas exchange
ii. Alveolar dead space Æ alveolar-capillary units with V/Q = ∞, no blood
flow, and therefore no gas exchange – this occurs in pulmonary embolism
1. Alveolar dead space Æ arterial-alveolar CO2 difference
a. the end tidal PCO2 is normally equal to the arterial PCO2,
but: alveolar dead space Æ CO2 cannot diffuse out of
blood into alveoli Æ CO2 cannot be expired Æ raising
arterial PCO2 Æ arterial PCO2 greater than the end tidal PCO2
2. The Bohr equation is used to determine the physiologic dead space
by determining CO2 tensions in expired and alveolar gases…
Vd Pa CO2 − PE CO2
=
VT Pa CO
2

• Arterial CO2 (PaCO2) almost always = Alveolar CO2 (PACO2)


• PECO2 = CO2 tension in mixed expired air
• PACO2 = CO2 tension in alveolar gas (estimated by end tidal
samples)
• Increase in dead space Æ lower CO2 tension in expired air Æ
greater ratio of dead space to minute ventilation
c. Alveolar-arterial O2 difference Æ The difference or gradient between the partial
pressure of O2 in the alveolar spaces and the arterial blood
v. Normally, 5-15 mm Hg (represents mismatch between arterial and
alveolar pressure)
1. factors include normal anatomic shunt, some degree of V/Q
mismatch, and diffusion limitation in some parts of the lung
vi. Larger-than-normal differences between alveolar and arterial PO2,
indicate significant V/Q mismatch
vii. May also be caused by anatomic or intrapulmonary shunts, diffusion
block, low mixed venous PO2s, breathing higher than normal O2
concentrations, or shifts of the oxyhemoglobin dissociation curve (p.,
180, Table 8-6)
d. Single-breath CO2 test:
i. Expired concentration of CO2 can be monitored to determine V/Q
mismatch
ii. Patient inspires and then exhales to end-tidal volume, and a plateau of
expired CO2 is recorded
Brown, Eriksen, Jones, Heffernan, Kanjanavaikoon, Leo, Mishkin, Stern, Verlander &Wasserman
Ed. Mishkin 2006
Page 89 of 168
1. expired air is (in this order):
a. air from the anatomic dead space
b. mixed alveolar and anatomic dead space air
c. gas from all the ventilated alveoli (which have CO2 in them
to be expired)
iii. V/Q mismatch is observed if regions of lung empty asynchronously
e. Lung scan of inhaled and injected radioactive markers can be used to inspect the
location and amount of ventilation and perfusion to the various regions of the lung
f. Multiple Inert Gas Elimination Technique Æ Elimination via the lungs of different
gases dissolved in the mixed venous blood is affected differently by variations in the
ventilation-perfusion ratios of alveolar-capillary units, according to the solubility of
each gas in the blood
i. Gases with low solubility in the blood would be retained in the blood only
by units with very low (or zero) V/Qs
ii. Gases with high solubility in the blood would be eliminated mainly in the
expired air of units with very high (like maybe 10) V/Qs
iii. Mixture of 6 gases dissolved in saline infused into peripheral vein at 2-
5mL/min until steady state of gas exchange is established (about 20
minutes)
1. Samples of expired air are collected along with measurements
including cardiac output by indicator dilution, minute ventilation,
and arterial and MVB gases
2. In a young healthy patient at rest, V/Q is consistently near 1, with
some range. Having units with a V/Q <0.3 or >3.0 is extremely
unusual.
3. A middle-aged healthy person, there is a wider dispersion of V and
Q, with more units with ratios >3.0 and to units with ratios <0.3
4. See Levitzky’s 5-5 for a graph
III. Describe the methods used to determine the uniformity of the distribution of the
inspired gas and pulmonary blood flow
a. Non-uniform gas distribution Æ Non-uniform ventilation of the alveoli can be
caused by uneven resistance to airflow or nonuniform compliance in different parts
of the lung
i. Uneven resistance to airflow Æ result of collapsed airways (e.g.
emphysema); bronchoconstriction (e.g. asthma); decreased lumen diameter
due to inflammation (e.g. bronchitis); obstruction by mucus; compression
by tumors of edema
ii. Uneven compliance Æresult of fibrosis, regional variations in surfactant
production, pulmonary vascular congestion or edema, emphysema,
atelectasis, pneumothorax, compression by tumors or cysts
1. Single-breath-of-O2 test Æ Monitor expired N2 concentration in
"alveolar plateau" phase (p.80, Figure 3-14). Steep plateau = non-
uniform gas distribution.
a. In a healthy person, the slope of phase III is nearly
horizontal. To review what phase III means (also
discussed in 3/21 notes) here is a picture.
Brown, Eriksen, Jones, Heffernan, Kanjanavaikoon, Leo, Mishkin, Stern, Verlander &Wasserman
Ed. Mishkin 2006
Page 90 of 168
I. S
e
e

n
o
t
e
s

f
o
r

M
a
r
c
h

II.
b. In those with areas of increased airway resistance, phase
III slope is steep Æ alveoli supplied by high-resistance
airways fill more slowly than those supplied by the normal
airways during the 100% O2 inspiration Æ These alveoli
have a higher N2 concentration Æ Over the course of one
expiratory effort, as the varyingly supplied alveoli empty at
different rates, oxygen concentration of expired air will
decrease and nitrogen concentration of expired air will
increase, as the source of expired air shifts from “faster” to
“slower” alveoli
2. Nitrogen-Washout Test Æ Patient breathes 100% oxygen and the
expired N2 concentration is monitored over a number of breaths.
With successive respirations, expired end-tidal N2 concentration
falls as N2 is “washed” out of the lung
a. Patients with normal distribution of airways resistance will
reduce their expired end tidal N2 concentration to less
than 2.5% within 7 minutes
b. Patients breathing normally who take more than 7 minutes
to reach an alveolar N2 concentration of less than 2.5%
have high resistance pathways, or “slow alveoli”
3. Trapped Gas Æ Differences between the FRC determined by
helium-dilution technique and the FRC determined using a body
plethysmograph may indicate gas trapped in the alveoli because of
airway closure
4. Radioactive Markers Æ Patients take breath of 133Xe or 99mTc
DTPA and oxygen mixture, and a picture of the whole lung is
taken with a scintillation counter to indicate which regions of the
lung are poorly ventilated
iii. Nonuniform Distribution of Pulmonary Blood Flow Å embolization or
thrombosis; compression of pulmonary vessels by high alveolar pressures,
tumors, exudates, edema, pneumothorax, hydrothorax; destruction or
Brown, Eriksen, Jones, Heffernan, Kanjanavaikoon, Leo, Mishkin, Stern, Verlander &Wasserman
Ed. Mishkin 2006
Page 91 of 168
occlusion of pulmonary vessels, pulmonary vascular hypertension, or
collapse or overexpansion of alveoli
1. These tests indicate the locations of relatively large regions of poor
perfusion…
1. Pulmonary angiograms
2. Lung scan after injection of 131I or technetium labeled
macroaggregates of albumin
3. Lung scan after intravenous 133Xe
IV. Explain the regional differences in the matching of ventilation and perfusion of
the normal lung
a. See LOs from 3/21
V. Predict the consequences of the regional differences in the ventilation and
perfusion of the normal upright lung
a. See LOs from 3/21
VI. Classify and explain the causes of tissue hypoxia
a. Hypoxia Æ inadequate O2 supply to the body tissues. Disease processes can
severely limit the O2 supply anywhere between the atmosphere and the body’s cells.
Hypoxia occurs “downstream” of the limitation (i.e. toward the cells); a normal O2
tension may be present “upstream” (i.e. toward the environment)
b. Types of Hypoxia
i. Arterial hypoxia Æ inadequate oxygenation of the arterial blood, caused by
breathing gas with a low O2 tension or by pathology
1. Hypoventilation Æ reduces alveolar and arterial O2 tensions,
increases the alveolar and arterial CO2 tensions Æ Hypercapnia
2. Diffusion limitation Å reduction in diffusing capacity of lung
secondary to pulmonary disease that prevents equilibration
between O2 tension in alveoli and pulmonary capillaries
3. Physiologic shunts (V/Q ratio imbalances) Æ produce low O2
tensions in the areas of the lung with low V/Q ratios
a. V/Q imbalance is by far the most common cause of
hypoxia
b. Administration of 100% O2 to affected patients can
correct hypoxia because O2 flushes N2 from alveoli, and
alveolar O2 tension, even in low V/Q areas will rise to
functional levels
4. Anatomic Shunts Æ mixing of true venous blood and arterial
(oxygenated) blood Æ diluting normal O2 concentration
a. Arterial O2 tension reduced in proportion to the fraction
of cardiac output that is shunted Æ Normal individuals
have an anatomic shunt of less than 5% of cardiac output
ii. Ischemic hypoxia Æ inadequate blood flow
1. Reduced blood flow may involve entire body (e.g. congestive heart
failure) or localized area (e.g. arteriosclerosis)
2. Atherosclerosis most common cause of arterial obstruction Æ
increases local vascular resistance and reduced blood flow
3. Æ Decreased O2 delivery to tissue Æ infarction and dysfunction
4. Arterial O2 tension and content may be normal, but tissues
withdraw large amounts of O2 from capillary blood Æ as a result,
venous O2 content is reduced
iii. Anemic Hypoxia Æ insufficient amount of functional Hb
Brown, Eriksen, Jones, Heffernan, Kanjanavaikoon, Leo, Mishkin, Stern, Verlander &Wasserman
Ed. Mishkin 2006
Page 92 of 168
1. May be caused by deficiency of nutrients (e.g. iron, B12) or due to
blood loss or large amounts of methemoglobin or
carboxyhemoglobin
2. Æ ↓ O2 capacity Æ ↓ O2 content
iv. Histotoxic Hypoxia Æ inactivation of certain metabolic enzymes (e.g.
cytochromes) and by chemical poisons (e.g. cyanide)
1. Tissues unable to use O2, as a result, venous O2 tension and
content are high

Diffusion
Fick’s Law of Diffusion; Hypoxia; Diffusion Capacity of the Lung; Diffusion v. Perfusion
Limits to Gas Transfer
March 22nd, 2006
Levitzky
I. Diffusion vs. “bulk flow”
a. Diffusion is net flow down a concentration gradient
i. Faster at higher temperatures
ii. Net movement of a gas is always down the partial pressure gradient for that
gas
iii. The blood must be exposed to a partial pressure for a finite time for gas to
equilibrate between the gas and liquid phases.
1. The time required for equilibration is a function of the contact area
between the liquid and the gas (surface area), the solubility and
diffusion properties of the gas, and the diffusion gradient
2. Physiologic, resting time spent in pulmonary capillaries for each
RBC is 0.75 – 1.25 seconds
b. Gas moves through airways by bulk flow until it reaches the terminal bronchioles Æ
diffuses into pulmonary capillaries Æ bulk flow though pulmonary veins Æ
diffusion out of arterial blood into tissue, and from tissue into venous blood Æ
moves by bulk flow from the capillary beds to the pulmonary vasculature Æ
diffuses from venous blood into alveoli Æ expired by bulk flow
II. State Fick’s Law for Diffusion
a. Fick’s Law Æ defines the rate of pulmonary gas diffusion (i.e. the volume of gas per
minute that crosses the alveolar-capillary membrane):

Vgas = gas flow (mL/min)


A = area
T = thickness
D = diffusivity
P1 – P2 = partial pressure gradient
The rate of gas moving across a membrane is directly proportional to the surface
area of the sheet, the diffusivity, and the difference in gas concentration between the
two sides, but is inversely proportional to the membrane thickness.
b. The potential surface area (A) of the blood-gas barrier is 70m2 in an average healthy
adult at rest
i. ↑ capillary recruitment, as in exercise, ↑ surface area
ii. ↓venous return, as in hemorrhage or ↑alveolar pressure by
PEEPÆcapillaries may be derecruited and ↓surface area available for
diffusion
Brown, Eriksen, Jones, Heffernan, Kanjanavaikoon, Leo, Mishkin, Stern, Verlander &Wasserman
Ed. Mishkin 2006
Page 93 of 168
c. Thickness (T) of the alveolar-capillary diffusion barrier = 0.2 – 0.5 μm Æ
i. This can increase in interstitial fibrosis or edema
d. Diffusivity, or diffusion constant (D), for a gas is directly proportional to the
solubility of the gas in the diffusion barrier and is inversely proportional to the
square root of the molecular weight (MW) of the gas:

i. The solubility of CO2 in the liquid phase is 24 times that of O2, and diffuses
about 20 times more rapidly though the alveolar-capillary barrier than does
O2
1. Æ patients develop problems in O2 diffusion before CO2
diffusion retention problems
III. Distinguish between perfusion limitation and diffusion limitation of gas transfer
in the lung
a. Limitations of gas transfer Æ the partial pressure of a gas in the mixed venous
blood and in the pulmonary capillaries is just as important as the alveolar partial
pressure in determining its rate of diffusion
i. Summary of gas transfer limitations
1. If partial pressure of a gas in the plasma equilibrates with the
alveolar partial pressure of the gas within the amount of time
the blood is in the pulmonary capillary Æ perfusion limited
a. Example: Nitrous oxide (N2O), which dissolves into
blood and is not taken up by any carriers, therefore
exerting its full partial pressure, diminishing its partial
pressure gradient for that unit of blood and forcing it
to wait for new blood in order to continue diffusing
2. If equilibration does not occur within the time the blood is in
the capillary Æ diffusion limited
a. Example: Carbon monoxide, which binds to Hb and
therefore does not exert its partial pressure, allowing it
to keep diffusing as fast as it is able to diffuse given its
own properties and those of the capillaries
IV. Describe the diffusion of O2 from the alveoli into the blood
a. Gas moving from alveolus to blood must pass through: pulmonary surfactant Æ
Alveolar epithelium Æ Interstitium Æ Capillary endothelium Æ Plasma Æ RBC
membrane to reach Hb
b. Oxygen Æ O2 binds to Hb and exerts no partial pressure in pulmonary capillary
blood, so the partial pressure gradient is well maintained across the alveolar-capillary
membrane Æ O2 transfer occurs
i. Diffusion of O2 Æ normal alveolar PO2 is 100 mm Hg, blood entering the
pulmonary capillary has a PO2 of 40 Æ After dissolving across the alveolar-
capillary membrane, the O2 diffuses into the plasma, raising the plasma O2
tension Æ O2 then diffuses into the RBC where it combines with Hb
ii. The binding of O2 to Hb occurs within 0.01s, and at normal alveolar partial
pressure of O2, Hb becomes saturated very quickly Æ Thus, the partial
pressure of O2 in the blood rises to that in the alveolus, and from that
point, no further O2 transfer occurs
iii. Under normal alveolar PO2 and a normal resting cardiac output Æ O2
transfer from alveolus to pulmonary capillary is perfusion-limited
Brown, Eriksen, Jones, Heffernan, Kanjanavaikoon, Leo, Mishkin, Stern, Verlander &Wasserman
Ed. Mishkin 2006
Page 94 of 168
iv. During exercise ↑ COÆ blood spends less time in the pulmonary capillary
Æ O2 transfer then approaches diffusion limitation
V. Describe the diffusion of CO2 from the blood to the alveoli
a. Gas moving from blood to alveolus must pass through: RBC membrane Æ Plasma
Æ Capillary endothelium Æ Interstitium Æ Alveolar epithelium Æ pulmonary
surfactant
b. CO2 Æ time course for CO2 transfer is similar to O2
i. Average CO2 tension in pulmonary capillary blood is 46 mm Hg, and 40
mmHg in the alveoli Æ the CO2 diffusion gradient is only 1/10 of the O2
gradient Æ But remember, CO2 diffuses 20 times more rapidly than O2
ii. CO2 transfer is normally perfusion-limited, although it may be diffusion
limited in a person with abnormal alveolar-capillary barrier
VI. Define the diffusion capacity and discuss its measurement
a. The diffusion properties of the lungs are evaluated by measuring the diffusing
capacity: (Vco = ventilation rate of carbon monoxide, PACO = arterial tension of
carbon monoxide)

b. The diffusing capacity of the lungs is measured using


a single-breath technique and dilute carbon monoxide
(about 0.01%)
c. Diffusing capacity (or transfer factor) Æ rate at which O2 or CO is absorbed from
the alveolar gas into the pulmonary capillaries (in mL/min) per unit of partial
pressure gradient (in mm Hg)
d. Normal DLCO Æ 30 mL/min/mm Hg
e. Diffusing capacity of lungs for O2 (DLO2) Æ 25 mL/min/mm Hg
f. DLCO is decreased in diseases associated with…
i. Interstitial or alveolar fibrosis (sarcoidosis, scleroderma, and asbestosis)
ii. Interstitial or alveolar pulmonary edema
iii. Decrease in surface area available for diffusion (emphysema, tumors, low
cardiac output, low pulmonary capillary blood volume, decreased V/Q)

Transport of Respiratory Gases in the Blood


March 23, Levitzky
Oxygen: Physically dissolved & combined with hemoglobin
Carbon Dioxide: Physically dissolved as carbamino compounds & bicarbonate
1. Relationship between partial pressure of blood oxygen and the actual amount of
oxygen dissolved in the blood
a. Partial pressure reflects the amount of oxygen physically dissolved in the blood,
which is only a very small portion of the total blood oxygen content
i. 0.00003 ml O2/ torr PO2
ii. normal arterial blood (100 mm Hg) has about .3mL O2/100mL blood
b. Oxygen content also includes the amount of oxygen that binds to the Hb in
erythrocytes
2. Chemical combination of oxygen with hemoglobin – and dissociation curve
a. Each of four polypeptide chains bind a molecule of oxygen to the iron atom in its
heme group
b. Hemoglobin rapidly, reversibly combines with oxygen (if it weren’t reversible, our
blood would have plenty of oxygen and our muscles would have none)
3. Hemoglobin saturation, oxygen-carrying capacity, oxygen content of blood
a. saturation:
i. 1 gram hemoglobin carries 1.34 ml of oxygen at saturation
Brown, Eriksen, Jones, Heffernan, Kanjanavaikoon, Leo, Mishkin, Stern, Verlander &Wasserman
Ed. Mishkin 2006
Page 95 of 168
ii. O2 bound to Hb = grams Hb X 1.34 X %saturation
iii. % Hb saturation = O2 bound to Hb / O2 capacity of Hb X 100%
b. capacity:
i. Capacity is in mL oxygen per 100 mL blood, so it depends on the Hb
capacity and also how much Hb you have
ii. O2 capacity of Hb = grams of Hb x 1.34
iii. Normal = 13-15 grams of hemoglobin per 100 ml of blood
iv. Average capacity = 20.1 mL (per 100 mL)
c. content:
i. content = Hb-bound oxygen + dissolved oxygen
4. Physiologic consequences of the oxygen dissociation curve shape
a. The oxygen dissociation curve is S-shaped, because it is steep at lower oxygen
pressures and almost flat as pressure rises above 70 mm Hg.
i. the shape reflects 4 binding events, and because there is positive
cooperativity among these binding sites, the curve is not linear
ii. there is also cooperativity in oxygen dissociation, facilitating efficient
oxygen delivery
iii. between 10 mm and 40 mm Hg (PO2), the curve is very steep, indicating
that this is the range in which the outcome is most associated with the
partial pressure
5. Physiologic factors influencing oxygen dissociation curve and their effects on oxygen
transport
a. decreased PCO2, increased pH, decreased temp and decreased BPG shift curve to
the left (opposites shift to right)
i. In other words, all of the signs that tell erythrocytes they are in muscle shift
the curve to the right, while decreasing the oxygen affinity and allowing
oxygen to be released to the muscles, and all of the signs that tell
erythrocytes they are in the lungs, shift the curve to the left, increasing the
oxygen affinity and pulling more oxygen onto the heme groups.
ii. All of these effects are more pronounced at lower PO2, so the role of each
of these factors is probably greater in the muscle than in the lungs
iii. Temperature: oxygen is more soluble in water or plasma at lower
temperatures. At 20oC, 50% more oxygen will dissolve in plasma
b. Anemia decreases peak (depressing content curve)
i. anemia reduces the amount of Hb, so even at saturation, there is lowered
oxygen content in the blood
c. CO decreases peak
i. Carbon monoxide is a high-affinity competitive inhibitor
ii. CO also shifts the oxyhemoglobin dissociation curve to the left –
preventing unloading of oxygen
iii. Normal blood levels for carboxyhemoglobin (CO bound to Hb) range from
1% in a rural non-smoker to 8% in an urban smoker.
d. Nitric oxide reacts with oxyhemoglobin (oxygen bound to Hb) Æ methemoglobin
and nitrate
i. Nitric oxide can also be transported by HB Æ S-nitrosohemoglobin or
SNO-Hb Æ released along with O2 therefore released in low PO2 Æ
vasodilates
e. Methemoglobin
i. When the iron is in the ferric state (Fe3+) it is “methemoglobin” and will
not combine with oxygen.
1. This can be caused by toxins and lack of vitamin C (I heart
nutrition tests!)
Brown, Eriksen, Jones, Heffernan, Kanjanavaikoon, Leo, Mishkin, Stern, Verlander &Wasserman
Ed. Mishkin 2006
Page 96 of 168
f. Genetic variants
i. Other Hb variants exist with altered characteristics
1. Normal fetal Hb has higher affinity for oxygen (because of γ
subunits)
2. Hb Seattle and Hb Kansas have lower affinities for oxygen
3. Hb Rainier has a higher affinity for oxygen
g. Fluorocarbons can be useful as emergency transfusion sources because a lot of
oxygen can just dissolve in fluorocarbon solution
h. Cyanosis occurs when more than 5g Hb/100 mL arterial blood is in the deoxy state
6. Relationship between partial pressure of carbon dioxide and the actual amount of
carbon dioxide dissolved in the blood
a. Carbon dioxide is 20X more soluble in plasma than is oxygen
i. .06mL CO2/100 mL plasma at 37oC so a total of 2.4 mL in physical
solution
ii. a little more than 5% of total carbon dioxide content of venous blood is in
physical solution
7. Transport of carbon dioxide as carbamino compounds with blood proteins
a. Carbon dioxide + terminal amine Æ carbamino
i. can be bound to hemoglobin Æ “carbaminohemoglobin”
b. Deoxyhemoglobin supports carbamino formation – it can hold more CO2 than
oxyhemoglobin can Æ as oxygenation occurs in the lungs, carbon dioxide is
released
8. How most carbon dioxide in the blood is transported as bicarbonate
a. CO2 + H20 ↔ H2CO3 ↔ H+ + HCO3-
b. catalyzed by carbonic anhydrase
c. Hb facilitates by accepting the liberated H+ ion Æ improves outcome via mass
action
d. bicarbonate is not a very effective buffer, but it is immediate
9. Carbon dioxide dissociation curve for whole blood
a. Steeper than the oxygen dissociation curve
b. Largely linear
10. Bohr and Haldane effects
a. Deoxyhemoglobin is a weaker acid than oxyhemoglobin – therefore in less
oxygenated states, hydrogen ions are more readily absorbed
b. Carbon dioxide can dissolve more at lower pH because its dissolution produces a
hydrogen ion: “Isohydric shift”
c. Hydrogen ion association with hemoglobin lowers oxygen affinity Æ
oxyhemoglobin dissociation curve is shifted to the right at low pH or high PCO2
(i.e., in tissue)
d. Bohr effect:
i. Hydrogen ions released by dissociation of carbonic acid/formation of
carbamino compounds bind to globin residues and facilitate oxygen release
e. Haldane effect:
i. blood can load more carbon dioxide at the tissues, where there is more
deoxyhemolgobin, and unload more carbon dioxide in the lungs, where
there is more oxyhemoglobin

Neural control of Respiration


March 23, Levitzky
Generation of Spontaneous Rhythmicity
1. General organization of respiratory control system
Brown, Eriksen, Jones, Heffernan, Kanjanavaikoon, Leo, Mishkin, Stern, Verlander &Wasserman
Ed. Mishkin 2006
Page 97 of 168
a. Brainstem automatically generates inspiration and expiration
b. Also affected by:
i. Reflexes can arise from the lungs, airways, and cardiovascular system
ii. Cerebrospinal receptors
iii. Cortical intervention
c. Frequency of neural discharge, the number of motor units activated, and the
duration of discharge all are correlated with outcome
2. Localization of neural centers that generate spontaneous rhythmicity of breathing
a. Medullary center (aka medullary respiratory center) is in the reticular formation of
the medulla, beneath the fourth ventricle
b. Lesions above this level spare respiration (though pattern might be irregular), lesions
below this level do not
c. Exact neuronal activity is not yet fully characterized, leading hypotheses involve
separate groups of inspiratory and expiratory neurons, which are thought to
influence the muscles of inspiration and expiration, respectively, and to be mutually
inhibitory
i. Dr.Levitzky seems rather unconvinced of these theories
3. Groups of neurons controlling inspiration and expiration
a. Two groups of respiratory neurons are found in the reticular formation
b. Dorsal respiratory groups (DRG): are thought to be responsible for driving
diaphragmatic contraction and therefore inspiration, and is probably the initial
integration site for reflexes influencing respiratory rhythm
i. located in bilateral nucelus tractus solitarius
ii. project contralaterally to spine and then diaphragm
iii. largely inspiratory
iv. ventral efferents are many, ventral afferents are few
v. largest center for CN IX and X afferents– therefore receives a lot of the
systemic information about pH, concentrations and pressures
vi. I cells increase activity when lung inflation is withheld
vii. I cells decrease activity when lung inflation is withheld
viii. P cells (pump cells) are likely to be interneurons
c. Ventral respiratory groups (VRG)
i. located in retrofacial nucelus, nucleus ambiguus, nucleus retroambiguus
ii. inspiratory and expiratory
iii. possibly pacemaking in the “pre-Bötzinger complex”
4. Other centers in brainstem that may influence spontaneous breathing rhythm
a. Apneustic center: a breathing pattern of prolonged inspiration with occasional
expirations called apneusis is caused by sustained discharge of the medullary
inspiratory neurons
i. likely to be site of inspiration termination
b. Pontine respiratory groups
i. nuclei:
1. parabrachialis medialis
2. Kölliker-Fuse nucleus
ii. Inferior colliculus as a dividing line:
1. transection just caudal to inferior colliculus Æ normal breathing
2. transection slightly lower, caudal to PRG Æ apneusis
iii. Role is to fine-tune breathing pattern
1. may modulate response to hypercapnia and hypoxia
iv. pneumotaxic center: inhibits inspiration Æ regulates inspiratory volume and
respiratory rate
c. Spinal pathways
Brown, Eriksen, Jones, Heffernan, Kanjanavaikoon, Leo, Mishkin, Stern, Verlander &Wasserman
Ed. Mishkin 2006
Page 98 of 168
i. All of these pathways go through the spine to get to the diaphragm, and
afferents resulting in respiratory adjustments also travel via the spine. No
news here.
5. Cardiopulmonary and other reflexes influencing breathing patterns: This table is Levitsky’s
9-1 with my notes.
Stimulus Reflex Name Receptor Afferent Pathway Effects
Sustained lung Hering-Breuer Stretch receptors within Vagus – projects to Cessation of inspiratory effort, apnea,
Inflation inflation reflex smooth muscle of large DRG, apneustic ↓ breathing frequency; bronchodilation
and small airways – center, PRG ↑ HR, slight vasoconstrcition
“Slowly-adapting
pulmonary stretch
receptors”
Abrupt lung Hering-Breuer Juxtaopulmonary-capillary Vagus Hyperpnea
Deflation deflation reflex or J receptors (maybe),
irritant receptors in lungs,
stretch receptors in
airways
Lung Inflation Paradoxical Stretch receptors in lungs Partially functional Deep Inspiration
reflex of Head (precise location Vagus (cold)
unknown)
Negative Pharyngeal Receptors in nose, mouth, Trigeminal, Pharyngeal dilator muscle contraction
pressure in upper dilator reflex upper airways laryngeal,
airway glossopharyngeal
Mechanical or Cough In upper airways, Vagus Cough, bronchoconstriction
chemical airway tracheobronchial tree
irritation
Mechanical or Sneeze In nasal mucosa - Trigeminal, Sneeze, bronchoconstriction, ↑BP
chemical airway “Rapidly-adapting pulm olfactory
irritation stretch receptors”
Facial immersion Diving reflex In nasal mucosa/face Trigeminal Apnea, ↓ HR, vasoconstriction
Pulmonary J receptors in pulmonary Vagus Apnea, tachypnea
embolism vessels
Pulmonary J receptors in pulmonary Vagus Tachypnea, sensation of dyspnea
vascular vessels
congestion
Chemicals Pulmonary J receptors in pulmonary Vagus Apnea, tachypnea, bronchoconstriction
chemoreflex vessels
↓PaO2, ↑ PaCO2, ↓ Arterial Carotid bodies, aortic Glossopharyngeal, Hyperpnea, bronchoconstriction,
pHa chemoreceptor bodies vagus dilation of upper airway
reflex
↑ systemic Arterial Carotid sinus stretch Glossopharyngeal, Apnea, bronchodilation, ↓ HR,
arterial BP baroreceptor receptors, aortic arch vagus vasodilation
reflex stretch receptors
Muscle, tendon, Muscle spindles, tendon Spinal Feedback about work of breathing;
joint stretch organs, proprioceptors stimulation of proprioreceptors in
joints causes hyperpnea
Somatic Pain Nociceptors Spinal Hyperpnea, ↑ HR, vasoconstriction
Visceral pain Apnea, ↓ ventilation

6. Temporary cortical override of breathing patterns


a. Voluntary override
Brown, Eriksen, Jones, Heffernan, Kanjanavaikoon, Leo, Mishkin, Stern, Verlander &Wasserman
Ed. Mishkin 2006
Page 99 of 168
i. MVV: Maximum voluntary ventilation
ii. Breathholding can temporarily override involuntary breathing
iii. Useful for speech, singing, playing a wind instrument, playing dead, etc.
iv. Chronic hyperventilation Æ respiratory alkalosis

Chemical control of Ventilation


March 23, Levitzky
Response to Hypoxia, Hypercapnia & Hydrogen ions
1. Effects of alterations in body oxygen, carbon dioxide and hydrogen levels on breathing
a. Oxygen
i. Hypoxia
1. Peripheral chemoreceptors only
2. Carotid bodies primarily
3. increased oxygen Æ depression of central respiratory controller
4. 50-60mmHg is normal
5. High PCO2 potentiates
6. PO2 and not oxygen content is what controls this – so anemia
without acidosis does not stimulate ventilation
ii. Exercise
1. increased need for oxygen can increase minute ventilation
a. increases tidal volume and breathing frequency
2. up to 60% of capacity, minute ventilation increases linearly
3. above 60%, minute ventilation increases faster than oxygen
consumption, but continues to rise proportionally to increase in
carbon dioxide
a. this is because of increased lactic acid Æ H+ Æ arterial
chemoreception
4. Neural element possibly in hypothalamus
b. Carbon Dioxide
i. High CO2 increases respiration
1. High = >40mm Hg
2. Linear relationship, steep slope
ii. Negative feedback system
iii. Can lead to increased H levels Æ hard to parse which drives ventilation
iv. Inspired CO2 above normal levels
1. leads to dyspnea, headaches, restlessness, faintness, dulling of
consciousness
2. > 15% can lead to LOC, muscular rigidity, tremors
3. >20% leads to convulsions
v. Other influences
1. sleep, narcotics, anesthesia shift curve to the right, i.e., decreases
the amount of ventilation in response to increased carbon dioxide
level
a. respiratory depression is the most common cause of death
in overdose of opiate alkaloids, barbiturates, anesthetics
c. Hydrogen Levels
i. Between 20nEq/L and 60nEq/L, there is a linear relationship between
[H+] and ventilation
ii. Metabolic acidosis:
1. Acidotic stimulation of the peripheral chemoreceptors Æ increased
alveolar ventilation Æ arterial CO2 falls Æ CSF CO2 falls (CSF and
blood are in dynamic equilibirum) Æ pH of CSF increases Æ
Brown, Eriksen, Jones, Heffernan, Kanjanavaikoon, Leo, Mishkin, Stern, Verlander &Wasserman
Ed. Mishkin 2006
Page 100 of 168
decreased stimulation of central chemoreceptor Æ breathing is
normalized
2. This is counterintuitive because hyperventilation would help reduce
acidosis – now, instead, metabolic factors will have to increase pH
3. Central chemoreceptor responds more to CO2 in the CSF than to
pH in the blood, which is why this overly complicated mechanism
is the one driving ventilation
2. Sensors of respiratory system for oxygen, carbon dioxide and hydrogen levels
a. Peripheral chemoreception
i. control 10-20% of the steady state respiration
1. up to 1/3 of control when arterial carbon dioxide levels are rapidly
shifting
ii. arterial chemoreceptors – in carotid and aortic bodies
1. exposed to blood
iii. Increase firing rate in response to
1. Increased PCO2
2. Decreased PO2
3. Decreased pH
iv. System is rapid and sensitive – i.e., the speed and air amount of one breath
v. Carotid bodies effect breathing more than aortic bodies do
vi. Some drugs act on these sensors
1. cyanide, dinitrophenol stimulate carotid body Æ hypoxia
b. Central chemoreception
i. control 80-90% of the steady state respiration
ii. located bilaterally near the ventrolateral surface of the medulla
1. exposed to CSF
2. On brain side of blood-brain barrier
a. CO2 can diffuse, but bicarb can’t
b. Carbon dioxide is normally at 50mmHg in the brain
3. Do not respond to hypoxia – only carbon dioxide and H

Acid-Base Balance
3/24/06, Levitzky

1. Define acids, bases, and buffers.


a. Acids can donate hydrogen ions
i. Strong acids almost completely dissociate in water
b. Bases can accept hydrogen ions
c. Buffer- a mixture of substances in solution that resists changes in hydrogen ion
concentration when acids or bases are added
i. Buffers are usually composed of a weak acid and its conjugate base
2. List the buffer systems available in the human body.
a. Buffering ability is measured as buffer value
i. Amount of hydrogen ions (mEq/L) that can be added to or removed from
a solution with a resultant change of one pH unit
b. Bicarbonate Buffer System- the major buffer system in our bodies
i. CO2 + H2O↔ H2CO3↔H+ + HCO3-
ii. carbonic anhydrase is catalyst
c. Phosphate Buffer System
i. H2PO4-↔ H+ + HPO42-
d. Proteins
i. Plasma proteins, specifically hemoglobin
Brown, Eriksen, Jones, Heffernan, Kanjanavaikoon, Leo, Mishkin, Stern, Verlander &Wasserman
Ed. Mishkin 2006
Page 101 of 168
1. the imidazole group of histidine has buffering capacity, and there
are many histidines in hemoglobin
3. Describe the interrelationships of the pH, the PCO2 of the blood, and the plasma
bicarb concentration and understand the modified Henderson equation.
a. Henderson-Hasselbalch equation
i. pH= pK + log [A-]/[HA]
ii. This equation can be modified for the bicarbonate buffering system
1. pH=pK’ + log [HCO3-]p/[0.03xPCO2]
a. pK’ here is the pK for bicarb
2. [HCO3-]p is the plasma concentration of bicarb, which is the base
in the dissociation equation (see 2.b.i.)
3. Carbon dioxide and carbonic acid are the undissociated acids,
(concentration of carbonic acid is negligible).
4. 0.03 mmol of CO2 per mmHg will dissolve in a liter of plasma
b. The relationship between pH, the PCO2 of the blood, and the plasma bicarb can be
expressed in the pH-bicarbonate diagram (Figure 8-1)
i. Bicarb is the y-axis, pH is the x-axis
ii. ↑ pHÆ ↓ [HCO3-]p
iii. Curve gets shifted to the right with increasing PCO2
4. State the normal ranges of arterial pH, PCO2 and bicarb concentration and define
alkalosis and acidosis.
a. Plasma bicarb: 24 mmol/L (normal range 23-28mmol/L)
b. Arterial PCO2: 40 mmHg (normal range 35-45 mmHg)
c. pH: 7.40 (normal range 7.35-7.45)
d. Acidosis
i. Respiratory: ↑arterial PCO2 Æ↓ arterial pH
ii. Metabolic: Arterial PCO2 remains constant but ↓ arterial pH
e. Alkalosis
i. Respiratory: ↓arterial PCO2 Æ↑ arterial pH
ii. Metabolic: Arterial PCO2 remains constant but ↑ arterial pH
5. List the potential causes of respiratory acidosis and alkalosis and metabolic acidosis
and alkalosis.
a. Respiratory acidosis
i. Depression of respiration
ii. Neuromuscular disorders
iii. Chest wall restriction
iv. Lung restriction
v. Pulmonary parenchymal diseases
vi. Airway obstruction
b. Respiratory alkalosis
i. CNS problems (ex: anxiety, tumors)
ii. Drugs or hormones
iii. Bacteremias
iv. Pulmonary disease
v. Overventilation with mechanical ventilator
vi. Hypoxia- high altitude
c. Metabolic acidosis
i. Ingested drugs or toxins
ii. Loss of bicarb (ex: diarrhea, pancreatic fistulas, renal dysfunction)
iii. Lactic acidosis
iv. Ketoacidosis (ex: diabetes)
v. Inability to excrete hydrogen ions
Brown, Eriksen, Jones, Heffernan, Kanjanavaikoon, Leo, Mishkin, Stern, Verlander &Wasserman
Ed. Mishkin 2006
Page 102 of 168
d. Metabolic alkalosis
i. Loss of hydrogen ions (ex: vomiting, gastric fistulas)
ii. Ingestion or administration of excess bicarb or base (ex: antacids)
6. Discuss the respiratory and renal mechanisms that help to compensate for acidosis
and alkalosis.
a. Respiratory compensation
i. Metabolic acidosis
1. ↑ [H+]Æ activate chemoreceptorsÆ ↑ alveolar ventilationÆ ↓
arterial PCO2
ii. Metabolic alkalosis
1. ↓ alveolar ventilationÆ ↑ arterial PCO2
b. Renal compensation
i. Respiratory acidosis and Metabolic acidosis of non-renal origin
1. ↑ secretion H+ ions into the tubular fluid
ii. Respiratory alkalosis and Metabolic alkalosis or non-renal origin
1. ↓secretion H+ ions and ↓ bicarb reabsorption
iii. Renal compensation operates more slowly than respiratory. Respiratory is
instantaneous and renal can take 3 to 6 days.
7. Evaluate blood gas data to determine a subject’s acid-base status.
a. The best way to do this is by doing the examples in the book. Here are some useful
terms defined:
b. Base excess (deficit)- # mEq acid/base needed to titrate 1L of blood to pH 7.4 at
37ºC if Pa CO2 were held constant at 40 mmHg
i. Base excess ranges from -2 to +2
c. Anion gap= [Na+]-([Cl-] + [HCO3-])
i. Normal range is 12±4 mEq/L
ii. Can help determine the cause of metabolic acidosisÆ if anion gap is great,
acidosis caused by lactic acidosis or ketoacidosis.

The Respiratory System Under Stress:


Exercise; Hypobaric and Hyperbaric Environments
3/24/06 Levitzky
1. Identify the physiological stresses involved in exercise.
a. Respiratory stress
i. ↑ oxygen consumption
ii. ↑ carbon dioxide production
b. ↑ lactic acid production
c. ↑ metabolic needs of muscles and other peripheral tissues
2. Predict the responses of the respiratory system to acute exercise.
a. The data below reflect experiments done on people exercising in the verticle positon
only. Experiments evaluating horizontal exercise can be performed for 1 point for
foundations in medicine.
b. ↑VT (↑ RV, FRC, ↓IRV, TLC, & ERV and Vc remain constant)Æ ↑ work to
overcome elastic recoil of lungs and chest wall (which are greater at higher lung
volumes)
c. ↑ airway resistance because greater rates of airflow
d. ↑ venous returnÆ ↑ central blood volumeÆ slight ↓ TLC
e. ↑ breathing frequency
f. slight ↑ anatomic dead space due to airway distention; ↓ alveolar dead spaceÆ
physiological dead space remains constant
i. Since VT increases, ratio of physiological dead space to tidal volume
(VD/VT) decreases
Brown, Eriksen, Jones, Heffernan, Kanjanavaikoon, Leo, Mishkin, Stern, Verlander &Wasserman
Ed. Mishkin 2006
Page 103 of 168
g. ↓ PVR due to recruitment and distention (overcoming effects of extravascular
compression from ↑VT)
h. V/Q relationship improved (V/Q ratio increases, representing an optimal
relationship) throughout anatomical regions of the lung (now the entire lung is
getting matched, i.e., ratio of 1, or better) because ventilation increases more than
perfusion
i. V/Q will be in the range of 2.0 to 4.0 for the entire lung
i. ↑ diffusing capacity of lung
i. ↑ pulmonary blood flow
ii. recruitment of capillariesÆ ↑ SA for gas exchange
iii. ↑↑↑ blood flow velocity Æ ↑ possibility of diffusion limited gas transfer
iv. ↑PO2 and ↓ PCO2 Æ greater partial pressure gradients for gas diffusion
j. ↑ oxygen unloading
i. oxyhemoglobin dissociation curve shifted to right because ↑ PCO2, H+, and
temperature
ii. ↓ PO2 in exercising muscles
k. if exercise is severe enough to cause metabolic acidosis (from lactic acidosis)Æ
stimulate chemoreceptors for ventilatory compensation
3. Describe the effects of long-term exercise programs (training) on the respiratory
system.
a. Max and resting ventilation remain the same
b. ↓ Ventilation at submaximal loads
c. ↑ Strength and endurance of respiratory muscles
d. ↑ Pulmonary diffusing capacity b/c of ↑ blood volume and CO
4. Identify the physiological stresses involved in the ascent to altitude.
a. ↓ Total barometric pressure, but not a constant decrease with ascent
i. Decreases pressure gradient
b. Reduced oxygen available
5. Predict the initial responses of the respiratory system to high altitude.
a. ↓ alveolar and arterial PO2 Æ stimulate arterial chemoreceptorsÆ ↑ alveolar
ventilationÆ ↓alveolar and arterial PCO2 Æ respiratory alkalosisÆ “diffusion” of
CO2 out of CSFÆ ↑pH of CSF
b. ↑ rate and depth of breathingÆ ↑ work of breathing
i. ↑ breathing rateÆ hypocapnia and respiratory alkalosis
ii. Oxygen is still too low because there isn’t much available, but carbon
dioxide is also too low, leading to alkalosis even in hypoxia
c. greater transpulmonary pressure needed to generate greater VT ; need to overcome
vascular engorgement and ↑ interstitial fluid volume of lungÆ ↓ VC
d. ↑ ventilatory rateÆ ↑ active expirationÆ dynamic compression of airwaysÆ ↑
resistance work of breathing
e. ↑ airflow rate b/c of ↓ gas density
f. ↓(VD/VT) with greater VT
g. more uniform regional distribution of alveolar ventilation
i. previously collapsed alveoli will be ventilated
h. ↑ arterial chemoreceptor stimulation and lung inflationÆ symp stimulationÆ ↑ CO,
HR, and systemic BP
i. hypoxic pulmonary vasoconstriction + ↑CO + ↑ symp stimulation of large
pulmonary vesselsÆ ↑MPAP, recruiting capillaries and abolishing Zone 1
j. PO2 decreases more in alveoli than in mixed venous blood Æ ↓ partial pressure
gradient for oxygen; partially offset by ↑ CO and pulmonary artery pressure
Brown, Eriksen, Jones, Heffernan, Kanjanavaikoon, Leo, Mishkin, Stern, Verlander &Wasserman
Ed. Mishkin 2006
Page 104 of 168
k. ↑ Hb concentration due to fluid shifting to extravascular space, not increased
erythrocyte production
l. two possibilities of effect of cerebral circulation
i. first thought altitude caused cerebral hypoperfusion: hypocapniaÆcerebral
vasoconstrictionÆ↓ blood flow (and a lower oxygen content) and alkalosis
of CSF
ii. now think cerebral hyperperfusion and edema: hypoxiaÆ cerebral
vasodilation Æ ↑hydrostatic pressure on cerebral capillariesÆ edemaÆ ↑
ICPÆ distort intracranial structures
6. Describe the acclimatization of the cardiovascular and respiratory systems to long-
term exposure to altitude.
a. Renal compensation- within 1 day
i. ↑ base excretion; H+ conserved
b. Erythropoiesis- within 3 to 5 days
i. ↑ blood viscosity and ventricular work
ii. ↑ 2, 3 BPG to release oxygen to tissues
c. hypoxic stimulation of arterial chemoreceptors persists
d. Ventilatory response curve to CO2 shifts to the left
i. For any given alveolar or arterial PCO2, the ventilatory response is greater
after several days at high altitude
ii. Occurs simultaneously with CSF return to normal pH and relief of cerebral
edema and ↑ICP
1. due to ↑ reabsorption of CSF, autoregulation of cerebral blood
flow, sympathetic-mediated vasoconstriction
e. CO, HR and BP return to normal within a few days
i. ↓ symp activity or change in symp receptors
ii. HPV and pulmonary hypertension persistÆ RV hypertrophy Æ RV failure
secondary to pulmonary hypertension (known as cor pulmonale)
See Table 11-2 for chart describing physiological responses and timeline after ascent to high altitude.

GI Epithelial Barrier Function:


Esophageal, Gastric, and Duodenal Mucosa
Dr. Orlando
4/3/2006

I. Pre-epithelial, epithelial, and post-epithelial defense against acid injury for the
esophageal, gastric and duodenal mucosae
A. Pre-epithelial defenses - on the luminal side of the epithelium.
1. Mucus
1. Thick, viscoelastic layer that provides lubrication against
mechanical injury
2. Secreted via exocytosis from gastric glands in isthmic and foveolar
regions
2. Unstirred water layer
1. There is a layer of water that is stagnant near the epithelium
2. Provides an additional barrier of defense and a place for the
bicarbonate to reside (for maximum effectiveness)
3. Bicarbonate buffer
1. ⇑ [HCO3-] in unstirred water layer; buffers against H+
2. Carbonic anhydrase Æ H+ and HCO3-
a. H is the stomach acids
Brown, Eriksen, Jones, Heffernan, Kanjanavaikoon, Leo, Mishkin, Stern, Verlander &Wasserman
Ed. Mishkin 2006
Page 105 of 168
b. HCO3 buffers
3. Alkaline tide – when parietal cell takes carbonic acid and it
dissociates into H+, and HCO3 Æ H+ is secreted into lumen
HCO3 exits basolateral membrane via Cl/HCO3 exchanger Æ
enters blood ÆHCO3 taken to buffer downstream surface cells
a. Only in stomach
B. Epithelial defense – components of epithelium itself
1. Structural
1. Phospholipid bilayer
a. Reflects ions with hydrophobic core
b. Reflects larger particles except at channels/pores
c. Selective entrance into cell
2. Apical junctional complex (AJC) = tight junctional complex
a. Zonula occludens = tight junction Æ circumferential
component that restricts paracellular diffusion and
maintains cell surface domains
b. Zonula adherens = adhesion belt Æ circumferential,
transmembrane linking proteins Æ cell-cell adhesion
c. Desmosomes = macula adherens Æ spot-welds Æ
reinforced cell-cell adhesion
2. Functional
1. Cellular bicarb – buffers acid entering epithelial cells Æ when
buffering capacity is exceeded Æ cell becomes acidic and pumps
protons out of basolateral membrane
C. Post-Epithelial defense – intercellular space’s defense
1. Bicarbonate in blood
1. Neutralizes acid that leaks through
2. H+ can leak b/t cells Æ tricks body to thinking blood is acidicÆ
wants to equalize pH Æ neutralized by cellular HCO3- and
exports H+ via H/Na exchanger
2. Esophageal intercellular glycoproteins
1. “peanut-butter” material that fills intercellular space Æ ⇑ stickiness
to ⇓ H+ that can get through
II. Discuss the major differences between the above defenses for the esophagus,
stomach and duodenum
A. Esophagus
1. No mucus secretion here
2. Lined with stratified squamous epithelium
1. Relied on heavily against mechanical abrasions
3. Has the intercellular glycoproteins (peanut butter) Æ allows for a better
post-epithelial defense b/c of limitations of stratified squamous
epithelium
B. Stomach
1. Lined with simple cuboidal epithelium
2. Has no intercellular glycoproteins Æ has better tight junctions
3. Has mucus and unstirred water with bicarbonate
4. Alkaline tide
C. Duodenum
1. Mucus
2. Simple cuboidal epithelium
3. No intercellular glycoproteins
Brown, Eriksen, Jones, Heffernan, Kanjanavaikoon, Leo, Mishkin, Stern, Verlander &Wasserman
Ed. Mishkin 2006
Page 106 of 168
4. No alkaline tide
5. Dilutes ⇑ [H+] with water
6. Lots of reflexes to slow stomach if too acidic (see later lectures)
III. Describe the two mechanisms by which the esophagus, stomach, and duodenum
repair their epithelial lining following acid injury
A. Regeneration/Replication
1. Synthesis of new cells – takes time
B. Restitution
1. When cells migrate to fill a hole – faster but only works superficially
2. 30-60 minutes
C. Mechanism: Ulcer Æ from NSAIDs/ASA/H. Pylori Æ no Δ in amount of H+,
the Δ is in the defense mechanism Æ in stomach and duodenum Æ irritant Æ
prostaglandin/COX release Æ ⇑ mucus secretion, HCO3 secretion, and ⇑ blood
flow.

GI Motility I: Esophagus and Stomach


Regulation of Swallowing and Gastric Emptying
Orlando
April 4, 2006

1. List the neuroanatomic components involved in the act of swallowing


a. Upper 1/3 of esophagus is striated muscle and lower 2/3 is smooth muscle
b. Musculature of esophagus is divided into inner circular and outer longitudinal layers
c. Each end of the esophagus is closed off by sphincters
i. UES, upper esophageal sphincter, striated Æ prevents inspired air from
entering esophagus (anatomically the cricopharyngeus muscle)
ii. LES, lower esophageal sphincter, smooth Æ prevents reflux of gastric
contents into the esophagus
d. Motor Innervation of esophagus Æ Vagus nerve innervates the entire esophagus
e. Sensory innervation of esophagus Æ submucosal (Meissner’s) plexus located
between the muscularis mucosae and circular muscle layers
2. Discuss the oral/pharyngeal and esophageal phases of swallowing
a. Oral (voluntary) phase Æ the tongue forms a bolus Æ pushes up and back against
hard palate Æ bolus forced into oropharynx
b. Pharyngeal phase (involuntary) Æ coordinated by the swallowing center in the
medulla and lower pons (mediated by ACh via CN V, IX, and X)
i. initiated by oropharyngeal sensory fibers that detect food
ii. Nasopharynx is closed by the soft palate, preventing regurgitation of food
into the nasal cavities
iii. Palatopharyngeal folds are pulled medially Æ forms a passageway
iv. Glottis and vocal cords are closed, epiglottis swings down over the larynx
Æ food goes toward the esophagus and away from the airways
v. The bolus of food is pushed into the esophagus by the peristaltic
contractions of the pharynx and the opening of the UES
vi. Respiration is inhibited for the duration of swallowing (1-2 seconds)
c. Esophageal (involuntary) phase Æ food is propelled into the stomach, strength of
peristaltic contractions is proportional to the size of the bolus
i. UES contracts to prevent regurgitation, LES relaxes to facilitate passage
ii. Peristaltic wave travels slowly (3-4 cm/sec) taking about 8 seconds to push
food from mouth to stomach
Brown, Eriksen, Jones, Heffernan, Kanjanavaikoon, Leo, Mishkin, Stern, Verlander &Wasserman
Ed. Mishkin 2006
Page 107 of 168
iii. As food enters stomach, LES contracts to prevent regurgitation of food
into esophagus
3. Define the differences between vagal innervation of esophageal skeletal and smooth
muscle
a. Cholinergic fibers that innervate the striated muscle (Upper 1/3 and UES) arise
from the Nucleus Ambiguus and directly synapse on muscle motor end plates
b. Pre-ganglionic cholinergic fibers that innervate the smooth muscle (Lower 2/3 and
LES) arise from the Dorsal Motor Nucleus of X and synapse on neurons in the
myenteric (Auerbach’s) plexus (located between circular and longitudinal muscle)
c. Post-ganglionic cholinergic fibers that innervate smooth muscle synapse on motor
end plates for contraction of longitudinal and circular smooth muscle
d. Post-ganglionic non-adrenergic, non-cholinergic (NANC), possibly Vasoactive
intestinal peptide (VIP), are inhibitory fibers that cause LES relaxation Æ NO is the
final mediator of LES relaxation
4. Discuss the physiologic role of the UES and LES and esophageal peristalsis in
digestion and (whole organism) protection
a. UES Æ striated; contracts to prevent respired air from entering esophagus and
relaxes to allow ingested material to enter esophagus
b. LES Æ smooth; contracts to prevent gastric regurgitation into esophagus and
relaxes to allow ingested material into the stomach
c. Esophageal peristalsis (primary and secondary) function to keep ingested material
moving toward the stomach
5. Describe the differences between primary and secondary peristalsis in the esophagus
a. Primary esophageal peristalsis is initiated by swallowing
i. Coordinated by the sequential activation of motor units in a craniocaudad
sequence by cholinergic excitatory stimuli received from the Vagus nerve
ii. Vagotomy would prevent the initiation of primary esophageal peristalsis
b. Secondary esophageal peristalsis is initiated by the presence of food within the
esophagus; any material remaining in the esophagus stimulate mechanical or irritant
receptors Æ
i. Coordinated by the intrinsic nervous system of the esophagus (myenteric
plexus) Æ
ii. Stretch Æ Afferents Æ intrinsic nervous system
1. Because intrinsic rather than vagal nerves are involved, vagotomy
would have little or no effect on secondary esophageal peristalsis
6. Define the mechanism responsible for transient LES relaxations (TLESRs) and the
importance of these phenomena in health and disease
a. TLESRs Æ spontaneously occurring reflex characterized by LES relaxation in the
absence of swallowing or overt esophageal distension
b. Mediated by vagal activation of NANC inhibitory stimuli to the LES
c. TLESRs are associated with almost all reflux events in healthy subjects and a
primary cause of reflux events in those with reflux esophagitis
i. “Belch” reflex Æ air is swallowed during eating and produces fundic
distension which results in coordinated relaxation of both UES and LES
1. Gaseous distension of fundus produces a TLESR Æ air enters and
distends long segment of esophagus
2. Distension of esophagus induces a reflex relaxation of the UES
7. List the anatomical compartments of the stomach and explain how each contributes
to the processing of a meal
a. The 3 functional parts of the stomach are the fundus, corpus (body), antrum
Brown, Eriksen, Jones, Heffernan, Kanjanavaikoon, Leo, Mishkin, Stern, Verlander &Wasserman
Ed. Mishkin 2006
Page 108 of 168
b. The 3 major functions of gastric motility are storage, mixing/grinding/digestion,
and emptying
c. Storage Æ when food enters the stomach, the orad region, primarily the fundus,
enlarges to accommodate food
d. Mixing/Grinding/Digestion Æ The presence of food in caudad stomach, primarily
the corpus and antrum, increases the contractile activity of the stomach
i. The enhanced contractile activity (a combination of peristalsis and
retropulsion) mixes the food with stomach acid and enzymes, breaking it
into smaller and smaller pieces
ii. Mushed up, food is “Chyme”
e. Emptying Æ When the chyme is broken down into small enough particles, it is
propelled through the pyloric sphincter into the intestine
8. Define the terms basic electrical rhythm (BER), cells of Cajal, and action potentials.
Explain the role of each in producing peristaltic gastric contractions
a. Peristaltic contraction are initiated near the fundal-corpus border and proceed
caudally, producing a peristaltic wave that propels food toward the pylorus
i. At pylorus, a mass contraction of the terminal antrum pushes the food back
toward the corpus through a narrow antral ring Æ retropulsion
ii. <2mm pieces can get through
b. Peristaltic contractions produced by periodic change in membrane potential, called
slow waves, or the basic electrical rhythm Æ these waves are responsible for the
rhythm and force of gastric contractions
c. Gastric slow waves are initiated by pacemaker cells within the wall of the stomach,
the cells of Cajal, which have automaticity and depolarize spontaneously
d. Slow waves consist of an upstroke and plateau phase and occur at a rate of
approximately 3-4 waves/min
e. Velocity of waves is 1cm/sec when they sweep over the corpus and increases to 3-
4cm/sec in the antrum
f. Force of peristalsis Æ force of contractions regulated by gastrin and ACh
i. ACh released by vagus nerve in response to gastric distension
ii. Gastrin released from G cells in antrum in response to rising antral pH and
presence of peptides from protein digestion
iii. These hormones increase the size of the slow wave plateau potential, which
increases the amount of Ca2+ entering the cell from the extracellular fluid
1. Increased contractile activity due to increased amplitude and
duration of the plateau phase of the slow wave
9. Discuss the mechanism responsible for being able to eat a complete meal before
feeling full
a. As food enters stomach, contractile activity of the fundus is inhibited, enabling the
stomach to store 1 to 2 L of food
b. Accommodation is initiated in response to a bolus of food entering stomach.
Stretch receptors in orad stomach detect presence of food and initiate a vagovagal
reflex producing receptive relaxation
c. Receptive relaxation is the process by which the fundus accommodates the meal
without increasing gastric pressure
d. The inhibitory neurotransmitter responsible for receptive relaxation is either VIP or
NO
e. Vagotomy prevents or greatly diminishes receptive relaxation because vagal
reflexes produce both processes

GI Motility II: Small and Large Bowel


April 5, Orlando
Brown, Eriksen, Jones, Heffernan, Kanjanavaikoon, Leo, Mishkin, Stern, Verlander &Wasserman
Ed. Mishkin 2006
Page 109 of 168
Food Processing, Bowel Sterility and Storage, and the Act of Defecation
1. 3 ways the duodenum regulates gastric emptying
Chyme is sampled by the dudodenum and looks at acidity, tonicity and fattiness, which
determines what is released.
a. acidity Æ secretin release
i. released from S cells of the duodenum
ii. reduces gastric contractions
iii. stimulates bicarb secretion
1. neutralizes acid
b. fat Æ cholecystokinin release
i. reduces gastric contractions
1. reduces peristalsis Æ less fat coming into duodenum
ii. stimulates bile secretion
1. fat absorption
c. hypertonicity Æ vagal innervation
i. reflex Æ reduces gastric contractions
ii. stimulates salt and water secretion
1. dilution
2. 2 important functions of migrating motor complex:
a. The Migrating Motor Complex or the MMC has 3 phases:
i. phase I – intestine is quiescent
ii. phase II- sporadic contractions
iii. phase III – regular peristaltic contractions originating in gastric antrum
b. MMC takes 2 hours to cycle, and ceases if oral ingestion occurs
i. the peristaltic wave occurs every 60-90 minutes during the interdigestive
period, (aka fasting state)
ii. One bolus can take 3-4 hours to travel the entire small intestine
c. 2 Functions:
i. Removes undigested material from the stomach
1. including items larger than 2 mm which therefore could not be part
of normal digestion, such as condoms with heroin
ii. Sweeps residual chyme from the small intestine into the cecum – keeps
bacterial colonization in the small intestine to a minimum
1. In scleroderma, for example, MMC is absent and bacteria overgrow
the small intestine, interfering with digestion and absorption of
nutrients
a. treat with antibiotics, somewhat effective
3. 3 major differences between fed and fasting states
Fed State Fasting State
= at least 2 hours since last meal
Duodenal action MMC: from the antrum of the stomach to the
end of the ileum. Stops immediately when oral
ingestion begins. More info above in 2C.
Gastroileal reflex / Gastrocolic reflex: MMC in the stomach is initiated by vagal
The stomach sends neurohumoral signals to the impulses that release motilin
ileum and colon to create a mass movement
because the stomach responds to its being filled,
so there needs to be room made.

The gastroileal reflex is mediated by extrinsic


ANS and maybe gastrin. The gastrocolic reflex is
Brown, Eriksen, Jones, Heffernan, Kanjanavaikoon, Leo, Mishkin, Stern, Verlander &Wasserman
Ed. Mishkin 2006
Page 110 of 168
mediated by the parasympathetic system, CCK
and gastrin.
Secretin Æ stimulating bicarb secretion from Motilin, secreted by enterochromaffin cells of
pancreas for acid neutralization the small intestine, is important in initiating
CCK Æ stimulating bile secretion for fat MMC in the duodenum and small intestine,
absorption which does not depend on extrinsic nerves
Vagus Æ stimulates salt and water secretion for
dilution of luminal content

4. Difference between small intestinal segmentation and peristaltic waves


a. segmentation waves
i. mix intestinal contents
ii. break food into smaller pieces that get pushed away from the point of
contraction (one bit oral, one bit caudal)
iii. no net movement of chyme
b. peristaltic waves
i. propel food caudally
ii. Smooth, progressive, coordinated
iii. still in short segments
iv. basic electrical rhythm
1. 12/minute in duodenum
2. 10/minute in jejunum
3. 8/minute in ileum
5. Anatomical parts of the colon and their functions in processing luminal contents
a. Cecum
i. ileocecal valve – smooth muscle
ii. remove ions in water
b. Ascending colon
i. remove ions in water
c. Transverse colon
i. storage
d. sigmoid colon
i. storage
e. rectum
i. waste elimination
ii. internal anal sphincter – smooth muscle
iii. External anal sphincter, puborectalis muscle – skeletal muscle
6. “Call to stool” after eating
a. Gastrocolic reflex: a meal enters your stomach Æ contents of colon move into
rectum Æ
b. Rectum fills with material Æ rectosphincteric reflex: internal anal sphincter relaxes
c. at 25% capacity, there is the urge to defecate, which is prevented by voluntary
contraction of the external anal sphincter
7. Role of haustrations and mass movements in colon function
a. haustrations –remember haustra are the compartments created by the contraction of
circular muscle – bring material into contact with the epithelium
i. this is the same thing as segmentation waves
b. mass movements – occur over long segments of colon – are peristaltic and occur 1-
3 times a day
i. move luminal content from right to left colon Æ increasingly dehydrated
ii. over 2-3 days material reaches rectum
Brown, Eriksen, Jones, Heffernan, Kanjanavaikoon, Leo, Mishkin, Stern, Verlander &Wasserman
Ed. Mishkin 2006
Page 111 of 168
8. Defecation reflex and the role of sphincters and puborectalis muscle in maintaining
continence
a. mass movement moves solid material into rectum
b. rectum is distended
c. reflex relaxation of internal anal sphincter
i. NO
ii. vasoactive intestinal peptide (VIP)
d. luminal content moves to anal canal
e. sensory receptors Æ matter state of waste is determined
i. anal canal samples
f. External anal sphincter – mediated by acetylcholine
i. relax to allow passage of waste (cholinergic output inhibited)
1. levator ani, puborectalis (reins in anal canal), and external anal
sphincter relax
2. pelvic floor relaxation, along with squatting position, straightens
the anorectal angle facilitating evacuation
ii. contract to prohibit passage of waste (cholinergic output stimulated)
1. rectum undergoes receptive relaxation to accommodate
2. internal anal sphincter relaxes
3. external anal sphincter and levator ani are under voluntary control
a. contraction prevents evacuation
4. rectum exhibits receptive relaxation – just like the fundus of the
stomach – and then the internal sphincter stops sensing stretch, so
it contracts, returning matter upwards
5. rectosigmoid stores waste
6. other mammals can also put off defecation

GI secretion I: Mouth and salivary glands


The role of secretions in digestion and protection
4/6/06 Dr. Orlando, Handout by Dr. Edd Rabon
1. Discuss the structure and function of the salivary glands
a. 3 major pairs of salivary glands
i. Submandibular- mixed seromucinous-secreting
ii. Sublingual- mixed seromucinous-secreting
iii. Parotid- primarily serous-secreting
1. all water all the time
b. Glands composed of units called salivons
i. Each salivon composed of an acinus, an intercalated duct, and a striated
collecting duct
ii. Ducts are lined by simple columnar epithelium
c. Acinus surrounded by myoepithelial cells on serosal side
i. Can contract to enhance rate of salivary secretion
d. Salivon has both sympathetic and parasympathetic efferent innervation
i. Symp- T1-T2 of spinal cord
ii. PSymp- superior and inferior salivary nuclei, transmitted by CN VII and IX
2. Describe how saliva is produced and its flow regulated
a. Saliva production begins in acinus as ultrafiltrate of blood
i. Serous salivary acinar cells transport ultrafiltrate of blood into lumen of
acinus (see Fig 32-6 in the physio handout for images)
1. Basolateral membrane:
a. Na/K ATPase
b. Na/K/2Cl cotransporter
Brown, Eriksen, Jones, Heffernan, Kanjanavaikoon, Leo, Mishkin, Stern, Verlander &Wasserman
Ed. Mishkin 2006
Page 112 of 168
2. Apical membrane:
a. K channel
b. Cl, HCO3- “co-channel”
c. Na enters lumen partly via tight junctions between
epithelial cells
b. Ultrafiltrate enters ducts where it is modified by absorption and secretion
i. Striated and excretory secrete K+ and HCO3- and reabsorb Na+ and Cl- via
a series of ion exchangers and channels, driven by a basolateral Na/K
pump (see Fig 32-5 in the physio handout for images)
1. Apical membrane:
a. Na channel into the cell
b. Cl/HCO3- exchanger, pumps HCO3- into duct lumen
c. Na/H exchanger, pumps Na into cell
d. H/K exchanger, pumps K into duct lumen
2. Basolateral membrane:
a. Na/K ATPase, pumps Na out of cell, K in
b. K channel, releases K into blood
c. Cl channel, releases Cl into blood
d. H/Na exchanger, pumps H into blood, Na into cell
c. Final product is hypoosmolar and rich in K+ and HCO3- in comparison to blood
i. Basal rate of saliva flow 0.5 ml/min, can be stimulated up to 4 ml/min
ii. During a meal salivary flow rates increase, so saliva has less time to be
modified by absorption and secretionÆ saliva is less hypoosmolar and
lower in K+ and HCO3-
d. Completely under neural control (not hormonal like other GI systems)
e. Simultaneously innervated by sympathetic and parasympathetic
i. It’s the only thing we’ve seen that uses both arms of the autonomic nervous
system, and it makes sense – you always need to lube when things will be in
your mouth, whether it’s food (parasympathetic) or people (sympathetic)
ii. Norepi (acting on α and β adrenergic receptors), Ach, and substance PÆ
↑Ca2+ intracellularyÆ ↑ amylase secretion and fluid secretion
iii. Norepi also ↑ cAMP levelsÆ ↑ amylase secretion
f. Effects of sympathetic and parasympathetic stimulation on salivary secretion
i. Sympathetic
1. scant salivary output
2. transient duration of flow
3. protein-rich
4. cAMP is cytosolic messenger
5. if denervated, glands will not atrophy
6. Note: cotton mouth occurs during sympathetic stimulation because
of #s 1, 2 and 3, especially 3
ii. Parasympathetic
1. copious salivary output
2. sustained duration of flow
3. protein-poor, watery (no cotton-mouth)
4. Ca2+ is cytosolic messenger
5. if denervated, glands will atrophy, so psymp involved in gland
growth, not just salivation
3. List 5 major components in saliva and identify their contribution to digestive or
protective function of the individual
a. Water
i. Digestive function
Brown, Eriksen, Jones, Heffernan, Kanjanavaikoon, Leo, Mishkin, Stern, Verlander &Wasserman
Ed. Mishkin 2006
Page 113 of 168
1. lubrication of bolus for mastication and swallowing
ii. Protective function
1. dilute potentially noxious components of food (makes bolus less
osmotic Æ protective because it brings hyperosmotic foods closer
to isoosmotic)
2. bolus temperature control
3. oral hygiene
b. Mucus
i. Digestive function
1. lubrication of bolus for mastication and swallowing
ii. Protective function
1. lubrication of bolus against mechanical trauma
c. Alpha amylase
i. Digestive function
1. starch digestion
ii. Probably vistigial; not necessary
d. Lingual lipase
i. Digestive function
1. fat digestion
ii. Probably vistigial; not necessary
e. HCO3-
i. Protective function
1. neutralize acid in food
2. neutralize HCl reflux from stomach
3. antibacterial by neutralization
f. Other components
i. Lactoferrin- antibacterial by binding iron
ii. Muramidase- antibacterial by hydrolyzing cell walls
iii. Epidermal Growth Factor (EGF)- cell growth and repair
iv. R protein – binds B12 temporarily before intrinsic factor binds

Gastric Secretions II – Stomach


Dr. Lenard Lichtenberger
4/7/2006
I. Endocrine Regulation of Gastrointestinal Function
A. Mostly in the antrum of the stomach
B. Gastrin – made by G cells in antrum; granules are dumped into blood
1. Released with presence of proteins in stomach/mouth (sham)
2. Stimulates the release of H+ into lumen (80% of acid output)
3. Stimulates GI mucosal proliferation (GI trophic hormone) in the body of
the stomach
4. Stimulates pancreatic enzyme secretion (CCK)
5. Stimulates contractile activity of GI smooth mm – promotes gastric
motility
C. Secretin – released by duodenal cells when there is acid in the duodenum; secretin
inhibits gastrin-stimulated parietal cell activity
D. CCK – released by I cells of duodenum in presence of food (protein and fat)
1. initiates gallbladder contraction
E. GIP – released by K cells in the small bowel in response to fat , inhibits parietal cell
function (HCl secretion)
F. Neurotensin – released by terminal ileum, in response to fat inhibits gastrin food-
stimulated acid secretion
Brown, Eriksen, Jones, Heffernan, Kanjanavaikoon, Leo, Mishkin, Stern, Verlander &Wasserman
Ed. Mishkin 2006
Page 114 of 168
G. Peptide YY (PYY) – released from intestine from fatty meal, inhibits feeding and
vagal stimulated gastric acid secretion
II. List the overall physiological functions of the GI system
A. Food
1. Digestion
2. lubrication
3. absorption of food
B. Acid barrier against bacterial infection
III. List the major functions of the GI system that are regulated by GI hormones –
see LO I
IV. Describe and contrast the three forms of regulation; provide examples of each
A. Endocrine – see LO I/III
B. Paracrine –
1. Histamine
1. ECL cells store histamine Æ paracrine regulator of parietal cell
2. ECL cells stimulated by ACh and gastrin to release histamine
3. Histamine binds and activates parietal cells via H2 receptors
4. cAMP is second messenger Æ increased H+ secretion
2. Somatostatin – released by D cells; release is initiated by stomach acid,
inhibits parietal cell directly and indirectly (opposes gastrin cells)
C. Neurocrine – ACh
1. Binds M3 receptor on parietal cell to release intracellular Ca2+
2. Indirectly stimulates acid output Æ activates G cells Æ release of Gastrin
releasing peptide (GRP) and inhibiton of somatostatin release
V. Gastric secretion
A. Histamine plays a major role Æ opens Cl- channels
B. Proton pumps as α (catalytic) and β (localization/membrane) subunits
1. Can be blocked with drugs
C. Parietal cells
1. Mitochrondria Æ pump H+ against gradient
2. Canaliculi Æ fuse w/ tubular vesicles Æ ⇑ surface area and exposes
proton pumps and K+ channels
D. Gastric secretion is isotonic to the blood at all secretory rates
VI. List the major physiological functions of the stomach – See LO II
VII. Describe/diagram an acid secreting (parietal) cell in the resting and stimulated
state
Refer to figures 3, 4 and 5 in the lecture notes.
A. Stimulated state: Morphological changes occur that allow parietal cells to secrete
HCl.
1. secretory canaliculi are formed from tubulovesicles (which have
membranous H+/K+ ATPase exchangers) fusing with parietal cell
membranes. H+ is secreted via exchanger pump, Cl- is secreted via
membranous Cl- channels.
2. Gastrin and ACh act via calcium messenger to cause morphological
change, while histamine acts via cAMP messenger
3. Resting state: Tubulovesicles are sequestered into the cell, away from the
cell membrane, so H/K exchanger can no longer actively secrete H+ into
lumen, and HCl secretion ceases.
1. High H+ (low pH) Æ release of somatostatin & secretin Æ
inhibition of parietal cells (somatostatin directly & indirectly,
Brown, Eriksen, Jones, Heffernan, Kanjanavaikoon, Leo, Mishkin, Stern, Verlander &Wasserman
Ed. Mishkin 2006
Page 115 of 168
secretin indirectly only – indirect mechanisms are via gastrin
inhibition)
VIII. Describe the biochemical steps involved in gastric acid secretion
A. H+ and Bicarb are produced in parietal cells via carbonic anhydrase conversion of
water and carbon dioxide
B. Bicarb and Chloride are exhcanged on basolateral membrane; Chloride into cell,
Bicarb into blood (=alkaline tide)
C. Cytosolic Cl and K diffuse down leak channels in canaliculi to enter the lumen
D. H+ ions are pumped against their concentration gradient into the lumen via
H+/K+ ATPase
IX. Describe the anatomical changes that occur when a parietal cell goes from the
resting to the stimulated state Æ see LO VII
X. Describe how proton pump inhibitors such as omeprazole interact with the H/K
ATPase to inhibit gastric acid secretion
A. Accumulates at pH = 3.0
B. In activated form, makes a disulfide bridge with the parietal cells H/K ATPase Æ
irreversible inhibition b/c can’t pump H+ into lumen any more
C. Takes a few days because it can only inhibit pumps in active cells Æ cumulative
effect to alkalinize stomach
XI. Describe the fluxes of Na, Cl, and H in the parietal cell during acid secretion
A. Na/H exchange (uses Na gradient from Na/K ATPase) pumps H+ out
1. on basolateral membrane\
1. Dr.Lichtenberger’s Notes mention this but do not explain the
significance
B. H/K ATPase pumps H+ to lumen and K+ into the cell (K+ leaks back into the
lumen through leak channel)
1. on apical membrane
C. Cl/HCO3- exchanges Cl- in (down gradient) for HCO3 out (as alkaline tide). Æ Cl is
removed to lumen of stomach
1. Cl/HCO3 exchange is on basolateral membrane
2. Cl channel is on apical side
D. See Figure 9 for the picture in the lecture handout
XII. Describe/diagram the changes in concentrations of Cl, H, K, and Na in gastric
juice during varying rates of gastric secretion
A. Cl
1. Concentration is high regardless of secretion rate (120<)
2. highest at high rates of secretion (<140)
B. H+
1. At low secretion rates – very low levels, close to 0
2. At high secretion rates – high (<120)
C. K+
1. Concentration is low (<20) regardless of secretion rate
2. Highest at high secretion rates
D. Na+
1. Highest at low secretion rates (about 80)
2. Lowest at high secretion rates (<20)
E. Pump concentrations are regulated hormonally Æ graded reactions for varying rates
of acid secretion
F. As more parietal cells are activated, more H/K ATP pumps are added to cell
surfaces Æ increases H+
G. At low secretion rates, gastric juice has high Cl and low H+
H. See figure 10 for the picture in the lecture handout
Brown, Eriksen, Jones, Heffernan, Kanjanavaikoon, Leo, Mishkin, Stern, Verlander &Wasserman
Ed. Mishkin 2006
Page 116 of 168
XIII. Describe/diagram which cells gastrin, histamine, and acetylcholine act on to
induce gastric acid secretion
A. Gastrin Æ acts on parietal cells directly Æ H+ secretion
B. Histamine Æ opens Cl channels of parietal cells Æ K+ travels with Cl to lumen,
then recycles back to cell via H+/K+ ATPase exchanger
C. ACh Æ Acts on both ECL cells to secrete histamine and on parietal cells directly Æ
H+ secretion
XIV. Describe the structure-activity relationships for gastrin (e.g. importance of active
site and amidation of the carboxy terminus).
A. Both terminal ends have an amide group on them Æ important protector against
carboxy-peptide digestion
B. Both big (G34) and little (G17) are biologically active
C. Postulated that longer gastrins (less post-translational modifications) may Æ gastric
tumors
XV. Discuss the mechanisms and stimuli for release of gastrin. See LO I-B
XVI. Other actions of gastrin other than regulation of acid secretion See I-B.
XVII. Name the receptor on which histamine acts to induce acid secretion and
describe the second messenger system involved.
A. Histamine acts on H2 receptor on parietal cells
1. Mediation by adenylate cyclase/cAMP messenger system
2. Inhibition of H2 receptors Æ blocks acid secretion Æ stops ulcers
1. Cimetidine blocks H2 receptors (see below, XVIII)
XVIII. Discuss the rationale for the use of histamine receptor blocking agents in the
treatment of peptic ulcer disease
A. Block receptor Æ ↓ cAMP Æ ↓ acid Æ ↓ mucosal erosion Æ ↓ ulcers
B. Add receptor blocker (cimetidine) to increase pH in lumen
C. Blocking histamine has a triple effect
1. Histamine Receptors blocked
2. ACh unpotentiated
3. Gastrin unpotentiated
XIX. Name the receptor on which acetylcholine acts to induce acid secretion and
describe the second messenger system involved.
A. ACh Æ M3 type mAChR on parietal cells Æ Second messenger system involves ⇑
intracellular Ca2+, IP3 Æ H+ secretion
1. This represents the direct pathway of ACh’s action
XX. Describe how ACh can act indirectly to stimulate acid secretion through
modulating the release of gastrin
A. Vagus Æ ACh Æ Enteric nerve Æ GRP Æ binds and activates parietal cells for
acid secretion
B. Vagus Æ ACh Æ M1 type mAChRs Æ ECL cells Æ histamine Æ activate parietal
cells
XXI. Discuss/diagram the three-receptor model for the interaction of ACh, histamine,
and gastrin in the regulation of acid secretion by the parietal cell
A. See Figure 18 in Lecture Notes
B. Potentiative interaction of the 3 major gastric regulators
1. All 3 occupy different membrane receptors in parietal cell. Because each
agent has a different mechanism of action, their combined effect is
greater than the sum of the individual effects
2. Blocking receptors
1. Atropine blocks ACh from binding M3 mAChRs
2. Cimetidine blocks Histamine from binding H2 receptors
Brown, Eriksen, Jones, Heffernan, Kanjanavaikoon, Leo, Mishkin, Stern, Verlander &Wasserman
Ed. Mishkin 2006
Page 117 of 168
3. Blocking either receptor Æ parietal cell’s response to all activating
agents is decreased Æ a drug that blocks just one of these
receptors is usually effective in treatment of ulcers
4. In addition, cimetidine blocks the potentiative effect itself, making
cimetidine the most effective treatment for ulcers
XXII. Describe the role of enterochromafin-like cells (ECL) in the control of acid
secretion
A. ECL cells store Histamine
B. Gastrin &ACh Æ ECL cells release histamine Æ acts locally on parietal cells
C. Histamine release increases greatly just before acid secretion begins
XXIII. Discuss how luminal acid can inhibit acid secretion
A. Decreased pH Æ ⇓ gastrin secretion Æ⇓ H+ release
B. Decreased pH Æ increased somatostatin secretion Æ inhibits parietal & G cells
XXIV. Describe the three phases of gastric acid secretion and their regulation
A. Cephalic phase
1. Vagal stimulation from mouth
2. Direct stimulation of parietal cells
3. Vagal stimulation of gastrin release via GRP
B. Gastric phase
1. Starts w/ entry of bolus into stomach
2. Products of protein digestion Æ gastrin release,
3. Gastric distention Æ vagovagal reflex Æ stimulates parietal cell Æ ⇑ acid
release
C. Intestinal phase
1. Starts with entry of bolus into duodenum
2. Represents ~10% of acid release
3. Mediators = duodenal gastrin, entero-oxyntin (intestinal hormone) and
circulating amino acids
XXV. Describe the biochemical processes involved in pepsinogen synthesis, storage,
secretion, and activation to pepsin
A. Pepsinogen Æ isozymes 1-5 (pepsinogen I, stored in chief and mucous neck cells of
oxyntic mucosa) and isozymes 6 and 7 (pesinogen II, stored in chief and mucous
neck cells of oxyntic mucous and in mucous cells of antrum and Brunner’s glands)
B. Secreted into lumen and activated by low pH Æ chop off part of peptide to expose
active site Æ pepsin. Can be done spontaneously or by another pepsin molecule Æ
dynamic process over time
XXVI. Discuss the regulation of pepsinogen/pepsin release
A. Vagal stimulation Æ ACh-mediated
B. Acidic stimulation
C. Secretin simulated
XXVII. Discuss the role of intrinsic factor in the absorption of Vitamin B12
A. Intrinsic factor (IF) is released by parietal cells before acid secretion (See figure 28 in
lecture notes)
B. Binds and protects vitamin from proteolysis Æ promotes absorption in ileum
C. Addressed in later LO sets
XXVIII. List the cells of origin and the chemical components of gastric mucosa
A. parietal cells – HCl, intrinsic factor
B. Chief cells – pepsinogen
C. G cells – gastrin
D. Mucous cells
Brown, Eriksen, Jones, Heffernan, Kanjanavaikoon, Leo, Mishkin, Stern, Verlander &Wasserman
Ed. Mishkin 2006
Page 118 of 168
XXIX. Discuss the physiological functions and mechanisms of secretions of gastric
mucus
A. Functions – protective against abrasive and acidic environment Æ ⇓ erosion and ⇑
lubrication
B. Secreted by surface mucous and mucous neck cells
1. Exocytosis – 1 granule at a time
2. Apical expulsion – entire packages at once
3. Mucous cell exfoliation (type of apoptosis) – cell turnover of entire
mucous cell layer occurs every few days
XXX. (Now you know why we used roman numerals for this set). Discuss the
physiological regulation of gastric mucus secretion.
A. Vagal stimulation Æ discharge of soluble mucous from neck cells
B. Mechanical stimulation Æ noxious Æ gelatinous mucous from surface cells
C. Chemical irritant Æ noxious Æ gelatinous mucous from surface cells
D. Gastroprotective prostaglandins Æ made in response to noxious stimuli Æ ⇑
mucous secretion

GI Secretions III – The Pancreas


4/10/06 Dr. Lichtenberger
1. List the physiological functions of the secretions of the exocrine pancreas.
a. Secretion of enzymes to hydrolyze dietary carbohydrates, proteins and lipids (this
function is essential for life).
i. trypsinogen
ii. chymotrypsinogen
iii. pancreatic lipase
iv. amylase
b. Secretion of HCO3- both to buffer gastric acid entering the proximal intestine, and
to maintain a neutral/alkaline pH in the intestinal lumen to assure optimal
pancreatic enzymatic activity.
i. Secretin
1. Secreted by S cells in duodenum in response to H+ in duodenal
lumen.
2. makes pancreatic ductal cells secrete more HCO3-
3. H+ delivery from stomach to duodenum Æ secretin release Æ
pancreas secretes HCO3- Æ bicarb in duodenal lumen neutralizes
the H+.
2. Describe the functional anatomy of the “pancreon”.
a. Anatomy:
i. Acinus – the grape-shaped secretory unit of an acinous gland
1. acinar cell – a secreting cell that lines an acinus
a. produces a small volume of initial pancreatic secretion,
which is mainly Na+ and Cl- in zymogen granules
2. centroacinar cell – a nonsecretory cell of a pancreatic ductule
occupying the lumen of an acinus –aka “Langerhans”
ii. Ductule – a small duct
1. duct cell – modifies the initial pancreatic secretion by secreting
HCO3- and absorbing Cl- via a Cl-/HCO3- exchange
mechanism in the luminal membrane
a. because the pancreatic ducts are permeable to water,
water moves into the lumen to make the pancreatic
secretion isosmotic
Brown, Eriksen, Jones, Heffernan, Kanjanavaikoon, Leo, Mishkin, Stern, Verlander &Wasserman
Ed. Mishkin 2006
Page 119 of 168
b. Electrolyte composition of pancreatic juice:
i. Pancreatic juice is characterized by:
1. High volume
2. virtually the same Na+ and K+ concentrations as plasma
3. Much higher HCO3- concentration than plasma
4. Much lower Cl- concentration than plasma
5. Isotonicity
6. Pancreatic lipase, amylase, and proteases
3. Discuss/diagram the steps in the synthesis and secretion of pancreatic enzymes.
a. Polysomes synthesize the proteins in the acinar cells that will become the
functional enzymes of the pancreas
b. These proteins are transported from the RER to the Golgi to become modified into
zymogens
c. After modification, the zymogens are sent to condensing vacuoles to consolidate the
zymogens (now called a zymogen granule)
d. The zymogen granule is transported to the apical surface to be secreted
e. Once secreted, the zymogens become activated amylase, lipases, and proteases
close to a pH that is neutral or slightly basic
4. Discuss/diagram the steps in the secretion of bicarbonate by ductal and acinar cells.
a. CO2 diffuses from the basal side of the cell
b. Carbonic Anhydrase (CA) takes H2O and CO2 and converts it to H2CO3, which
dissociates to H+ and HCO3-
c. HCO3- is secreted on the luminal side by an HCO3-/Cl- antiport
d. H+ is secreted on the basolateral side by an H+/Na+ antiport
e. Na+/K+ ATPase regenerates Na+ gradient (which we already knew)
5. Describe the relationship between the rate of secretion of the pancreas and the ionic
composition of the secretion. Explain the mechanisms for this relationship.
a. Composition of aqueous component of pancreatic secretion varies with Flow Rate
i. At low flow rates, the pancreas secretes an isotonic fluid composed mainly
of Na+ and Cl-
1. Mainly Cl- in pancreatic fluid because the HCO3-/Cl- antiport on
the apical side of the duct cell is given ample time to exchange
HCO3- for Cl-
ii. At high flow rates, the pancreas secretes as isotonic fluid composed mainly
of Na+ and HCO3-
1. Mainly HCO3- in pancreatic fluid because exchange time is
shortened (refer to Fig. 4 and 5 in handout for a visual)
iii. REMEMBER: regardless of flow rate, pancreatic secretions are isotonic
6. Describe the “cephalic phase” of pancreatic secretion in response to a meal and
discuss the mechanism of regulation of this phase.
a. Cephalic Phase – represents 10-15% of the pancreatic response to feeding
i. Categorized by seeing, smelling, tasting, chewing, swallowing, hypoglycemia
or thinking about food (conditioned reflex) results in the secretion of a
pancreatic juice rich in enzymes.
1. “Sham feeding” can cause up to 50% of the total pancreatic
secretion
ii. Mechanism of regulation:
1. Major regulation – mediated by direct efferent impulses sent by
vagal centers in the brain to the pancreas
2. Minor regulation – indirect effect of parasympathetic stimulation of
gastrin release (CCK-like effect on acinar cells)
Brown, Eriksen, Jones, Heffernan, Kanjanavaikoon, Leo, Mishkin, Stern, Verlander &Wasserman
Ed. Mishkin 2006
Page 120 of 168
7. Describe the “gastric phase” of pancreatic secretion in response to a meal and
discuss the mechanism(s) of regulation of this phase.
a. Gastric phase – begins when food enters the stomach and distends it
i. Represents 10-15% of the total response to feeding
ii. Pancreatic secretion is then stimulated by vago-vagal reflex (process that
uses both afferent and efferent nerve fibers of the vagus nerve)
iii. Gastrin-mediated pancreatic enzyme secretion
8. Describe the “intestinal phase” of pancreatic secretion in response to a meal.
a. Intestinal Phase – most important phase where acidic chyme from the stomach
moves into the small intestine
i. Acid in chyme stimulates release of secretin by S cells in intestinal mucosa
ii. More information in #s 10, 11 & 12
9. What are the relative importances of the cephalic, gastric, and intestinal phases in
the regulation of the pancreatic secretion?
a. Cephalic Phase = 10-15%
b. Gastric Phase = 10-15%
c. Intestinal Phase = 70-80%
10. Describe the mechanisms regulating the release of secretin in response to a meal.
a. Secretin is secreted by S cells of the duodenum in response to H+ in the duodenal
contents (pH < 4.5) and total length of intestine that is exposed to these acidified
contents (total acid)
i. Acts on pancreatic duct cells to increase HCO3- secretion
b. Other stimulants of secretin release
i. Fatty acids, bile salts
11. Discuss the role of secretin in regulating pancreatic bicarbonate secretion.
a. When H+ is delivered from the stomach to the duodenum, secretin is released by S
cells in the intestinal mucosa. As a result, HCO3- is secreted from the pancreas
into the duodenal lumen to neutralize the H+
b. Second messenger for secretin is cAMP
12. Discuss the interaction of secretin and CCK in regulating pancreatic bicarbonate
secretion.
a. CCK, released by I cells in the intestinal mucosa, is stimulated by exposure of the
intestinal mucosa to long-chain fatty acids (lipid digestion products) and free amino
acids
b. CCK potentiates the effect of secretin on duct cells to stimulate HCO3- secretion
i. CCK is very similar in structure to secretin, which is important for the
tertiary structure
13. Explain why secretin is effective only in its intact form.
a. Every one of the 27 amino acids in secretin is necessary for it to function
14. Describe the physiological actions of CCK.
a. CCK is secreted by duodenal I cells in response to small peptides, amino acids, and
fatty acids in the duodenal lumen.
b. CCK increases pancreatic acinar cell secretion of amylase, lipases, and proteases
i. the second messenger for CCK is IP3 and increased intracellular
[Ca2+]. The potentiating effects of CCK on secretin are explained by the
different mechanisms of action for the two GI hormones (i.e. cAMP for
secretin and IP3/Ca2+ for CCK)
c. Potentiation of pancreatic HCO3 secretion
d. Stimulation of gallbladder contraction
e. Relaxation of the Sphincter of Oddi
f. Other actions (physiological significance uncertain)
i. Inhibition of gastric emptying
Brown, Eriksen, Jones, Heffernan, Kanjanavaikoon, Leo, Mishkin, Stern, Verlander &Wasserman
Ed. Mishkin 2006
Page 121 of 168
ii. Induction of satiety
iii. Stimulation of intestinal contractile activity
iv. Stimulation of pancreatic growth
v. Inhibition of gastric acid secretion
vi. Stimulation of insulin secretion (incretin effect)
15. Discuss the structure of CCK in terms of its “active site” and the importance of the
tyrosine sulfation.
a. CCK
i. The structure of the CCK active site is similar to that of gastrin
ii. 5 C-terminal amino acids are the same in CCK and gastrin
iii. the biologic activity of CCK resides in the C-terminal heptapeptide.
Thus, the heptapeptide contains the sequence that is homologous to gastrin
and has gastrin activity as well as CCK activity
b. Tyrosine sulfation
i. A posttranslational modification of tyrosine; sulfation strengthens protein-
protein interactions.
ii. some adhesion molecules, GPCRs, coagulation factors, serpins, extracellular
matrix proteins, and hormones have tyrosine sulfation
iii. sulfation of cholecystokinin (CCK) is required for CCK receptor activation
1. also postulated to be involved in solubility, stabilization, and
functional interaction
2. Tyrosine sulfation is necessary for CCK to be able to bind to its
own receptor; otherwise it will act as gastrin!
16. Discuss the role of cholinergic nerves in the regulation of pancreatic enzyme
secretion.
a. ACh, via vagovagal reflexes, is released in response to H+, small peptides, amino
acids, and fatty acids in the duodenal lumen
b. ACh stimulates enzyme secretion by the acinar cells and, like CCK, potentiates
the effect of secretin on HCO3- and H2O secretion.
17. Describe the interactions of stimulants of pancreatic enzyme secretion at the cellular
and molecular level.
a. ACh, CCK, and gastrin act on pancreatic acinar cells
i. Increase turnover IP3
ii. Release cellular calcium, which
1. depolarizes the cell
2. ↑ cGMP
3. ↑enzyme secretion
b. Secretin, VIP act on pancreatic acinar cell
i. ↑ cellular cAMP, Æ
ii. ↑ enzyme secretion
18. Describe the potentiative interaction of CCK and secretin on pancreatic bicarbonate
secretion.
a. CCK Æ slight ↑ in HCO3- secretion
b. Secretin Æ slightly higher ↑ in HCO3- secretion
c. CCK + Secretin Æ over three times as much HCO3- is released into the pancreatic
lumen
i. So CCK greatly potentiates secretin
19. Explain the potential physiological importance of this potentiation.
a. Potentiation is when the total is greater than the sum of its parts: the interaction of
multiple agents results in a pharmacologic response greater than the sum of the
individual responses to each agent.
Brown, Eriksen, Jones, Heffernan, Kanjanavaikoon, Leo, Mishkin, Stern, Verlander &Wasserman
Ed. Mishkin 2006
Page 122 of 168
b. Potentiation of CCK + Secretin Æ ↑↑ HCO3- Æ better prepare the small intestine
for the chyme
c. This potentiation is explained by different mechanisms of the two GI hormones
(secretin uses cAMP and CCK uses cGMP)
20. Discuss the possible mechanism for the inhibitory effect of intraluminal trypsin and
its possible degradative effect on CCK-Releasing Peptide on pancreatic secretion.
a. Trypsinogen produced by the pancreas is activated to trypsin by a brush border
enzyme, enterokinase
b. Trypsin then converts chymotrypsinogen, proelastase, and procarboxypeptidase A
and B to their active forms (even trypsinogen is converted to more trypsin by
trypsin!)
c. After their digestive work is complete, the pancreatic proteases degrade each other
and are absorbed along with dietary proteins.
d. Intraduodenal trypsin inhibits pancreatic secretion mediated by CCK
21. Discuss the potential role of peptide YY (PYY) and of pancreatic polypeptide (PP) in
inhibiting pancreatic secretion.
a. PP inhibits pancreatic enzyme output & gallbladder contraction
i. PP is responsible for coordinating exocrine and islet enzyme release
b. The distal gut hormone peptide YY (PYY) mediates feedback inhibition of gastric
acid secretion, gastrointestinal motility, and pancreatic enzyme output. PYY is
released in proportion to calorie intake, and it inhibits eating.
i. PYY acts on the pancreas to increase its secretion of digestive juices and on
gallbladder to stimulate the release of bile
c. Sham feeding makes both of these proteins plus NPY increase
i. In comparing sham feeding to actual feeding, PP is released in significant
amounts during actual feeding
22. Explain why insulin release is greater after oral glucose as compared to an
intravenous glucose, and how this fits into the concept of an incretin.
a. Release of Glucose-dependent insulinotropic peptide (GIP)
i. Stimulated by the presence of glucose or fat in the intestinal lumen NOT
by glucose in the blood
ii. May inhibit gastric acid secretion
iii. Causes insulin release
b. Release of insulin
i. Only slightly stimulated by glucose in blood
ii. Heavily stimulated by oral glucose
c. A comparable rise in blood glucose by IV infusion of the sugar has little stimulatory
effect on the blood levels of either insulin or GIP
d. Incretins are hormones that increase insulin release and decrease glucagon release
in response to sugar intake – that act before blood glucose levels elevate. They reduce
gastric emptying, slow nutrient absorption and reduce food intake. If these
hormones generally work before blood glucose levels elevate, it fits together with
their mechanism that oral ingestion initiates a pathway that IV glucose does not.
23. Discuss the possible insulin – stimulatory (incretin) roles of GIP and enteroglucagon
/ glucagon-like peptide (GLP-1).
a. GIP – stimulated by the presence of glucose or fat in the intestinal lumen
i. Was thought to inhibit gastric acid secretion (entero-gastrone effect)
ii. Is now thought to cause insulin release (incretin)
b. GLP-1
i. Stimulates insulin release (incretin)
ii. 50% of the body’s store of glucagon is present in the intestine
iii. Shares many actions with secretin, glucagon and GIP
Brown, Eriksen, Jones, Heffernan, Kanjanavaikoon, Leo, Mishkin, Stern, Verlander &Wasserman
Ed. Mishkin 2006
Page 123 of 168

GI Secretions IV: The Liver and Biliary System


Lichtenberger
April 11, 2006
1. Biliary Secretion
a. Bile is formed by the liver epithelial cells (hepatocytes) and by epithelial lining the
bile ducts, called ductal cells. Between 250-1100 mL of bile are secreted daily
b. Synthesis, 0.2-0.6 g/day
c. Biliary Secretion = 12-36 g/day
d. Fecal Excretion = 0.2-0.6 g/day
e. Urinary excretion = <0.5 mg/day
f. Portal venous return Æ >95% of biliary secretion
2. List the physiological functions of bile
a. Bile is required for the digestion and absorption of lipids in the intestinal lumen
b. Bile serves as a vehicle for the elimination of endogenous (cholesterol, bile
pigments) and exogenous (drugs, metals) substances from the body
3. Describe the functional anatomy of the Biliary tract/gallbladder system
a. Bile is secreted by the hepatocytes into the bile canaliculi
b. Bile flows in the ductules and ducts, lined by biliary epithelium
c. Bile flow through the canaliculi runs counter to the flow of blood through the
hepatic sinusoids assuring efficient extraction of bile acids by the hepatic
parenchymal cells and secretion into the canaliculi
d. Bile flows through canaliculi into the bile ductile Æ bile duct Æ common bile duct
Æ sphincter of Oddi
i. Between meals (interdigestive period) the Sphincter of Oddi is closed so
there is retrograde flow of bile into the cystic duct Æ gallbladder, for
storage
4. List the organic components of bile
a. Bile Acids (BA-), constitute 50% of solids
i. Primary Bile acids
1. Cholic acid (trihydroxycholic acid)
2. Chenodeoxycholine acid (dihydroxychenodeoxycholic acid)
ii. Secondary Bile acids
1. Deoxycholic acid
2. Lithocholic acid (monohydroxy acid)
b. Phospholipids (30-40% of the solids), are normally insoluble in water, so they are
solubilized by the bile salt micelles
i. Lecithin/phosphatidylcholine represent the major components
c. Cholesterol (4%) of the solids), is essentially insoluble in water and thus must be
solubilized by bile salt micelles (addressed later) before it can be secreted in the bile
d. Bile Pigments (2% of solids)
5. Explain the difference between a “primary” and a “secondary” bile acid
a. Primary Bile acids are synthesized from cholesterol and converted by the
hepatocytes into bile salts
b. Secondary Bile acids are formed by deconjugation and dehydroxylation of the
primary bile salts by intestinal bacteria
6. Discuss the importance of conjugation of bile acids with glycine and taurine
a. Secondary bile acid are conjugated to glycine (Deoxycholic acid) or taurine
(Lithocholic acid) in order to remain soluble in bile secretion
b. Unconjugated bile acid’s buffering capacity is best at pH =5-7 Æ if pH falls bellow
that, they would precipitate out of solution
Brown, Eriksen, Jones, Heffernan, Kanjanavaikoon, Leo, Mishkin, Stern, Verlander &Wasserman
Ed. Mishkin 2006
Page 124 of 168
c. Conjugated bile acids have a much lower pKa and are more completely ionized
making them more water soluble and thus they stay in solution when secreted at
physiological pH
7. Describe bile acid biosynthesis, the rate-limiting enzyme in the synthesis of bile
acids and how it is regulated
a. Bile acids are derivates of cholesterol
i. 7 -hydroxylase catalyzes the formation of cholesterol to bile acid (e.g.
cholic acid) intermediates
ii. The 7 hydroxylation step is rate-limiting
iii. This step is inhibited by bile acids which have been taken up by the
hepatocytes from the portal blood, thereby stopping bile acid synthesis
8. Describe the amphipathic properties of bile acids, and their role in lipid
emulsification and digestion
a. Bile salts are amphipathic molecules because they have both hydrophilic and
hydrophobic portions
b. In aqueous solution, bile salts orient themselves around droplets of lipid and keep
the lipid droplets dispersed (emulsification)
c. Bile salts thus aid in the intestinal digestion and absorption of lipids by emulsifying
and solubilizing them in micelles
9. Describe a “micelle,” list the components of a “mixed micelle,” and discuss the
properties of micelles in lipid absorption
a. Above a critical micellar concentration (CMC), bile salts form micelles
b. Bile salts are positioned on the outside of a micelle, with their hydrophilic portions
dissolved in the aqueous solution of the intestinal lumen and their hydrophobic
portions dissolved in the micelle interior
c. Free fatty acids and monoglycerides are present in the inside of the micelle,
essentially “solubilized” for subsequent absorption
d. Mixed micelles, composed of bile salts and phospholipids Æ are able to solubilize
other lipids more effectively than when they are composed of bile salts alone
because phospholipids can bind to the lipid itself (it’s sortof stickier)
e. Micelles bring the products of lipid digestion into contact with the absorptive
surface of the intestinal cells
10. Describe the major steps in the catabolism of hemoglobin to bile pigment
a. Hemoglobin is metabolized into bilirubin and biliverdin (the two principal bile
pigments) in the liver and conjugated as glucoronides from excretion
b. Intestinal bacteria metabolize bilirubin further to urobilin, which is responsible for
the brown color of stool
c. If bilirubin is not secreted by the liver, it builds up in the blood and tissues,
producing jaundice
11. Contrast the composition of hepatic bile and gallbladder bile in terms of electrolyte
composition and osmolality
a. Hepatic bile has a water and electrolyte concentration that equals that of plasma
i. Na+ = 140-150, K+ = 4.0-4.5, Cat+ = 2.5-4, Cl- = 80-100, HCO3 =25
b. Gallbladder bile, however, is more concentrated than plasma
i. Gallbladder concentrates bile by absorbing Na+, Cl-, HCO3-, and water
from the bile, increasing the bile acid concentration 5-20 fold
ii. Na+ = 300, K+ = 10, Cat+ = 2.0, Cl- = 5, HCO3 = 12
12. Explain why the osmolality of gallbladder bile is not as great as expected from its
ionic composition
a. The osmolality of bile is less than its molality because the micellar properties of a
bile salt solution results in the formation of macromolecular ion complexes
Brown, Eriksen, Jones, Heffernan, Kanjanavaikoon, Leo, Mishkin, Stern, Verlander &Wasserman
Ed. Mishkin 2006
Page 125 of 168
b. Concentration of bile in the gallbladder can be explained by the absorption of an
isotonic solution of NaCl and NaHCO3, leaving what is essentially a concentrated
but isoosmotic solution of sodium bile salts
c. The sum of hydrostatic and osmotic pressures in the intraepithelial space,
interstitium, and capillaries favors fluid absorption from the gallbladder lumen, thus
concentrating the organic components of bile 5-20 fold
13. Describe the role of bile acids in stimulating bile flow (bile-acid-dependent
secretion)
a. The bile-dependent secretion refers to the quantity of bile salts secreted by the liver
b. The amount of bile salts secreted is directly related to the amount of bile reabsorbed
by the hepatocytes (i.e. the more bile reabsorbed from the portal circulation, the
more bile secreted by the liver)
i. The total amount of bile is relatively constant Æ the liver has a limited
synthetic capacity, there is a limit to the amount of bile that can be secreted
ii. Substances that enhance bile secretion are called choleretics Æ Bile salts
and bile acids are the major choleretics
1. That makes bile salts their own biggest fans
2. To be a choleretic, you have to work on the liver directly
iii. The synthesis and secretion of bile by the liver is not under any direct
hormonal or nervous control
14. Describe the role of secretin in stimulating biliary bicarbonate secretion and flow
(bile-acid-independent secretion)
a. The bile-acid-independent secretion refers to the amount of fluid, composed of
electrolytes and water, that is secreted each day by the liver
b. Secretion of this fluid is controlled by the hormone secretin
c. This fluid has a high concentration of HCO3-
15. Describe fluid reabsorption by gallbladder, and its role in concentrating the organic
constituents of bile
a. See #12 a-c
16. Discuss the role of CCK and nerves in bringing about gallbladder contraction and
sphincter of Oddi relaxation in response to a meal
a. CCK is the major stimulus for gallbladder contraction and sphincter of Oddi
relaxation
i. When chyme enters the small intestine, fat and protein digestion products
directly stimulate the secretion of CCK
b. Vagal stimulation of the gallbladder also causes gallbladder contraction and
sphincter of Oddi relaxation
i. Vagal stimulation occurs directly during the cephalic phase of digestion and
indirectly via a vagovagal reflex during the gastric phase of digestion
17. Describe the enterohepatic circulation of bile acids. Include passive as well as active
mechanisms.
a. Enterohepatic circulation Æ recirculation of bile salts from the liver to the small
intestine and back again. This circulation is necessary because of the limited pool of
bile salts available to help break down and absorb fat
b. Intestinal transport of bile acids (>95% of all bile acids are reabsorbed by the small
intestine)
i. Passive transport of deconjugated bile acids Æ occurs in jejunum and ileum
1. Bacteria in terminal part of ileum and colon deconjugate bile acids
and also dehydroxylate them to produce secondary bile acids Æ
both deconjugation and dehydroxylation lessen the polarity of bile
acids, thereby enhancing their lipid solubility and their absorption
by simple diffusion
Brown, Eriksen, Jones, Heffernan, Kanjanavaikoon, Leo, Mishkin, Stern, Verlander &Wasserman
Ed. Mishkin 2006
Page 126 of 168
ii. Active transport of conjugated bile salts Æ occurs in the ileum
1. Na+ coupled secondary active transport, are present on brush
border of terminal ileum for uptake of conjugated bile acids against
a large concentration gradient
18. Discuss the effect of the enterohepatic circulation on bile acid synthesis and
conservation.
a. Bile acids, whether absorbed by active transport or simple diffusion, are transported
away from the intestine in the portal blood, mostly bound to albumin in plasma
b. In the liver, hepatocytes avidly extract the bile acids from the portal blood
i. In a single pass through the liver, the portal blood is cleared of the majority
of acids
ii. Bile acids in all forms, primary and secondary, conjugated and
deconjugated, are taken up by hepatocytes
iii. Hepatocytes reconjugate nearly all the deconjugated bile acids and
rehydroxylate some of the secondary bile acids Æ these bile acids are
secreted into the bile along with newly synthesized bile acids
c. The circulating pool of bile salts (primary and secondary acids) is approx. 3.6g Æ 4-
8g required to digest and absorb a meal Æ total pool of salts must circulate twice
during digestion of each meal Æ bile salts usually recirculate 6-8 times daily
d. Rate of bile salt synthesis is determine by rate of return to liver Æ Synthesis rate =
0.2-0.4 g/day, which replaces normal fecal losses
19. Discuss in general terms; the pathogenic mechanism(s) of gall stone formation and
jaundice.
a. Gallstones are chiefly composed of cholesterol; the remainder are pigment stones,
composed chiefly of calcium bilirubinate
i. Cholesterol is insoluble in water Æ when bile contains more cholesterol
than can be solubilized in the bile acid phospholipids micelles Æ crystals of
cholesterol form in the bile Æ bile is supersaturated with cholesterol and
stones form
ii. Bile pigment stones Æ mainly comprised of calcium salt of unconjugated
bilirubin Æ Induced by bacterial deconjugation of bilirubin glucoronides
b. Jaundice (a.k.a. cholestasis) Æ a condition where bile cannot flow from the liver to
the duodenum Æ Can be due to biliary obstruction by mechanical means or by
metabolic factors in hepatocytes Æ accumulation of bilirubin in the bloodstream
and subsequent deposition in the skin causes jaundice (icterus)
20. Discuss the effect of ileal resection on the enterohepatic circulation, bile acid
synthesis, gallstone formation and lipid absorption
a. Disruption of the enterohepatic circulation
i. ileal resection or small intestinal disease such as sprue or Crohn’s disease
1. Celiac sprue is a disease of the intestinal mucosa; it causes protein
sensitivity and is treated by putting the patient on a gluten-free diet
ii. Leads to decreased bile acid pool and malabsorption of fat and fat-soluble
vitamins
iii. Clinical manifestations Æ Steatorrhea and nutritional deficiency
iv. Increase in fecal losses of bile salts results in watery diarrhea, because bile
salts inhibit water and Na+ absorption in the colon
v. Removal of distal ileum decreases the rate of return of bile acids in he
portal blood, resulting in a much greater rate of synthesis of bile acids
vi. Formation of cholesterol gallstones occurs with ileal resection Æ
presumably due to the increased synthesis of bile and potential to become
supersaturated leading to stone formation
Brown, Eriksen, Jones, Heffernan, Kanjanavaikoon, Leo, Mishkin, Stern, Verlander &Wasserman
Ed. Mishkin 2006
Page 127 of 168

GI Digestion and Absorption I: Small Intestine, Part I


April 17, Abdulnour-Nakhoul
Functional anatomy, digestion and absorption of carbohydrates and proteins
1. Relationship between surface area and gross/microscopic anatomy of the small
bowel
a. Mucosal surface area can be amplified to form specialized features
i. circular folds increase surface area X3
ii. villi X 30
iii. microvilli X600
2. Major functions of small intestine
a. Mixing and propulsion of luminal contents
b. secretion of hormones
c. secretion of mucous
d. digestion
e. absorption
3. Major starches, disaccharides and monosaccharides
a. Starches – about 55-60% (220-240g per person) of consumed carbohydrate (total
average daily consumption is 400g)
i. amylopectin
1. plant starch
2. glucose polymer with α 1-6 and α 1-4 linkages
ii. amylose
1. glucose polymer with α 1-4 linkages
iii. cellulose
1. plant starch
2. glucose polymer with β 1-4 linkages
iv. glycogen
1. animal starch
v. α-amylases from the saliva and pancreatic secretions hydrolyze 1,4 bonds to
yield maltose, maltotriose and alpha-limit dextrins
vi. maltase, α-dextrinase and sucrase in the intestinal brush border hydrolyze
these oligosaccharides to glucose
b. Dissaccharides
i. sucrose
1. 30% of intake (120g)
2. sucrase Æ glucose and fructose
ii. lactose
1. 6% of intake (24g)
2. lactase Æ glucose and galactose
a. Absence of lactase in the brush border leads to lactose
intolerance, producing osmotic diarrhea in reaction to
lactose consumption.
iii. maltose
1. 2% of intake (8g)
2. Glucose dimers
3. maltase Æ glucose
iv. Trehalose
1. α 1-1 glucose
2. trehalase Æ glucose
c. Monosaccharides: ONLY monosaccharides can be absorbed
Brown, Eriksen, Jones, Heffernan, Kanjanavaikoon, Leo, Mishkin, Stern, Verlander &Wasserman
Ed. Mishkin 2006
Page 128 of 168
i. Glucose
ii. Fructose
iii. Galactose
4. Most important brush border oligosaccharides
a. Glucose/Galactose carrier
i. SGLT1: Sodium/Glucose cotransporter
ii. Sodium gradient provides energy for sugar entry into intestinal epithelial
cells
1. Therefore sodium/potassium pump inhibitors also inhibit glucose
and galactose absorption
b. Fructose carrier
i. GLUT 5: Glucose transporter
1. Facilitated diffusion – cannot be absorbed against a concentration
gradient that does not favor absorption
c. Basolateral transport
i. Fructose, Glucose and galactose all leave basolateral membrane by GLUT2
1. Facilitated diffusion
5. The substrate that is co-transported by both SGLT and most amino acid carriers
a. Cabaret lied to us. It’s sodium that makes the world go around.
6. Features of peptide vs. carbohydrate transport across brush border membrane
a. Protein:
i. normal consumption is 70-90g/day
ii. adequate intake is 35-50g/day but it also needs to cover all of the essential
amino acids
iii. 60% of ingested protein has to be broken down before absorption
1. free amino acids, di- and tri- peptides are digestible
2. Intracellular peptidases break down larger peptides
a. endopeptidases hydrolyze interior peptide bonds
b. exopeptidases hydrolyze one amino acid at a time from the
end of the peptide (at the C-terminus, “carboxy” or N-
terminus “amino” side)
iv. Amino acid transport
1. some basolateral diffusion
2. At least 3 Na+-independent amino acid transporters in the
basolateral membrane
a. Na+-dependent
i. Neutral
ii. Acidic
iii. Imino
b. Na+-independent
i. Neutral
ii. Basic
iii. Larger/hydrophobic
3. amino acid carriers:
a. Neutral or amino mono carboxylic
i. aromatics
1. F, Y, W
ii. aliphatics
1. G, V, I, L, S
iii. H, M, N, Q, C, P, OH-P
Brown, Eriksen, Jones, Heffernan, Kanjanavaikoon, Leo, Mishkin, Stern, Verlander &Wasserman
Ed. Mishkin 2006
Page 129 of 168
iv. Autosomal recessive defect in neutral amino acid
carriers Æ Hartnup disease: Tryptophan can’t
diffuse Æ Pellagra
b. Basic or cationic
i. K, R, Ornithine, C
ii. Impairment of these channels Æ cystinuria
c. Acidic or anionic
i. D, E
d. Imino
i. P, OH-P, GABA, Taurine, G
4. Oligopeptide transporters:
a. Family: proton-dependent oligopeptide transporters aka
POT or PTR
i. Broad selectivity for di- and tripeptides
ii. Most of these transporters have a similar
structure, with 12 transmembrane domains
iii. Example: PepT1 – main intestinal H+/dipeptide
transporter protein. Plays critical role in oral
bioavailability of peptidelike drugs.
b. Major differences between carbohydrate and protein absorption:
i. Polymer absorption
1. Proteins can be absorbed as amino acids, dipeptides, tripeptides
2. Carbohydrates can only be absorbed as monosaccharides
ii. Carriers
1. Amino acids carriers are charge-specific – therefore 4 types

GI Digestion and Absorption Part II: Small Intestine Part II


April 18, 2006
Dr. Abdulnour-Nakhoul

1. Define the role of micelle formation in lipid digestion


a. Mixed micelles are spherical (usually) formations with free fatty acids in the interior
and bile acids on the surface. Micelles serve to bring products of lipid digestion into
contact with the absorptive surface of the intestinal cells.
i. 2-monoglycerides, esp. cholesterol and lysolecithin, are the fats that wind
up in micelles – NOT triglycerides
b. Mixed micelles allow lipids to move through the unstirred water layer – this
is the rate-limiting step in fat absorption, because once through the unstirred
water layer, the fatty acids are able to easily diffuse through the plasma membrane of
cells lining the small intestine
c. Bile acids emulsify lipids by allowing them to form mixed micelles, which are
relatively small units – so the surface area is increased
d. Bile acids are amphipathic
2. Define the major steps involved in lipid absorption
a. Visit McDonald’s, consume copious fat products
b. Fat in the duodenum Æ release of CCK Æ gallbladder contraction, Oddi relaxation
c. Pancreatic lipases are released Æ Cleave fat Æ Cholesterols, glycerols, fatty acids are
separated out Æ Bile acids emulsify fats Æ micelles form Æ diffusion through
unstirred water Æ micelles unform Æ fats diffuse into cells
d. In the intestinal cells, the products of lipid digestion are re-esterified to triglycerides,
cholesterol esters and phospholipids
Brown, Eriksen, Jones, Heffernan, Kanjanavaikoon, Leo, Mishkin, Stern, Verlander &Wasserman
Ed. Mishkin 2006
Page 130 of 168
e. Phospholipids + apoproteins Æ Chylomicrons Æ are exocytosed into the lymphatic
vessels
f. If you’re missing apoproteins Æ no chylomicrons Æ “Abetalipoproteinemia” – a
disease where you can’t digest fat (I wish) but you also can’t absorb fat-soluble
vitamins, often leading to Vitamin E deficiency
3. Describe the role of intrinsic factor in the utilization of dietary cobalamin (B12)
a. In the small intestine, pancreatic proteases digest binding proteins from the chyme,
releasing vitamin B12 (cobalamin)
b. Parietal cells from the corpus of the stomach release intrinsic factor, a glycoprotein
i. In a gastrectomy, you lose your parietal cells and then you have to get B12
shots like Courtney Love
c. R protein, found in saliva, binds to B12 in the stomach Æ R is cleaved off by
pancreatic enzymes Æ intrinsic factor binds in R’s place Æ absorption is facilitated
i. At acid pH, which is obviously how life in the stomach is, R has a higher
affinity for B12 – but in the small intestine, where pH is higher, intrinsic
factor has a higher affinity for B12.
ii. Change in affinity status facilitates change in protein bound to B12
iii. Receptors in the Ileum recognize intrinsic factor Æ B12 bound to intrinsic
factor gets absorbed
iv. B12 is important for the maturation of erythrocytes
1. Loss of B12 Æ pernicious anemia
a. secondary to atrophy of gastric mucosa
4. Describe the mechanisms of calcium absorption within the brush border and
basolateral membranes of the enterocyte
a. Brush border
i. Crosses through calcium channel
ii. Facilitated by Vitamin D (produced in kidney)
b. Inside cell
i. Binds to calbindin or is packaged into vesicles
1. Vitamin D promotes the synthesis of calbindin (most
important action of Vitamin D)
c. Basolateral membrane
i. Calcium in vesicles is released by exocytosis
ii. Calcium-Hydrogen ATPase antiporter
iii. Calcium-Sodium antiporter
iv. Calbindin facilitates
d. More information
i. Calcium is absorbed throughout the small intestine, but is only actively
transported into the cells in the duodenum
ii. Passive paracellular diffusion of calcium also contributes to absorption
iii. Increased acidity of the GI tract increases absorption
1. Calcium citrate is more effectively absorbed
2. Vitamin C helps you absorb calcium
3. Drink your milk, eat your vegetables.
iv. Vitamin D deficency or chronic renal failure results in inadequate intestinal
calcium absorption Æ rickets in children, osteomalacia in adults
5. Describe the mechanism of iron absorption and storage in the enterocyte
a. Average daily intake of iron is 10-20 mg
b. Approximately 10% of the iron entering the digestive system is absorbed
c. Most of the iron we consume is bound to hemoglobin or myoglobin from animal
meat (because we eat the muscle and there’s blood in there)
i. Heme iron is the form found in these globins
Brown, Eriksen, Jones, Heffernan, Kanjanavaikoon, Leo, Mishkin, Stern, Verlander &Wasserman
Ed. Mishkin 2006
Page 131 of 168
d. Other iron we absorb is free iron Fe2+
i. transported in blood, while bound to transferrin
e. Inside epithelial cells: 2 pools of iron
i. 1 pool of iron available for absorption into blood
1. in blood, free iron is bound to transferrin
a. transferrin transports iron from the small intestine to its
storage sites in the liver, and from the liver to the bone
marrow for hemoglobin synthesis
ii. second pool is bound to ferritin within epithelial cells
1. This process makes the iron unavailable for transport across the
basolateral membrane
2. This iron is lost into the lumen when the epithelial cell is sloughed
off – preventing excessive iron absorption
f. Iron deficiency is the most common cause of anemia
6. Identify the luminal and basolateral ion transporters involved in electrolyte secretion
across the epithelial cells in the crypt of Lieberkuhns
a. Luminal
i. Chloride enters the lumen via an electrogenic channel (actively, powered by
sodium gradient) Æ results in negative luminal charge Æ results in sodium
secretion into lumen (partly via tight junctions)
1. Water follows to maintain isoosmoticity
b. Basolateral
i. Na/K/2Cl cotransporter
ii. Potassium leaves the cell via potassium channels, facilitated by calcium and
cAMP

GI Digestion and Absorption III: Large Intestine


April 19, 2006
Dr. Abdulnour-Nakhoul

1. Components contributing to fluid load presented to the colon


a. Ingestion – 2000 ml/day
b. GI tract secretes 7L/day
i. Saliva – 1500 ml/day
ii. Gastric secretion – 2000 ml/day
iii. Bile – 500 ml/day
iv. Pancreatic Juices – 1500 ml/day
v. Intestinal secretions – 1500 ml/day
c. Small intestine absorbs 8500 ml/day
d. Colon absorbs 80-90% of the 500 ml passed to it
e. And the rest comes out fine in the end.
2. 4 major groups of sodium transporters present in the large and small intestines
a. luminal
i. SGLT 1 - Na/glucose cotransporter
1. Jejunum
2. Ileum
ii. Na/amino acid cotransporter
1. Jejunum
2. Ileum
iii. Na/H antiporter
1. Jejunum
2. Ileum
Brown, Eriksen, Jones, Heffernan, Kanjanavaikoon, Leo, Mishkin, Stern, Verlander &Wasserman
Ed. Mishkin 2006
Page 132 of 168
3. Colon
iv. Na/Cl cotransporter
1. BRS doesn’t say where.
v. Sodium channel
1. Passive diffusion
2. Colon
b. Basolateral
i. Na/HCO3 cotransporter (both into cell)
1. Ileum
2. Colon
ii. Na/K ATPase
1. Everywhere
c. And by 4, we mean 7.
3. Relationship between fluid and electrolyte absorption
a. Water absorption is secondary to solute absorption Æ maintains tonicity
4. Transporters directly influencing pH of large and small intestines
a. H+ transporters
i. Na/H antiporter – H goes into the lumen
1. Jejunum
2. Ileum
3. Colon
ii. K/H ATPase – active pump because lumen is negatively charged - H goes
into the cell
1. Colon
b. HCO3 transporters
i. Bicarb/Cl antiporters
5. ATPase responsible for hydrogen secretion in the stomach and potassium absorption
in the colon
a. We don’t know how to answer this question because potassium is secreted in the
colon. Geesh. They must mean K/H ATPase antiports.
i. In the stomach, H goes into the lumen while K goes into the cell (parietal
cells)
1. Believe it or not, some potassium is also secreted in the stomach
(also by parietal cells)
ii. In the colon, K goes into the lumen while H goes into the cell
6. Factors controlling salt and water transport in the gut
a. Endocrine control (all promote net absorption)
i. Aldosterone
1. >Epithelial Na Channels (ENaC)
2. Na/K ATPase
ii. Glucocorticoids
1. Na/K ATPase
iii. Epinephrine
1. >NaCl absorption
iv. Angiotensin II
1. >NaCl absorption
b. Paracrine (all promote net absorption)
i. Somatostatin
1. >absorption, <motility
ii. Opioids
1. >absorption, <motility
c. Neural regulations
Brown, Eriksen, Jones, Heffernan, Kanjanavaikoon, Leo, Mishkin, Stern, Verlander &Wasserman
Ed. Mishkin 2006
Page 133 of 168
i. Enteric – promotes net secretion
1. Gut distension, luminal glucose, acid pH
2. Acetyl choline
3. Vasoactive intestinal peptide (VIP)
4. Secretin
ii. Parasympathetic – promotes net secretion
1. Cholinergic
iii. Sympathetic – promotes net absorption
1. Catecholamines
2. Alpha-adrenergic
d. GI Immune system (all stimulate net secretion)
i. Histamine – secreted by enterochromaffin-like cells
ii. Serotonin
iii. Prostaglandins
iv. Leukotrienes
v. Arachidonic acid
vi. Nitric Oxide
e. Bacterial enterotoxins – stimulate net secretion
i. Cholera, E Coli, Chlostridium
f. Laxatives – stimulate net secretion
i. Bile acids, ricinoleic acid
7. 5 Major components of feces
a. Bacteria 30%
b. Undigested fiber 30%
c. Lipids 10-20%
d. Organic matter 10-20%
e. K+ and Bicarb
8. Major components of gas in the GI tract
a. Swallowed air
b. CO2 from HCO3/H combination
c. Volatile products of bacterial digestion
d. Diffusion from blood to lumen

Block IV: Endocrine


LOs for May 1st: Endocrine Hormones – Structure-function and mechanisms

1. Hormone classification and properites & overall functioning of endocrine system


a. A hormone is a chemical substance produced by specialized cells, often carried by
the bloodstream to act on distant target cells.
b. A hormone binds to a receptor on a cell, eliciting a specific response within that cell.
c. Hormones effect homeostasis, regulate processes of growth and development,
reproduction, and senescence (the process of aging).
2. Classify and differentiate between endocrine, paracrine, autocrine and neurocrine
hormone actions
a. endocrine: secreted into body fluids and acts away from its point of origin
b. paracrine: secreted and acts close to its point of origin (neighboring cells)
c. autocrine: acts on its own source (on the actual cell that secreted it)
d. neurocrine: secreted by a nerve ending and acts on non-nervous tissue
e. neurotransmission (for Sara): secreted by a nerve ending and acts on another neuron
Brown, Eriksen, Jones, Heffernan, Kanjanavaikoon, Leo, Mishkin, Stern, Verlander &Wasserman
Ed. Mishkin 2006
Page 134 of 168
f. These words describe actions, not hormones’ identities; any given substance can act
in more than one of these capacities.
3. know the major endocrine glands and depict the structure-function relationship of
various hormones
a. Classic endocrine glands
i. anterior pituitary
1. adrenocorticotropic hormone (ACTH)
a. ↑ steroid hormone synthesis in the adrenal cortex
b. Hypothalamus Æ Corticotropin releasing hormone (CRH)
Æ Anterior pituitary Æ ACTH Æ Adrenal Cortex Æ
Cortisol Hypothalamus and Anterior Pituitary
2. growth hormone (GH)
a. GH Æ IGF
i. IGF makes you grow, ↑ protein synthesis in
muscle Æ ↑in lean body mass, ↑ organ size
b. Hypothalamus Æ GHRH Æ [Ant pit Æ GH] Æ
[Hypothalamus Æ Somatostatin] [Ant Pit Æ GH]
3. prolactin (PRL)
a. Lactogenesis
b. PRL + estrogen Æ boobies
c. homologous to GH
d. Hypothalamus Æ Thyrotropin releasing hormone (TRH)
Æ Ant Pit Æ Prolactin Æ [Hypothalamus Æ Dopamine]
[Ant Pit Æ PRL]
i. This action of dopamine makes it an
important prolactin inhibiting factor or PIF
4. thyroid stimulating hormone (TSH)
a. Thyroid hormones secreted due to this pathway promote
growth, bone formation, bone maturation, and are
involved in CNS, ↑Basal metabolic rate, CV and
respiratory system (↑ CO and respiratory rate), etc
b. Hypothalamus Æ TRH Æ Ant Pit Æ TSH Æ Thyroid Æ
T3, T4 Ant Pit
5. luteinizing hormone (LH)
a. Ovulation, corpus luteum, estrogen and progesterone
b. In males, synthesis and secretion of testosterone in the
testis
c. Ant Pit Æ LH Æ Ovaries secrete estrogen Æ estrogen Æ
anterior pituitary releases LH
i. only works during periods of high estradiol, on
day 15 of the menstrual cycle
ii. Only positive feedback we’ve seen
iii. Sorry, I meant to say you’re doing great!
6. follicle-stimulating (FSH)
a. Stimulates maturation of ovum (i.e., follicle)
b. In males, spermatogenesis
c. In females: Hypothalamus Æ GnRH Æ Ant Pit Æ FSH
& LH Æ Ovary Æ Estrogen & Progesterone [Ant
Pit Æ FSH & LH]
Brown, Eriksen, Jones, Heffernan, Kanjanavaikoon, Leo, Mishkin, Stern, Verlander &Wasserman
Ed. Mishkin 2006
Page 135 of 168
d. In males: Hypothalamus Æ GnRH Æ Ant Pit Æ FSH Æ
Sertoli cells Æ maintain spermatogenesis, secretion of
inhibin [Ant Pit Æ FSH]
ii. posterior pituitary
1. vasopressin (ADH)
a. ADH increases water permeability of the principal cells of
the late distal tubule and collecting duct
b. ADH constricts vascular smooth muscle
c. Factors that ↑ADH:
i. ↑ serum osmolarity
ii. blood volume decrease (“contraction”)
iii. hypoglycemia
iv. nicotine and opiates
d. Factors that ↓ADH:
i. ↓serum osmolarity
ii. ethanol
iii. ANP
iv. alpha-adrenergic agonists
2. oxytocin
a. causes ejection of milk from the breast
i. stimulated by suckling, and psychological factors
relating to the presence of a baby
b. stimulates uterine contractions
c. in men, oxytocin is also released. also from suckling and
related activites.
i. function unknown, may be related to ejaculation
iii. Thyroid gland
1. thyroxine (T4) – see TSH
2. triiodothyronin (T3) – see TSH
3. calcitonin
a. produced by C cells of the thyroid
b. reduces blood calcium
i. ↓intestinal absorption of calcium
ii. ↓osteoclast activity in bones
iii. decreasing calcium and phosphate reabsorption in
kidney tubules
c. responds to ↑ plasma calcium
iv. Parathyroid glands
1. parathyroid hormone (PTH)
a. Secreted by chief cells of parathyroid glands
b. ↑ plasma calcium concentration
i. binds to osteoblasts Æ osteoclasts stimulated Æ
bone breakdown Æ ↑plasma calcium
concentration
v. testes
1. testosterone
a. synthesized by leydig cells in the testes
b. secondary sexual characteristics in men
c. LH Æ Leydig cells Æ testosterone [anterior pituitary
Æ LH]
Brown, Eriksen, Jones, Heffernan, Kanjanavaikoon, Leo, Mishkin, Stern, Verlander &Wasserman
Ed. Mishkin 2006
Page 136 of 168
i. only unbound testosterone has this suppressive
effect
vi. ovaries
1. estradiol
a. from granulosa cells of ovaries
b. Ægrowth and development of female reproductive organs,
secondary female characteristics, increased bone density
c. controls follicular phase of menstrual cycle, thickening of
endometrium
d. estradiol [hypothalamusÆGnRH Æ Ant pituitary Æ
LH, FSH]
i. this is the basis of oral contraceptives
2. progesterone
a. controls luteal phase of the menstrual cycle
b. released by placenta during pregnancy Æ helps maintain
pregnancy by maintaining endometrium
c. progesterone [hypothalamus Æ GnRH Æ Ant
pituitary Æ FSH & LH]
i. present in oral contraceptives because unopposed
estrogen would increases the risk of uterine cancer
3. shout out to Pamela Jones!
vii. pancreas
1. insulin
a. released in presence of blood glucose
b. cells in pancreas
i. Glucose Æ GLUT 2 receptors Æ ↑ ATP Æ
closes potassium channels Æ no more K+ in Æ
depolarization Æ open calcium channels Æ more
calcium influx Æ vesicular release of insulin
c. Æ ↓blood glucose, fatty acids and amino acids
d. in starvation Æ # of receptors ↑
e. in gluttony Æ # of receptors decreases
f. 3 ways insulin decreases blood glucose:
i. ↑ uptake
ii. promotion of glycogen formation
iii. ↓gluconeogenesis
2. glucagon
a. released from α cells
b. primarily acts on hepatic and adipose tissue
c. ↑ blood glucose
i. glycogenolysis
ii. ↑ gluconeogenesis
3. somatostatin
a. delta cells
b. glucagon, insulin, gastrin
b. Non-classic endocrine glands
i. hypothalamus
1. releasing and inhibiting hormones – GHRH, LHRH, TRH,
CRH…as per section a of this question
ii. heart
Brown, Eriksen, Jones, Heffernan, Kanjanavaikoon, Leo, Mishkin, Stern, Verlander &Wasserman
Ed. Mishkin 2006
Page 137 of 168
1. atrial natriuretic peptide
a. blood pressure Æ right atrium Æ ANP Æ Na+ excretion
Æ osmotic water excretion Æ ↓blood volume Æ ↓blood
pressure
iii. kidney – see block II. Honestly, now.
1. erythropoietin
2. renin
iv. liver
1. insulin-like growth factor “IGF-1”
a. covered under GH
v. platelets
1. platelet-derived growth factor – PDGF
a. regulates cell growth and division
2. transforming growth factor β – TGF β
vi. macrophages
1. cytokines
a. signalling particles for immune response
vii. gastrointestinal tract – see block III.
1. gastrin
2. secretin
3. VIP
c. structure-function relationships
i. proteins and polypeptides
1. generally water-soluble
2. circulate unbound in plasma
3. vary greatly in size, weight etc.
ii. amino acid derivatives
1. catecholamines are water-soluble
a. derived from tyrosine
2. thyroid hormones are lipid-soluble
a. derived from two iodinated tyrosine residues
b. the only substances in the body that contain iodine
i. salt is iodinated so that we can produce TH
3. circulate in plasma, bound mainly to binding globulins
iii. steroid hormones
1. lipid-soluble
2. circulate in plasma, bound to carrier proteins (steroid-binding
globulins)
4. Feedback regulation of hormone production
a. Positive feedback:
i. When presence of a substance leads to an increase in the release of the
same substance
ii. Only example we’ve learned is LH
b. Negative feedback:
i. When presence of a substance leads to a decrease in the release of that
substance. This is often accomplished, as we saw in biochem, via inhibition
of the pathway leading to the production of that substance.
1. See throughout list of hormones for their individual negative
feedback mechanisms – hopefully this was already obvious, but
I’ve shown inhibition as
5. Mechanisms of action of hormones and hormonal cascades
Brown, Eriksen, Jones, Heffernan, Kanjanavaikoon, Leo, Mishkin, Stern, Verlander &Wasserman
Ed. Mishkin 2006
Page 138 of 168
a. See answer to Question 1
6. Whole-body responsiveness to hormones
a. Hormonal control is dependent on receptors. Receptor types are what determine
body response. So the extent of hormonal influence is based on which receptors are
present throughout the body.
b. This may or may not be what Pandey meant for us to respond. If anyone has ideas
on what this question means, feel free to share.
7. Plasma binding proteins, free hormones, inactivation/degradation & excretion
a. Plasma binding proteins bind hormone Æ turnover rate of hormone
b. metabolic clearance rate
i. MCR = mg/min removed / mg/ml plasma = ml plasma cleared / min
ii. Kidney and liver are the major sites of metabolic degradation of hormones
c. Breakdown pathways:
i. proteolysis, oxidation, reduction, hydroxylation, decarboxylation,
methylation
d. **We’re still trying to figure out what the specific mechanism of action of the
plasma binding proteins is, and how it relates to inactivating hormones, but in
general, plasma binding proteins bind to hormones, and it changes the delivery of
hormones to the tissues. If you know more than this, please let me know.
8. explain the methodologies to measure hormone production
a. Radio Immunoassay (RIA)
i. good for very small quantities
ii. measures antigenic amounts of hormone – may even be sensing an amount
below the threshold of biological activity
iii. Technique:
1. A mixture is prepared of
a. radioactive antigen
i. Because of the ease with which iodine atoms can
be introduced into tyrosine residues in a protein,
the radioactive isotopes 125I or 131I are often used.
b. antibodies against that antigen.
2. Known amounts of unlabeled ("cold") antigen are added to
samples of the mixture. These compete for the binding sites of the
antibodies.
3. At increasing concentrations of unlabeled antigen, an increasing
amount of radioactive antigen is displaced from the antibody
molecules.
4. The antibody-bound antigen is separated from the free antigen in
the supernatant fluid, and the radioactivity of each is measured.
b. Enzyme-linked immunosorbent assay (ELISA)
i. It’s basically the same thing except that you use a chromogenic or a
fluorogenic marker, which fluoresce under UV light
ii. You don’t use radiolabels, so you won’t create radioactive labtechs
iii. this is what people working in labs actually use to diagnose HIV, HepB,
Pregnancy, hypothyroidism, etc.
9. Feedback regulation of hormone synthesis and secretion
a. peptide hormones
i. synthesis
1. Gene trasncription Æ mRNA Æ preprohormone synthesized on
ER ribosomes Æ signal peptide cleavage in ER membrane to
prohormone Æ initial glycosylations in ER Æ transport to trans-
Golgi apparatus for further proteolytic processing/modification of
Brown, Eriksen, Jones, Heffernan, Kanjanavaikoon, Leo, Mishkin, Stern, Verlander &Wasserman
Ed. Mishkin 2006
Page 139 of 168
glycosylated residues Æ mature hormone is placed in vesicle for
future secretion
ii. secretion
1. GTP binding protein attaches vesicles to specific sites within
cytoskeleton
2. calcium influx Æ activates myosin light-chain kinase on vesicle
surface Æ Phosphorylated myosin interacts with microfilaments Æ
creates conveyer belt made of microtubules Æ membrane of
secretory granule (vesicle) and plasma membrane fuse Æ common
membrane is lysed Æ hormone is released into interstitium
3. usually concurrently: cAMP Æ PKA Æ phosphorylation of tubulin
Æ microtubules form Æ contributes to cellular vesicle movement
apparatus Æ facilitates pathway of vesicle release
b. catecholamines
i. synthesis
1. Synthesized from tyrosine

ii. secretion
1. same as peptides (probably because they’re amino acids)
c. thyroid hormones
i. synthesis & secretion
1. Tyrosine attaches to thyroglobulin (a glycoprotein) Æ complex is
exocytosed into follicular lumen Æ at the membrane, tyrosine
residues interact with I2 Æ monoiodotyrosine (MIT) +
diiodotyrosine (DIT) on thyroglobulin Æ These complexes
combine: MIT +DIT is T3, DIT+DIT is T4 (i.e., you count the
iodines), either way, it’s still bound to thyroglobulin Æ complex is
endocytosed Æ thyroglobulin is cleaved off Æ T3 and T4 are
released into blood via simple diffusion
2. Endocytotic event is stimulated by TSH
d. steroid hormones
i. synthesis
1. Cholesterol Æ Pregnenolone (rate-limiting) Æ
a. progesterone Æ
Brown, Eriksen, Jones, Heffernan, Kanjanavaikoon, Leo, Mishkin, Stern, Verlander &Wasserman
Ed. Mishkin 2006
Page 140 of 168
i. 11-Deoxycorticosterone Æ corticosterone Æ
aldosterone
ii. 17-Hydroxyprogesterone Æ
1. 11 deoxycortisol Æ cortisol
2. androstenedione Æ testosterone Æ
estradiol
b. 17hydroxypregnenolone Æ
i. 17hydroxyprogesterone Æ (see above)
ii. dehydroepiandrosterone Æ androstenedione Æ
(see above)
ii. secretion
1. simple diffusion as it’s processed
2. therefore activation of synthesis is tantamount to activation of
secretion
10. understand the ubiquity of hormone control mechanisms and different levels of
cellular metabolism
a. Ubiquity: Oh, we’ve got it --they’re everywhere and do everything. Just read the last
6 ½ pages and you’ll be convinced too.

LOs for May 2nd: Endocrine Hormones and Mechanisms of Action – Receptors, Second
Messengers and Signalling

1. Know the major classes of signal-generating hormone receptors


a. Extracellular receptors aka Membrane receptors
i. Domains
1. Hormone binding domain is outside the cell
2. Active domain that will transduce the signal is inside the cell
ii. Ligands are hydrophilic
iii. Actions
1. Binding event Æ↑or ↓ second messenger Æoften ↑or
↓phosphorylation Æ various effects such as vesicle release and
indirect influence of transcription
iv. Categories
1. ionotropic (like ligand-gated ion channels)
2. metabotropic (like G-protein coupled receptors)
v. Action is on the level of minutes to hours
1. largely aided by second messengers Æ exponential ↑in action
b. Intracellular receptors
i. Bound to heat-shock proteins (HSPs) inside cytoplasm
ii. Binding to receptor Æ receptor can dissociate from HSP Æ ligand-receptor
complex travels in cell Æ binds to hormone response element (HRE)
located in the promoter region of a targeted gene
1. Transcription is influenced directly
2. Multiple complexes can bind to HREs on the same gene Æ
additive effect
iii. Action is on the level of hours to days
2. Differentiate function, location and action of different hormone receptor types
a. See #1.
3. Know the major types of hormone-induced second messengers involved in hormone-
dependent signal transduction
a. cAMP
Brown, Eriksen, Jones, Heffernan, Kanjanavaikoon, Leo, Mishkin, Stern, Verlander &Wasserman
Ed. Mishkin 2006
Page 141 of 168
i. Involved in metabolic pathways
ii. Often stimulates kinases (PKA)
iii. cAMP Æ increases calcium by opening calcium channels and allowing
extracellular calcium to enter
b. cGMP
i. Involved in vasodilation
c. Calcium
i. Involved in hormone and neurotransmitter release
1. calcium influx Æ activates myosin light-chain kinase on vesicle
surface Æ Phosphorylated myosin interacts with microfilaments Æ
creates conveyer belt made of microtubules Æ membrane of
secretory granule (vesicle) and plasma membrane fuse Æ common
membrane is lysed Æ hormone is released into interstitium
d. Inositol 1,4,5-triphosphate or IP3 + Diacylglycerol
i. Mobilizes calcium from intracellular storage and regulates cell growth
4. Classify four major types of second messenger-dependent protein kinases and their
roles in signal transduction
a. PKA
i. cAMP-dependent
ii. phosphorylates serine and threonine residues
b. PKG
i. cGMP-dependent
ii. phosphorylates serine and threonine residues
c. PKC
i. Activated by calcium and phospholipids such as DAG
ii. Stimulates cell division
iii. Dysregulation can result in tumors
iv. 12 isoforms, classified as classical, novel and atypical
d. Calmodulin-dependent protein kinase
i. Activated by calcium-calmodulin complex
ii. Most abundant in the nervous system
iii. Involved in exocytotic release of neurotransmitters
5. Describe the mechanisms of action and general functions of receptor tyrosine
kinases
a. This is a membrane-bound receptor with kinase activity
i. One transmembrane domain
ii. Tyrosine is phosphorylated
b. Main ligands are insulin and growth factors
c. Growth factor Æ Binds to receptor Æ
i. Calcium influx,
ii. ↑ Na+/H+ exchange,
iii. Stimulation of Phospholipase C (PLCα)
d. Overexpression or overactivation Æ hypertrophy, vascularization of hypertrophic
regions (i.e., cancer)
6. Describe the role of intracellular tyrosine kinases in signal transduction
a. JAK is an intracellular tyrosine kinase associated with a transmembrane receptor for
some growth hormones
b. Growth Hormone binding events cause receptor dimerization and bind JAKs
c. Bound JAKs phosphorylate JAKs and STATs
d. Phosphorylated STATs dimerize Æ translocate to the nucleus Æ phosphorylate
transcription factors
Brown, Eriksen, Jones, Heffernan, Kanjanavaikoon, Leo, Mishkin, Stern, Verlander &Wasserman
Ed. Mishkin 2006
Page 142 of 168
e. All of these phosphorylation events have taken place on the tyrosine residues of the
relevant proteins
f. The cat in the hat came back, stat.
7. Compare the active and inactive states of G-proteins and identify at least four main
types of effector systems activated by G-proteins
a. Heterotrimeric G-protein coupled receptors (GPCRs)
i. Inactive GPCR is bound to GDP Æ Binding event Æ GTP replaces GDP
on GPCR Æ GTP-bound is active Æ α, β, γ subunits fall off of GPCR
Æ α subunit dissociates from β, γ subunits Æ α subunit activates cAMP
ii. Meanwhile, Æ GTP on GPCR releases one phosphate, becomes GDP Æ
α, β, γ subunits reassociate themselves with the GPCR Æ return to
constitutive inactive state
iii. Different categories of G proteins
1. Depending on the activity of the particular complex, it can be
stimulatory (Gs, αs) or inhibitory (Gi, αi)
2. GQ Æ activates PLC Æ cleaves phosphotidylinositol 4,5
bisphosphate Æ IP3 + DAG
a. Can activate PLA2 Æ cleaves membrane phospholipids Æ
releases arachodonic acid Æ Prostoglandins, Prostocylcins,
Thromboxane, and Leukotrienes Æ Inflammation
3. GT (T for transducin) is a specialized GPCR:
a. Photon Æ Rhodopsin Æ Activates GT Æ cGMP
phosphodiesterase Æ ↓cGMP Æ closes cGMP-activated
Na channels Æ hyperpolarization
b. Monomeric GPCRs:
i. Ras, Rho, Rab – monomeric G proteins, all constitutively bound to GDP
(inactive in this state)
1. Activation occurs via Guanine Nucleotide Releasing protein or
GNRP, which transfers a phosphate from free GTP to the GDP
on the monomeric GPCR Æ GTP-bound is active
2. Inactivation occurs via GTPase-activating protein (GAP), which
cleaves a phosphate off, recreating the GDP constitutive, inactive
status
ii. Ras and Rho are involved in linking GF receptor tyrosine kinases to their
intracellular effects
8. Understand the role of G-protein mechanisms in the symptoms of cholera
a. Cholera toxin catalyzes the transfer of ADP-ribose to an s Æ can’t bind back on
to its GPCR but it can still activate cAMP Æ cAMP becomes constitutively
stimulated Æ Cl- channels are constitutively opened Æ Cl- leave cell Æ Na+
follows chloride out to maintain electric gradient Æ water osmotically follows Æ
significant ↑in water and salt excretion (up to 20L a day of water stool) Æ
dehydration
b. rehydration is accomplished via glucose-Na+ coupled absorption Æ water is pulled
in to follow glucose even if sodium is lost
9. Compare the functional properties of two main types of protein phosphatases
a. Serine-Threonine protein phosphatases
i. Protein Phosphatase I (PP1)
ii. Protein Phosphatase II (PP2) – based on regulation by divalent cations
1. PP2A
2. PP2B – regulated by calcium-calmodulin. aka Calcineurin.
3. PP2C
Brown, Eriksen, Jones, Heffernan, Kanjanavaikoon, Leo, Mishkin, Stern, Verlander &Wasserman
Ed. Mishkin 2006
Page 143 of 168
b. Tyrosine phosphatases – both membrane-bound (which are sometimes receptors as
well) and cytosolic
10. Know the significance of atrial natriuertic peptide receptor-coupled membrane
guanylyl cyclase and nitric oxide-depndent cytosolic guanylyl cyclase signal
transduction systems
a. ANP
i. ↑ blood pressure Æ right atrium Æ ANP Æ activates a membrane-bound
guanylyl cyclase/NP receptor (with an ANP-binding exracellular domain, a
single transmembrane domain and an intracellular guanalyl cyclase catalytic
domain) Æ ↑ intracellular cGMP Æ activates PKG Æ
1. vasorelaxation Æ ↓blood pressure
2. Na+ excretion which Æ osmotic water excretion Æ ↓blood
volume Æ ↓blood pressure
b. NO
i. paracrine that causes vasodilation
ii. NO Æ stimulates cytosolic guanylyl cyclase Æ cGMP Æ PKG Æ
vasorelaxation

LOs for May 3rd: Pancreatic Hormones: Insulin, Glucagon, Somatostatin

1. Biological actions of pancreatic hormones: insulin, glucagons and somatostatin


a. Insulin
i. Acts on the liver, adipose tissue, and muscle
ii. ↓blood gluocse concentration:
1. inserts glucose transporter GLUT4 Æ ↑ glucose uptake into cells
2. upregulates F2,6BPase Æ
a. stimulates glycolysis Æ promotes glycogen formation
b. inhibits gluconeogenesis Æ demotes glucose formation
iii. ↓fatty acid and ketoacid in blood
1. in adipose – insulin
a. Æ fat deposition
b. inhibits lipolysis
2. in the liver: ↓fatty acids Æ ↓Acetyl CoA Æ ↓ketoacid formation
a. Insulin is THE major antiketogenic hormone
iv. ↓amino acids in the blood
1. insulin Æ ↑ aa uptake, ↑ protein synthesis, ↓ protein degradation
(anabolic)
v. ↓blood potassium concentration
1. ↑ uptake into cells
b. Glucagon
i. Glucagon acts on the liver and adipose tissue
ii. Second messenger is cAMP
iii. Glucagon increases blood glucose concentration
1. ↑ glycogenolysis
2. prevents recycling of glucose into glycogen
3. ↓F26BPase Æ ↑ gluconeogenesis
iv. ↑ fatty acid and ketoacid blood concentrations
1. ↑ lipolysis
a. Beta-oxidation of fat uses NAD Æ ↓in NAD prevents
oxidation of ACoA in the TCA cycle Æ
2. Æ ↑ ACoA Æ ↑ ketoacid formation
Brown, Eriksen, Jones, Heffernan, Kanjanavaikoon, Leo, Mishkin, Stern, Verlander &Wasserman
Ed. Mishkin 2006
Page 144 of 168
v. ↑ urea production
1. Amino acids are used more by increased gluconeogenesis Æ more
amino acid groups are incorporated into urea Æ ↑ urea
c. Somatostatin
i. inhibits insulin, glucagon and gastrin secretion
2. Structure-function relationship between proinsulin and insulin molecules
a. Proinsulin is synthesized as a single-chain peptide. Within storage granules,
proteases remove a connecting (C) peptide Æ insulin
i. C peptide is secreted along with insulin
b. 99% of proinsulin is packaged in the golgi
i. 1% of proinsulin just diffuses out of the cell Æ still gets cleaved Æ low
level of constitutive insulin
c. Insulin has A & B chains, joined by two disulfide bridges
i. A – 21 amino acids
ii. B – 30 amino acids
d. Proinsulin has the additional C peptide – 33 additional amino acids
3. Synthesis, secretion & regulation of insulin
a. Synthesis – see # 2
b. Mechanism of secretion: Glucose binds to GLUT 2 on beta cells Æ glucose is
oxidized to ATP inside beta cells Æ K+ channels close Æ K+ stops flowing out Æ
depolarization Æ Calcium channels open Æ calcium flows in Æ vesicular fusion
with the membrane is stimulatedÆinsulin secreted
i. Result is a burst of insulin followed by sustained secretion in waves
(pulsatile) for about an hour following sugar consumption
c. Regulation of secretion:
i. On mechanism: ↑ blood glucose concentration Æ insulin secretion
ii. Off mechanism: Oxidation of glucose Æ ↑ ATP has a second effect:
1. Æ Uncoupling protein 2 (UCP-2) dissociates ATP generation from
glucose oxidation Æ ↓ insulin secretion
iii. Hypothalamic control of feeding behavior also effectively plays a role in
insulin levels
iv. Drug-based regulation
1. Sulfonyl urea drugs – used in type II diabetes
a. bind to SUR Æ close K+ channels Æ insulin release
2. Diazoxide – used in hyperinsulinemia
a. opens K+ channels insulin release
4. Describe the interaction of insulin with its receptor and the significance of insulin
receptor substrates (IRSs)
a. Insulin receptor is found on target tissues for insulin
b. Insulin receptor has two α subunits and two β subunits
c. β subunits span the cell membrane and have tyrosine kinase activity Æ insulin binds
receptor Æ tyrosine kinase autophosphorylates the β subunits Æ phosphorylated
receptor is activated to phosphorylate other proteins Æ insulin-receptor complex
enters target cells
i. insulin down-regulates its own receptors in target tissues
1. # of insulin receptors is ↑ in starvation and ↓in gluttony
d. IRS:
i. IRS-1 and IRS-2 are expressed in muscle, adipose, pancreatic β cells
ii. IRS-3 is expressed in the brain
Brown, Eriksen, Jones, Heffernan, Kanjanavaikoon, Leo, Mishkin, Stern, Verlander &Wasserman
Ed. Mishkin 2006
Page 145 of 168
iii. Insulin binding event activates IRS Æ IRS serves as a docking station for
other protein kinases to come get activated Æ subsequent phosphorylation
events lead to all of the effects of insulin
iv. IRS 1 Æ activates growth receptor binding protein 2 (GRB2) Æ
protooncogene RAS GTP complex Æ mitogen-activated protein kinase
(MAPK) Æ cell growth and differentiation
5. Differentiate between the source and structure of two variant forms of glucagons
a. 30-40% of plasma glucagon is of pancreatic origin
i. Preproglucagon precursor molecule Æ Islet alpha cells Æ glucagon
b. Preproglucagon is also expressed in brain and intestine
i. Sections that are cleaved are slightly different depending on where
preproglucagon is cleaved into glucagon
1. in pancreas we create glucagon
2. in other tissue we create glycentin, a slightly larger peptide
containing glucagon
6. Major stimuli and inhibitors of insulin, glucagons and somatostatin secretion
a. Insulin
i. Stimuli
1. Feeding -- high blood glucose, amino acids, fatty acids
a. Relationship between plasma glucose and plasma insulin is
sigmoidal:
i. <50mg/dl of glucose Æ 0 insulin secretion
ii. ½ Vmax = glucose level of 150mg/dl
iii. Vmax = 300mg/dl of glucose
2. GIP
3. ACh
4. Growth hormone, cortisol
5. Glucagon
ii. Inhibitors
1. Removal of stimuli
2. Fasting
3. Somatostatin
4. NE, Epi
b. Glucagon
i. Stimuli
1. Fasting – low blood glucose etc
2. High protein in the absence of carbohydrates
3. CCK
4. NE, Epi
5. ACh
ii. Inhibitors
1. Feeding – adequate or high blood glucose etc
2. Insulin
3. Somatostatin
4. Ketoacids
c. Somatostatin
i. Stimuli
1. Glucose, amino acids, fatty acids, glucagon, GI hormones,
β−adrenergic stimuli Æ stimulate synthesis
ii. Inhibitors
1. Insulin, α−adrenergic stimuli Æ inhibit synthesis
Brown, Eriksen, Jones, Heffernan, Kanjanavaikoon, Leo, Mishkin, Stern, Verlander &Wasserman
Ed. Mishkin 2006
Page 146 of 168
7. Somatostatin function, secretion, effect on insulin and glucagon
a. Functions of somatostatin (released by delta cells):
i. ↓glucose influx
ii. ↓amino acid influx
iii. ↓growth hormone
iv. ↓insulin release Æ ↓ glucose stroage and amino acid anabolism
1. Insulin somatostatin
2. insulin glucagon
v. ↓glucagon release Æ thereby decreasing glucose production and amino acid
catabolism
1. glucagon Æ ↑ somatostatin
2. glucagon Æ insulin
8. Understand the physiological basis for the major symptoms of diabetes mellitus
a. Insulin is either not produced or not received Æ Glucose can’t be absorbed Æ
Glucose is in urine, urination frequency is increased, increased loss of electroyltes
b. body behaves as if it’s starving
9. Differentiate between non-insulin-dependent and insulin-dependent diabetes
mellitus
Type 1 Type 2
Aka Juvenile onset, Insulin-dependent diabetes Aka adult onset or non-insulin diabetes
Mellitus mellitus
Onset by puberty Onset usually older but can be young
Generally undernourished, thin patients Concurrent obesity is frequent
TNF−α may be a factor in insulin resistance
10-20% of diagnoses 80-90% of diagnoses
β−cells destroyed by immune system (can originate Inability of β−cells to produce appropriate
with a viral infection), eliminated insulin production quantities of insulin; insulin-resistance
Can result from a mutation in the preproinsulin gene
Ketosis is common Ketosis is rare
Plasma insulin is low to absent Plasma insulin is normal to high – but not
high enough to prevent hyperglycemia
Acute complication: Ketoacidosis Acute complication: Hyperosmolar
coma
Unresponsive to oral hypoglycemic drugs Responsive to oral hypoglycemic drugs
Treatment with insulin is always necessary Usually doesn’t require insulin
10. Understand the physiological basis of the major symptoms of diabetes mellitus
a. see above questions 8 & 9

LOs for May 4th: Hormones of Pituitary glands

1. Understand the functional relationships between the hypothalamus and pituitary


glands
a. Hypothalamus release hormones that stimulate the anterior pituitary gland to release
its hormones
i. Hypothalamic releasing hormones are relased into the median eminence Æ
travel via blood to anterior pituitary Æ promote secretory activities
b. Hypothalamus synthesizes hormones that are relased by the posterior pituitary gland
aka neurohypophysis (therefore these are sometimes referred to as neurohormones)
2. Identify anterior and posterior pituitary hormones
a. See LO Question 3 for May 1 for a thorough explanation of pathways
Brown, Eriksen, Jones, Heffernan, Kanjanavaikoon, Leo, Mishkin, Stern, Verlander &Wasserman
Ed. Mishkin 2006
Page 147 of 168
b. Anterior
i. ACTH
ii. TSH
iii. LH
iv. FSH
v. GH
vi. Prolactin
c. Posterior
i. ADH
ii. Oxytocin
3. Know the function and characteristics of ADH and oxytocin
a. ADH
i. ↑ water permeability of the principal cells of LDTCD
ii. ADH constricts vascular smooth muscle
b. oxytocin
i. causes ejection of milk from the breast
1. stimulated by suckling, and psychological factors relating to the
presence of a baby
c. ADH and Oxytocin have preprohormones which are very similar – only differing at
2 out of 9 amino acids
i. structural similarity may result in similar effects
ii. cleavage of preprohormones Æ produce neurophysins
1. preADH Æ ADH + neurophysin 1
2. preOxytocin Æ oxytocin + neurophysin 2
iii. Neurophysins are secreted along with ADH and Oxytocin in their vesicles
iv. Absence of neurophysins Æ vesicles can’t be transported out properly
1. actual function is unknown
d. ADH
i. Stimulated by
1. ↑ plasma osmolarity
2. volume contraction of the blood
3. pain, nausea, hypoglycemia, nicotine, opiates, anti-neoplastic drugs
ii. Inhibited by
1. ↓osmolarity
2. ethanol
3. α−adrenergic agonists
4. ANP
iii. Receptors V1 and V2
1. V1A receptors Æ vasoconstriction
2. V1B receptors Æ ACTH secretion from ant pit Æ Cortisol etc.
3. V2 receptors Æ dilatory (counterbalance V1A)
a. in kidney, V2 stimulation Æ ↑ permeability of principal
cells in the LDTCD Æ decreases diuresis
iv. Syndrome of innappropriate ADH secretion
1. Tx is to limit water & salt intake or block V1 receptors
e. Oxytocin
i. Primary effect is the let-down reflex
1. Oxytocin Æ
a. myoepithelial cells contract Æ forces milk from alveola
into ducts
b. smooth muscle contraction in the uterus
Brown, Eriksen, Jones, Heffernan, Kanjanavaikoon, Leo, Mishkin, Stern, Verlander &Wasserman
Ed. Mishkin 2006
Page 148 of 168
ii. Stimulators
1. suckling
2. uterine/genital stimulation
iii. Inhibited by
1. opioids
4. Differentiate between target cells of different anterior pituitary hormones and their
main endocrine function
a. This was described in much more detail in the LOs for May 1st – Question 3
b. Categories of source cells:
i. Throughout chart, but somatotrophs are most common at 40-50%
c. Categories of effector cells:
i. Adrenal gland – cortex
ii. Adipose tissue
iii. Melanocytes

Hormone Name Source Cell Target Cell Main Functions


ACTH Corticotrophs Adrenal Cortex Increases steroid hormone synthesis
TSH Thyrotrophs Thyroid Growth, bone formation, etc
LH Gonadotrophs Gonads Ovulation, hormone release, etc
FSH
GH Somatotrophs Liver, adipose tissue Growth, lipid and carb metabolism
Prolactin Mammotrophs Breasts, Gonads Lactogenesis

Apparently the mnemonic is to think of a FLAT PiG in an ant pit.

5. Identify the glycoprotein hormones of the pituitary gland


a. Glycoprotein hormones
i. FSH
ii. LH
iii. TSH
b. Each glycoprotein hormone is made of an α and β subunit – non-covalently linked
i. α subunit is common to all 3
1. single peptide chain of 92 amino acid residues with 2 carbohydrate
chains linked to its structure
ii. β subunit is different in each
1. FSH and LH – 150 amino acids and 2 carbohydrate chains
a. 2 different ones
2. TSH – a single peptide chain of 112 amino acid residues with 1
carbohydrate chain
iii. because these are not covalently linked, it’s more like there are just the right
number of α and β subunits floating around together – neither subunit has
a strong effect alone, the two must come together to have a hormonal
effect
1. there’s slightly more α than β secreted and floating around
6. Explain the anterior pituitary disorders
a. Growth Hormone
i. GH deficiency – Shortness. Can lead to delayed puberty, mild obesity, etc
1. can be a lack of GH or its receptor
ii. GH excess – acromegaly (progressive enlargement of the hands, head, face,
feet and chest) – “Marie’s disease”
1. Giganticism
Brown, Eriksen, Jones, Heffernan, Kanjanavaikoon, Leo, Mishkin, Stern, Verlander &Wasserman
Ed. Mishkin 2006
Page 149 of 168
a. occurs before puberty
2. After puberty
a. bone growth, organomegaly, glucose intolerance
3. Tx with somatostatin analogs
b. Prolactin
i. Deficiency Æ failure to lactate
ii. Excess
1. Caused by
a. hypothalamic destruction
b. prolactin-secreting tumors
2. Leads to
a. Æ galactorrhea, aka lactorrhea (milk secretions in the
absence of nursing), ↓libido
b. Æ inhibition of GnRH Æ failure to ovulate, amenorrhea
(aka menostasis, failure of a woman to menstruate during
what should be childbearing years)
3. Tx
a. Bromocriptine Æ dopamine agonist Æ reduces prolactin
c. ACTH
i. Excess: Cushing’s disease
1. Characterized by
a. ↑ cortisol and androgens
i. Æ virilization of women (women take on
secondary male sexual characteristics)
ii. cortisol Æ aldosterone Æ hypertension
iii. cortisol Æ ↑ bone reabsorption Æ osteoporosis
b. hyperglycemia
c. ↑ protein catabolism and muscle wasting
d. central obesity (round face, supraclavicular fat, fat around
the neck)
e. poor wound healing
f. striae atrophicae (stretch marks)
2. Tx
a. Ketoconazole Æ inhibits steroid hormone synthesis
ii. Addison’s Disease: Primary adrenocortical insufficiency
1. Caused by
a. destruction of adrenal cortex
2. causes acute adrenal crisis
3. Characterized by
a. ↓androgens, mineralocorticoids, glucocorticoids
b. INCREASED ACTH because of a loss of negative
feedback
c. cortical deficiency Æ hypoglycemia
d. ACTH Æ MSH Æ hyperpigmentation
e. ↓androgens Æ ↓pubic and axillary hair in women
f. ↓aldosterone Æ ECFV contraction, hypotension,
hyperkalemia, metabolic acidosis
4. Tx: replacing the missing cortisol and, if necessary, fludrocortisone
as replacement for the missing aldosterone
iii. Secondary adrenocortical insufficiency
1. deficiency in ACTH Æ
Brown, Eriksen, Jones, Heffernan, Kanjanavaikoon, Leo, Mishkin, Stern, Verlander &Wasserman
Ed. Mishkin 2006
Page 150 of 168
a. same symptoms as Addison’s, with the following
exceptions:
i. no hyperpigmentation
ii. no volume contraction, hypotension,
hyperkalemia or metabolic acidosis
d. Hypothalamic-pituitary dysfunction is a term to describe a nonorganic relative
inactivity of (GnRH) Æ reduces FSH and LH
7. Understand treatment approach to anterior pituitary diseases
a. See throughout answer to #6
8. Explain regulation of luteinizing hormone secretion
a. LHRH
i. LHRH is essential for gonadotrophin secretion Æ gonadal hormone
regulation of LH and FSH
ii. LHRH release is pulsatile – frequency is important in regulating LH and
FSH differentially
1. more frequent Æ more LH
2. less frequent Æ more FSH
b. Feedback effects of gonadal steroids and peptide hormones (inhibin)
i. LH upregulates itself midcycle (see LOs for May 1)
1. LH interacts with membrane receptors Æ regulates LH and FSH
synthesis and release – essential for normal gonadotroph function
c. Calicum/IP3/DAG 2nd messenger cascade
d. Estradiol, dihydrotestosterone suppresses frequency of LHRH secretion
9. Know the significance of hormone replacement therapy
a. Hormone replacement therapy (HRT) is a system of medical treatment for
perimenopausal and postmenopausal women, based on the assumption that it may
prevent discomfort and health problems caused by diminished circulating estrogen
hormones. The treatment involves a series of drugs designed to artificially boost
hormone levels. The main types of hormones involved are estrogens, progesterone
or progestins, and sometimes testosterone. We covered this more in biochem.

LOs for May 5th: Growth hormone

1. Describe the regulation of growth hormone (GH) synthesis and secretion


a. GH is a single large polypeptide of 191 amino acids, in a helical conformation, with
two disulfide bridges
b. GH stimulates DNA, RNA and protein synthesis
c. Hypothalamus Æ GHRH Æ Ant pit Æ GH Æ [Hypothalamus Æ Somatostatin]
[Ant Pit Æ GH]
i. GHRH release from the hypothalamus is upregulated by sleep, stress &
dietary amino acids
ii. GHRP also acts on the hypothalamus and the ant pit
iii. somatostatin is aka somatotropin releasing inhibiting factor (SRIF)
d. GH Æ IGF (aka somatomedin)
i. IGF makes you grow, ↑ protein synthesis in muscle Æ ↑in lean body mass,
↑ organ size
ii. IGF [Ant Pit Æ GH]
iii. IGF Æ ↓cAMP and calcium levels in the Hypothalamus Æ somatostatin
[Ant Pit Æ GH]
iv. IGF GHRH
Brown, Eriksen, Jones, Heffernan, Kanjanavaikoon, Leo, Mishkin, Stern, Verlander &Wasserman
Ed. Mishkin 2006
Page 151 of 168
e. Second messengers:
i. GHRH binds to a receptor on anterior pituitary
1. Æ Adenylyl cyclase into cAMP Æ
a. Calcium rushes in from outside
b. Æ PKA
2. Æ Phospholipase C/Diacylglycerol / IP3 pathway Æ Calcium is
released from intracellular stores in the ER Æ activates PKC
2. Know the mechanisms through which GH release is controlled
a. See answer to question 1
b. Upregulation
i. long-term GHRH Æ ↑ GH transcription, activated by Pit-1 transcription
factor
ii. Thyroid hormone, cortisol Æ synergistically enhance transcription of GH
iii. Estrogen, testosterone Æ mildly enhance transcription of GH
3. Discuss GH concentrations and factors that affect release into circulation
a. GH is detectable in fetal serum and increases rapidly to reach a peak of 100-150μg/l
at the 20th week of gestation
b. Mean halflife of exogenously administered GH ranges from 9-27 minutes
(physiologically)
Figure 43-20 Lifetime pattern of GH secretion.
GH levels are higher in children than in adults,
with a peak period during puberty. GH
secretion declines with aging. Premature babies
have higher levels of GH at birth than do babies
carried to full term, but the levels drop over the
first few months of life before increasing as
shown in this graph. The clear decrease in GH
during senescence may play a role in aging. This
graph represents a normal pattern of GH
secretion, but levels can be altered by nutrition,
sleep, stress and exercise. During childbearing
years, women have more GH than men.

4. Explain the effect of somatostatin on GH release


a. 14 or 28 amino acid peptide released from hypothalamus
b. potent inhibitor of GH release
c. GHRH noncompetitively (has its own receptor)
i. ↓intracellular calcium and cAMP
5. Relationship between GH actions and peripherally generated peptide mediators
a. Peripherally generated peptide mediators are IGFs
b. JAK-STAT pathway controls IGF expression
i. GH Æ binds to a 1-transmembrane receptor Æ receptors dimerize Æ
JAKs are docked and activated Æ autophosphorylate and then
phosphorylate STATs
6. Relationship between insulin-like growth factors (IGFs) and GH actions on growth
a. GH Æ IGFs
i. Circulating IGFs originate in the liver Æ circulate bound to large binding
proteins, which regulate their availability to tissues
1. GH Æ stimulates production of these binding proteins
2. All IGFs (especially IGF-1) are reduced in individuals with reduced
GH
Brown, Eriksen, Jones, Heffernan, Kanjanavaikoon, Leo, Mishkin, Stern, Verlander &Wasserman
Ed. Mishkin 2006
Page 152 of 168
ii. Example: GH Æ directly stimulates pre-chondrocyte differentiation Æ
chondrocytes release IGF Æ clonal expansion and maturation of
chondrocytes Æ collagen and proteoglycan chondroitin production are
stimulated in chondrocytes
iii. Cortisol, estrogens, and other antagonists of GH all IGFs
b. Different IGFs:
i. IGF-2 (and its receptor) are expressed early in fetal development;
ii. IGF-1
1. is correlated with pubertal growth
2. enhance longitudinal growth
a. longitudinal: running in the direction of the long axis of
the body, i.e., verticle unless you’re really the wrong
shape
c. Pathology
i. Fasting – ↑ GH but ↓IGF
1. if you lose your [GHÆIGF], you make more GH in vain
2. a dietary element mediates the relationship between GH and IGF
ii. GH deficiency
1. Defect in GH or IGF will each cause diminished growth
2. Tx Æ give IGF Æ plasma amino acid levels go down
a. all tissues respond, long bone growth is most
significantly increased
b. lean body mass ↑
c. fat mass decreases
d. resting metabolic rate, exercise capacity, general sense of
well-being all increase
e. tall people are happier than short people
f. sorry, Whitney
7. Relationship between IGFs and anabolic processes
a. See Answer to #6
8. GH opposition to insulin action
a. GH stimulates expression of insulin, but also induces resistance to insulin:
i. GH Æ insulin transcription
ii. GH glucose uptake Æ plasma glucose concentration rises
1. prevents hypoglycemia during fasting
iii. GH enhances lipolysis
iv. GH insulin-stimulated lipogenesis Æ plasma fatty acid level rises,
adipose tissue decreases
1. In this way GH is diabetogenic
9. Clinical manifestation of acromegaly
a. progressive enlargement of the hands, head, face, feet and chest) – “Marie’s disease”
b. Caused by:
i. pituitary tumor
ii. hyperpituitarism
iii. excessive GH secretion
10. GH deficiency and replacement therapy in children
a. Children without GH are short, are delayed in skeletal and sexual maturation
b. Dx
i. GH levels don’t rise after administration of GH stimulator
ii. Non-responsiveness to T4 thyroid hormone (thanks, Hurley)
c. Tx
Brown, Eriksen, Jones, Heffernan, Kanjanavaikoon, Leo, Mishkin, Stern, Verlander &Wasserman
Ed. Mishkin 2006
Page 153 of 168
i. Give GH
1. enhanced positive nitrogen balance
2. ↓urea production
3. redistributes fat
4. reduces carbohydrate utlization
a. no increase in diabetes incidence
5. A dose will have these effects for a few hours only

LOs for May 8th: Thyroid Hormone

1. Understand the synthetic process of thyroid hormones and importance of dietary


iodide in this process
a. synthesis & secretion: Because this reaction changes sides of the cellular
membrane, I have underlined portions that take place inside the cell and italicized
portions that take place outside the cell. Traversions have been left in unaltered font.
i. Iodide is imported into the cell via Na+/I- symporter Æ Tyrosine attaches
to thyroglobulin (a glycoprotein) Æ complex is exocytosed into follicular
lumen Æ at the membrane, tyrosine residues interact with I2 Æ monoiodotyrosine
(MIT) + diiodotyrosine (DIT) on thyroglobulin Æ These complexes combine: MIT
+DIT is T3, DIT+DIT is T4 (i.e., you count the iodines), either way, it’s still bound
to thyroglobulin Æ complex is endocytosed Æ thyroglobulin is cleaved off Æ
T3 and T4 are released into blood via simple diffusion
1. MITs and DITs that failed to combine are deiodinated so that the
components (especially I2) can be recycled
ii. Endocytotic event is stimulated by TSH
iii. T4 is converted into T3 or rT3 in tissues
1. Enzyme is: 5-monodeiodinase
2. T3, triiodothyrodine, is more biologically active than T4
3. rT3 is inactive (reverse T3)
4. T4 is tetraiodothyrodine or thyroxin
5. T4/T3 ratio is 10/1 (T3 is more biologically active, so you don’t
need as much of it)
iv. Hypothalamus Æ TRH Æ Ant Pit Æ TSH Æ Thyroid Æ T3, T4
1. Ant Pit
2. Hypothalamus
2. Discuss the regulation of thyroid hormone synthesis and secretion by the
hypothalamic-anterior pituiary axis
a. Endogenous
i. T3 TSH release, gene expression, and receptors
ii. Dopamine, somatostatin, cortisol, GH TSH secretion
b. Environmental
i. Insufficient dietary iodide Æ not enough I2 to make MIT and DIT
3. Steps in thyroid hormone synthesis both intracelluarly and outside the cell (inside
the colloidal space)
a. See Question 1
b. Active iodide uptake –
i. at basal membrane of thyrocyte by Na+/I- symporter
1. Na/K ATPase powers
ii. Normal thyroidal Iodide uptake is 150μg/day
c. Thyroid peroxidase
Brown, Eriksen, Jones, Heffernan, Kanjanavaikoon, Leo, Mishkin, Stern, Verlander &Wasserman
Ed. Mishkin 2006
Page 154 of 168
i. Converts I- to I2
ii. I2 can covalently bind to tyrosines of thyroglobulin
iii. Requires hydrogen peroxide
4. Reciprocal negative feedback of thyroid hormone on its synthesis and secretion
a. See Question 1 and 2
5. Importance of serum binding proteins on the metabolism and availability of free
hormone and its degradation/excretion
a. T4 and T3 circulate bound to thyroxine binding globulin (TBG), a hepatic
glycoprotein
i. binding is 1:1
ii. 20% of THs are bound to TBG
1. TBG concentration varies
a. Æ can disturb ratio of free to bound T4
b. Å acute hepatic disease, pregnancy, estrogen therapy,
kidney disease
iii. 80% of THs are bound to transthyretin (thyroxine-binding pre-albumin) or
albumin
iv. .03% of T4 and .3% of T3 are in the free state
1. T3 has lower affinity for binding proteins, producing the
differential binding patterns, and explaining the differential
biological activity
v. The best things in life are free
b. Degradation/excretion
i. Thyroid hormone is metabolized by deiodination, deamination, and
conjugation with glucoronic acid
1. the conjugate is secreted via the bile duct into the intestine
2. normally, T3 and T4 are excreted in the feces
6. Primary physiological effects of thyroid hormone and its effects on general metabolic
processes
a. Thyroid hormone increases
i. basal metabolic rate
1. more rapid drug metabolism
2. vitamin turnover
3. steroid hormone metabolism
ii. thermogenesis
1. moderated by blood flow, sweating and ventilation
iii. Na/K ATPase Æ ↑ ADP Æ ↑ oxygen need
1. T3 Æ adenine nucleotide translocase (a mitochondrial membrane
enzyme) activation Æ brings ADP into mitochondria and pushes
ATP out of mitochondria faster
iv. oxygen consumption
1. 150ml/min in hypothyroidism
2. 250ml/min in euthyroidism
3. 400ml/min in hyperthyroidism
v. renal effects
1. kidney size ↑
2. ↑ RPF, GFR
3. ↑ reabsorption
vi. cardiac effects
1. ↑ SV
2. ↑ sarcolemmal uptake of calcium Æ shortens diastole Æ ↑ HR
3. ↑ systolic pressure
Brown, Eriksen, Jones, Heffernan, Kanjanavaikoon, Leo, Mishkin, Stern, Verlander &Wasserman
Ed. Mishkin 2006
Page 155 of 168
4. ↓diastolic pressure
vii. development
1. normal fetal development
2. brain development
3. cartilage ossification
4. linear growth of bone
5. maturation/activity of chondrocytes
6. normal lactation
7. Iodide metabolism and intrinsic component of thyroid hormone synthesis
a. Addressed in question 2
8. Mechanisms of thyroid gland activity and its regulation
a. Addressed throughout
9. Thyroid hormone receptors and intracellular signals
a. Hypothalamus Æ TRH Æ receptor is on anterior pituitary Æ ↑ IP3 Æ ↑ Ca Æ
TSH release Æ receptor on thyroid Æ ↑ cAMP Æ ↑ Ca Æ
i. ↑ iodide trapping
ii. coupling of MIT and DIT
iii. Endocytosis of colloidal Thyroglobulin-TH
iv. Proteolysis of thyroglobulin (cleavage)
b. TRH TRH receptors
10. Effects of excess or deficiencies of thyroid hormone and the consequences of
hyperthyroidism and replacement therapy
a. Excesses
i. Symptoms of hyperthyroidism: an increase in metabolic rate Æ
1. weight loss, ↑ food intake
2. excessive thermogenesis, sweating, thirst, ventilation
3. muscle weakness and atrophy, and osteoporosis (because of
protein breakdown)
4. ↑ HR and CO
5. Emotional lability -- irritable and hyperexcitable
a. what’s wrong with that?
ii. Causes: Proximal cause of symptoms is increase in T4 and/or T3 levels,
due to a variety of pathological conditions.
1. Graves disease
a. autoimmune
b. antibody binds to TSH receptor Æ agonizes Æ
overactivates TSH Æ too much THs
2. thyroid neoplasm
a. TSH-producing tumor
3. inflammation of the thyroid
4. excess TSH
5. Dietary excess of T3, T4 or iodine
a. What? You’ve never eaten thyroid? Where have you been?
6. Overdose of therapeutic THs
iii. Tx
1. β−adrenergic antagonists
2. Thiouracil Æ blocks synthesis of TH
3. Surgical or radioactive ablation of thyroid tissue
iv. Thyrotoxic crisis
1. When thyrotoxicosis causes acutely increased metabolism, it is
sometimes called "thyroid storm" and is life-threatening
Brown, Eriksen, Jones, Heffernan, Kanjanavaikoon, Leo, Mishkin, Stern, Verlander &Wasserman
Ed. Mishkin 2006
Page 156 of 168
b. Deficiencies
i. Symptoms of hypothyroidism
1. Perinatal
a. retarded growth (longitudinal and developmental)
b. marked ↓in myleination and arborization of neurons in the
brain Æ Cretinism
2. Adulthood
a. listlessness, sloth, somnolence, slowed speech, impaired
memory, decreased mental capacity, intolerance to cold,
decreased perspiration, dry skin, low CO, weight gain
ii. Causes
1. Surgical thyroid removal
2. Too much treatment for hyperthyroidism
3. Congenital cretinism
4. Idiopathic decreased TRH or TSH
5. Hashimoto's thyroiditis, the most common form of thyroiditis, is
an autoimmune disease where the body's own antibodies fight the
cells of the thyroid. It is more prevalent (8:1) in women than in
men, and its incidence increases with age. The genes implicated
vary in different ethnic groups and the incidence is increased in
patients with chromosomal disorders, including Turner's, Down,
and Klinefelter's syndromes. Hashimoto's thyroiditis usually results
in hypothyroidism, although in its acute phase, it can cause a
transient hyperthyroid state. Treatment is with daily thyroxine,
with the sodium salt of thyroxine liothyronine given when the need
to raise levels of circulating thyroxine is urgent.
iii. Tx
1. TH replacement such as thyroxine (sold as Synthroid)

LOs for May 9th: Adrenal Cortex Hormones


Note: Somehow the most basic information was not contained in these questions, so before
reading them:
1. The adrenal cortex hormones are cortisol (aka hydrocortisone, a glucocorticoid), aldosterone (a
mineralocortocoid) and the androgen precursors (especially dehydroepiandrosterone sulfate or
DHEA-S,which ultimately becomes both testosterone and estradiol)
2. The three zones of the adrenal cortex are (from out to in):
The zonula glomerulosa – which secretes aldosterone (“salt”)
The zonula fasciculata – which secretes cortisol (“sugar”)
The zonula reticularis – which secretes preandrogens (“sex”)
(Remember? Salt – sugar- sex – the deeper you go, the better it gets).

Ok. On to the LOs.

1. Regulation of adrenocortical function by hypothalamic-pituitary-adrenal axis


a. Hypothalamus Æ CRH Æ Ant Pit Æ ACTH
i. Æ Adrenal gland Æ cortisol
1. [Ant Pit Æ ACTH]
2. [Hypothalamus Æ CRH]
ii. [Hypothalamus Æ CRH]
iii. Æ aldosterone (slightly)
b. ↑K+ and ↓ ECFV Æ Aldosterone secretion
Brown, Eriksen, Jones, Heffernan, Kanjanavaikoon, Leo, Mishkin, Stern, Verlander &Wasserman
Ed. Mishkin 2006
Page 157 of 168
c. ↓Na+, ↓ECFV, ↓BP, ↓RBF Æ JG cells Æ Renin Æ Angiotensin II Æ adrenal
cortex Æ aldosterone secretion
2. General biochemical pathway for the synthesis of adrenal steroid hormones
a. synthesis
i. Cholesterol Æ Pregnenolone (rate-limiting) Æ
1. progesterone Æ
a. via 21 hydroxylase Æ 11-Deoxycorticosterone Æ via 11
hydroxylase Æ corticosterone Æ via aldosterone synthase
Æ aldosterone or [18- hydroxycorticosterone Æ
aldosterone]
i. The ability of 18-hydroxycorticosterone to convert
to aldosterone is critical
b. 17-Hydroxyprogesterone Æ
i. 11 deoxycortisol Æ cortisol
1. 11 deoxycortisol is hydroxylated in the 11
position by 11−β−hydroxylase. This is
the critical final step in synthesis of
cortisol.
ii. androstenedione Æ testosterone Æ estradiol
2. 17-hydroxypregnenolone Æ
a. 17hydroxyprogesterone Æ (see above)
b. dehydroepiandrosterone (DHEA) Æ androstenedione Æ
(see above)
ii. Note: 11 deoxycorticosterone and 18 hydroxycorticosterone, although they
are also intermediates, are active mineralocorticoids. Corticsterone can
exhibit cortisol-like functions in the absence of corticol.
3. Regulatory mechanisms of aldosterone and cortisol synthesis and secretion
a. See questions 1 and 2
b. ACTH
i. Rapid inhibition:
1. Cortisol blocks CRH
ii. Slow inhibition:
1. Cortisol blocks gene transcription
c. Secretion is via simple diffusion – no storage takes place, therefore processing must
be expedient
4. Pathways of hormone production in adrenal cortex, and whether deficiency of one
enzyme can lead to overproduction of a different hormone
a. See above answers (esp #1)
b. If you can’t synthesize cortisol, you can use corticosterone (an aldosterone
precursor) instead
5. Mechanisms of actions of cortisol and aldosterone, and how hormone receptor
binding causes the physiological response
a. Cortisol
i. essential for maintenance of plasma glucose and survival during fasting
ii. facilitates many physiological processes, including
1. metabolic processes
a. sustained glucose production from protein and reduces
sensitivity to insulin (strongly antagonistic)
i. diabetogenic
b. anabolic functions:
i. hepatic glucose production – gluconeogenesis
Brown, Eriksen, Jones, Heffernan, Kanjanavaikoon, Leo, Mishkin, Stern, Verlander &Wasserman
Ed. Mishkin 2006
Page 158 of 168
1. ↑ serum glucose
ii. hepatic glycogen production
iii. prevent hypoglycemia during fasting
iv. stimulates mobilization of amino acids and their
conversion to glucose
c. catabolic functions
i. breaks down muscular glycogen, protein and lipid
2. vascular responsiveness and muscle function
3. skeletal turnover
4. CNS modulation
5. hematopoeisis
6. renal function
7. immune responses
iii. ACTH Æ binds to fasciculata cell Æ cAMP
1. Immediate response:
a. steroidogenesis
b. ↑ cholesterol esterase
c. ↓cholesterol ester synthetase
d. ↑ cholesterol transport into mitochondria
e. ↑ cholesterol binding to P450 – the enzyme that catalyzes
[cholesterol Æ pregnenolone]
2. Subsequent response:
a. ↑ transcription of P450
b. ↑ transcription of adrenoxine
i. is required along with NADPH for conversion of
cholesterol to the active hormones
c. ↑ transcription of LDL receptor
i. important for initial absorption of cholesterol
3. Longterm response:
a. ↑ size and number of adrenocortical cells
b. Aldosterone
i. sustains ECFV by conserving Na+
ii. prevents K+ overload by accelerating its secretion
iii. Angiotensin II Æ activates type 1 receptors in glomerulosa cells Æ IP3/Ca
Æ aldosterone synthesis & release
iv. Stimulating factors (despite what Pandey wrote)
1. hypovolemia Æ low RPF Æ renin
2. acute diarrhesis
3. sodium deprivation
v. Inhibiting factors
1. excess sodium intake
2. hypervolemia
6. Action of aldosterone on kidney function and blood pressure regulation
a. reduced ECF Æ reduced RBF Æ ↓JG stretch Æ renin Æ [Angiotensinogen Æ
Angiotensin I] Æ [via ACE] Æ Angiotensin II Æ activates Type 1 receptors in
adrenal glomerulosa cells Æ IP3/Ca Æ
i. PKC Æ aldosterone synthesis Æ aldosterone is lipid and therefore is
released via diffusion Æ
1. K+ secretion
2. Na+ reabsorption Æ water reabsorption Æ rectifies low ECF
ii. ↑ K+ Æ ↑ aldosterone release
Brown, Eriksen, Jones, Heffernan, Kanjanavaikoon, Leo, Mishkin, Stern, Verlander &Wasserman
Ed. Mishkin 2006
Page 159 of 168
1. Explanation taken from
www.uhmc.sunysb.edu/internalmed/nephro/webpages/Part_D.ht
m
"Aldosterone stimulates distal nephron secretion of potassium.
The stimulation of secretion is related to the ability of aldosterone
to stimulate sodium potassium ATPase activity in cells of the distal
tubule as well as its ability to alter the apical (urinary) membrane
conductance of potassium in these cells. In the absence of
aldosterone, body potassium content and plasma K+ are increased
due to a decrease in renal excretion of potassium. In the presence
of excess aldosterone both total body K+ and plasma K+ are
decreased. An increased plasma K+ stimulates aldosterone
secretion and decreased plasma K+ suppressed it."
7. Discuss the clinical impications of cortisol in patients with serious illnesses
a. Excess cortisol can occur in sepsis, fractures, surgery, or hypoglycemia
i. levels can be 2-5 fold higher than normal Æ ↑ risk of fatality
b. Sx of excess steroids
i. Muscle weakness and atrophy
ii. osteoporosis
iii. inhibition of tissues’ ability to eliminate noxious substances and invaders
iv. delays normal wound healing
v. ↑ susceptibility to infection
c. Tx use of cortisol
i. in acute inflammatory reactions
ii. should not be used during infection, diabetes and osteoporosis
d. See LOs for May 4th – there is a whole section on Addison’s and Cushing’s
8. Understand the role of glucocorticoids in reaction to stress, injury and fasting
a. Stress – triggers cortisol release, can override negative feedback mechanisms Æ
therefore triggers cortisol release even in the overabundance of cortisol
i. prolonged exercise and other forms of severe pain promote cortisol release
ii. endorphic analgesia blocks cortisol release
b. Injury – cortisols are anti-inflammatory because they protect vesicles containing
proinflammatory proteins from being exocytosed
c. Fasting – see question 5
9. Clinical implications of aldosterone and its role in sodium, potassium, and water
balance
a. Aldosterone increases sodium and water retention and potassium secretion
b. See question 5
10. Significance and effect of atrial natriuretic peptide hormone on aldosterone secretion
a. ANP inhibits aldosterone release (strong effect)
b. ↑ blood pressure Æ right atrium Æ ANP Æ Na+ excretion Æ osmotic water
excretion Æ ↓blood volume Æ ↓blood pressure
c. ANP inhibits aldosterone release Æ prevents sodium and water reabsorption Æ
prevents increase of ECFV
11. No LO, but if you’re interested in androgens…
a. adrenal androgens undergo peripheral conversion to testosterone and estrogen:
adrostene dione Æ estrone and testosterone
i. in males, the amount of testosterone produced via this pathway is generally
insignificant in comparison to testicular testosterone
ii. in postmenopausal women, this is an important source of estrogens
Brown, Eriksen, Jones, Heffernan, Kanjanavaikoon, Leo, Mishkin, Stern, Verlander &Wasserman
Ed. Mishkin 2006
Page 160 of 168
iii. in fetal adrenal glands 16-α-hydroxylase is active and converts DHEA-S to
16-hydroxy-DHEA-S Æ converted via desulfation and aromatization Æ
estriol
1. estriol can increase up to 1000-fold during pregnancy
a. biomarker of fetal wellbeing and placental adequacy
b. adrogential syndrome
i. caused by androgen-secreting tumors or by lack of negative feedback on
ACTH production
c. insufficiencies can be caused by
i. autoimmune deficiency
ii. congenital enzyme deficiency

LOs for May 10th & 11th: Calcium Homeostasis

1. Distribution and metabolism of calcium in the body


a. Distribution
i. 99% in bones – 1 kg
1. Of this, all is theoretically exchangable but only .4% (or 4g) is
rapidly exchangable – labile or young bone, which is considered to
be in physiochemical equilibrium with the ECF
2. Of this, generally 500 mg calcium per day is deposited to bone and
reabsorbed from bone – so net bone mass stays constant during
this exchange
ii. 1% plasma calcium
1. Locations
a. 1300 mg ECF
b. 13,000 mg ICF
2. Types
a. ½ total diffusable
i. 90% ionized free calcium – biologically active,
second-messenger calcium
ii. 10% - complexed to bicarb, citrate, other small
anions
b. ½ total non-diffusable – bound to proteins, mostly
albumin
b. Metabolism
i. Recommended daily consumption 1000 mg
1. 350 mg absorbed
2. 250 mg secreted
3. 900 mg excreted (1000-350+250=900)
ii. Positive balance – seen in children when excess calcium is absorbed in
comparison to that excreted
iii. Negative balance – seen in women during pregnancy and during
postpartum lactation, when intestinal absorption of calcium is less than
calcium excretion
2. Physiological importance of plasma ionized free calcium levels
a. Plasma concentration of total calcium (ionized and non-ionized) is 10mg/dL or
2.5mM/L or 5 mEq/L
i. Biologically active aka ionized, free calcium is about 1.1mM/L
b. Calcium homeostasis is critical for
i. neuronal excitability
Brown, Eriksen, Jones, Heffernan, Kanjanavaikoon, Leo, Mishkin, Stern, Verlander &Wasserman
Ed. Mishkin 2006
Page 161 of 168
1. decreased calcium Æ hyperexcitability
2. increased calcium Æ sloth
ii. neurotransmitter release
iii. membrane integrity
iv. muscular excitation-contraction coupling
1. calcium binds to specific binding proteins (calmodulin in non-
muscle and smooth muscle cells, and troponin C in skeletal muscle)
v. endocrine secretion
vi. coagulation
c. Hypocalcemia
i. Signs and symptoms
1. CNS
a. Excitation thresholds decreased
i. Calcium competes with sodium in the sodium
channels Æ lower calcium Æ less inhibition Æ
Na+ permeability is increased Æ raising resting
potential Æ easier to meet threshold
b. CNS dysfunction – confusion to seizures
c. paresthesia in extremities
d. muscular cramps
e. muscular spasm Æ tetany
i. laryngospasm is life-threatening
2. CV
a. delayed repolarization
b. long QT
3. Chronic Æ paradoxical deposition of calcium
a. in basal ganglia
b. cataracts
4. Chvostek’s sign (pronounced Swastika)
a. Tapping on facial nerve at the angle of the mandible
produces ipsilateral facial muscle contraction
5. Trousseau’s sign
a. Hand spasm
b. can be triggered in a hypocalcemic person by inflating a
blood pressure cuff around the arm
ii. Etiology
1. malabsorption
2. inadequate dietary intake
3. hypoparathyroidism – often caused by thyroid surgery
4. renal disease
5. vitamin D deficiency
d. Hypercalcemia
i. Signs and symptoms
1. CNS
a. Excitation thresholds increased
b. lethargy
c. depression
d. psychosis
e. coma
f. neuromuscular weakness
2. CV
a. hypertension
Brown, Eriksen, Jones, Heffernan, Kanjanavaikoon, Leo, Mishkin, Stern, Verlander &Wasserman
Ed. Mishkin 2006
Page 162 of 168
b. bradycardia
3. Renal
a. *most common complaint
b. nephrocalcinosis (kidney stones)
c. reduced GFR
d. polyuria
e. dehydration
4. GI
a. nausea
b. vomiting
c. constipation
d. anorexia
e. pancreatitis
ii. etiology
1. primary hyperparathyroidism
2. vitamin D intoxication
3. secondary hyperthyroidism
4. malignancies:
a. release of PTH-related proteins
b. cytokines or prostaglandins
c. osteolytic bone metastases
3. Relationship of total plasma calcium levels and ionized free calcium levels – and the
effects of changes in blood pH
a. normally, 50% of calcium is bound to plasma proteins
b. acidosis and temporary hypercalcemia:
i. acidosis Æ decrease in pH Æ more H+ = more positive ions in the body
Æ plasma proteins becomes less negative Æ calcium is less attracted to
plasma proteins Æ the rate of divorce increases Æ more free calcium is
found in the serum Æ <50% of calcium will be bound (hypercalcemia)
Æ body will try to compensate by depositing more calcium into bone and
secretion Æ total bound+unbound calcium in the blood isreduced Æ
normalize
c. alkalosis and temporary hypocalcemia:
i. alkalosis Æ increase in pH Æ less H+ = fewer positive ions in the body Æ
plasma proteins becomes more negative Æ calcium is more attracted to
plasma proteins Æ more couples stay together Æ less free calcium is found
in the serum Æ >50% of calcium will be bound (hypocalcemia) Æ
body will try to compensate by absorbing more calcium into blood from
bone and from GI tract Æ total bound+unbound calcium in the blood is
increased Æ normalize
d. These changes in calcium binding occur very rapidly in response to pH changes
e. During pregnancy:
i. ↑ protein synthesis Æ ↓ plasma ionized calcium Æ ↑PTH Ænormalizes
plasma ionized calcium Æ total Ca2+ levels are high but plasma Ca2+
levels are normal Æ so if you do an overall assay for calcium, hypercalcemia
may be found but it’s not accurate
4. Production, effects and regulation of parathyroid hormone
a. Production
i. Parathyroid chief cells Æ PTH
ii. Synthesized as preproPTH Æ in ER, pre cleaved off Æ proPTH Æ in
golgi, pro cleaved off Æ PTH Æ secreted via granules
Brown, Eriksen, Jones, Heffernan, Kanjanavaikoon, Leo, Mishkin, Stern, Verlander &Wasserman
Ed. Mishkin 2006
Page 163 of 168
iii. PTH has 84 amino acids, only 34 are biologically active
1. synthetic PTH is 34 amino acids
b. Receptors
i. Found on kidney, bone, intestinal membranes
ii. Three types:
1. CPTH
a. binds to c-terminus of PTH
2. hPTH2
a. h for human
b. no, seriously
c. main receptor for PTH binding
d. Æ ↑cAMP Æ ↑ calcium permeability Æ mediates renal
effects
3. hPTH/PTHrP
a. binds both PTH and PTHrP
i. PTHrP its akin to PTH, mostly paracrine action, is
mostly in utero and for cartilage development
b. PTH binds Æ ↑cAMP
c. PTHrP binds Æ ↑PLC Æ IP3/DAG Æ ↑Ca2+ Æ ↑PKC
c. Effects
i. Increases reabsorption of calcium from bone into plasma
1. Side effect: releases phosphate from bone Æ decrease reabsorption
in renal tubules Æ excrete phosphate in urine Æ decrease plasma
phosphate concentration Æ overall, phosphate is lost from the
bone, but levels are maintained in the plasma
ii. Increase renal calcium reabsorption
iii. PTH is needed to make Vit D into its active metabolites (which are
necessary for calcium reabsorption)
iv. In the bone:
1. PTH stimulates osteoblasts and osteoclasts Æ net effect depends
on level of PTH
a. low levels Æ build bone
b. high levels Æ degrade bone
d. Regulation
i. Low serum calcium or magnesium Æ ↑PTH
ii. Severe decreases in magnesium inhibit PTH Æ hypoparathyroidism Æ
hypocalcemia
iii. ↑ phosphate levels Æ ↑ PTH
iv. Calcium Æ Calcium-sensitive receptors on chief cells of parathyroid gland
Æ IP3 turnover PTH Æ
1. ↓ conversion of Vitamin D to calcitriol
2. ↑ conversion of Vitamin D to 24,25 dihydroxycholecalciferol
a. Note: At normal plasma calcium level, we have 50% PTH
secretion
v. Active Vitamin D metabolite 1,25-dihydroxycholecalciferol Æ ↓preproPTH
mRNA
e. Degradation
i. in the liver by Kupffer cells Æ fragments Æ cleared by kidney
1. fragments can be found by assays
a. Old assays found all fragments Æ overestimate
Brown, Eriksen, Jones, Heffernan, Kanjanavaikoon, Leo, Mishkin, Stern, Verlander &Wasserman
Ed. Mishkin 2006
Page 164 of 168
b. New assays use two antibodies to identify only the number
of active segments – “Intact PTH ELISA” or “Sandwich”
i. That’s right. Yum.
f. Pathology
i. Pseudohypoparathyroidism:
1. Hypoparathyroidism with normal or high circulating PTH
2. pathology is in the receptors
a. More common: ↓GPCR activity Æ ↓cAMP
b. Less common Æ cAMP normal, problem is in phosphate
clearance in the kidney
3. Sx
a. Hypocalcemia
b. Hyperphosphatemia (can’t excrete phosphate)
ii. Parathyroidectomy
1. Sometimes some or all of the parathyroid glands are removed
during a thyroidectomy, though obviously never by a surgeon
trained at Tulane.
2. Æ Hypocalcemia, hyperphosphatemia
3. Symptoms develop 2-3 days after surgery – including tetany and
potential for laryngospasm (fatal)
4. Tx
a. Short term: Calcium
b. Long term: PTH replacement
iii. Hyperparathyroidism
1. Caused by PTH-producing tumor
2. Sx
a. hypercalcemia Æ calcium kidney stones
b. hypophosphatemia
3. Usually asymptomatic
iv. Secondary hyperparathyroidism
1. increased PTH excretion due to chronically low plasma Ca2+
2. common in renal disease – because kidneys activate vitamin D, so
a. Renal failure Æ↓ kidney mass Æ ↓1-α-hydroxylase Æ no
active vitamin D Æ can’t absorb Ca2+
i. Vitamin D supplements won’t help
v. Hypercalcemia of malignancy
1. Complication of cancer
2. Two types
a. 80% PTHrP secreted by tumors
b. 20% bone-eroding metastases
5. Production, effects and regulation of dihydroxycholecalciferol
a. Production
i. Vitamin D is synthesized in the skin under UVB light
1. 7-dehydrocholesterol Æ via sunlight Æ previtamin D3
ii. Vitamin D is a secosteroid, which means a steroid with open B ring (we do
not need to recognize the structure)
iii. Vitamin D3, aka cholecalciferol, has to be converted to 1,25
dihydroxycholecalciferol to be active:
1. Taken to the liver by vitamin D3 binding protein (DBP to friends)
2. In the liver: Cholecalciferol Æ via 25-hydroxylase Æ 25-
hydroxycholecalciferol Æ in the proximal renal tubules:
Brown, Eriksen, Jones, Heffernan, Kanjanavaikoon, Leo, Mishkin, Stern, Verlander &Wasserman
Ed. Mishkin 2006
Page 165 of 168
a. via 1α-hydroxylase Æ 1,25-dihydroxycholecalciferol
(active) – aka 1,25-(OH)2-D3 or calcitriol
b. via 24-hydroxylase Æ 24,25-dihydroxycholecalciferol
(inactive)
3. low serum calcium, increased PTH, low serum phosphate Æ all
increase 1α-hydroxylase activity
b. Effects/Mechanism of action
i. Calcitriol aka active Vitamin D aka 1,25-(OH)2-D3 binds to an intracellular
receptor Æ exposes DNA-binding region on receptor Æ ↑Calbindin-D
proteins Æ bind calcium Æ ↑ Calcium absorption
ii. Increases levels of two different calbindins
iii. 1,25-(OH)2-D3 Æ ↑ production of Ca2+/H+ ATPase Æ ↑ Calcium
absorption
1. Favored in the presence of PTH
iv. 1,25-(OH)2-D3 increases PO4 and Ca2+ absorption in intestine and kidney
v. 1,25-(OH)2-D3 increases osteoblast synthetic activity and also promotes
degradation Æ overall, it increases bone turnover, but the exact
mechanism is not understood
c. Regulation
i. 1- hydroxylase converts 25-(OH)-D3 to1,25-(OH)2-D3
ii. PTH Æ ↑1-αhydroxylase
iii. Calcium PTH
iv. Phosphate 1-αhydroxylase
v. 1,25-(OH)2-D3 1-αhydroxylase and PTH
vi. 1,25-(OH)2-D3 Æ ↑24-hydroxylase
d. Pathology
i. Deficiency Æ rickets in children, osteomalacia in adults (very rare in adults
in developed countries)
1. Sx
a. weakness
b. bowing of weight-bearing bones
c. dental defects
d. hypocalcemia
2. Tx
a. Vit D
i. effective in most cases, though Vitamin-D
resistant osteomalacia is possible
3. Difference between osteomalacia and osteoporosis:
a. In osteomalacia: Defect in mineralization of osteoid – lack
of incorporation of calcium or phosphate into bone matrix
i. effects appendicular skeleton
ii. may involve generalized myopathy and bone
tenderness (can be severe)
b. In osteoporosis: Quantity of normally mineralized bone is
reduced
i. predominantly effects axial skeleton
ii. no Sx until first fracture
c. Thanks to the University of Washington school of
medicine for this information.
Brown, Eriksen, Jones, Heffernan, Kanjanavaikoon, Leo, Mishkin, Stern, Verlander &Wasserman
Ed. Mishkin 2006
Page 166 of 168
ii. Deficiency from lack of sun exposure is more common in people of
African-American descent, migrants exposed to less light than they were
previously, pregnant women, and in people living in sunless, cold, heartless,
places
1. It’s a good thing they have lots of cows and cheese up in
Wisconsin
iii. Deficiency is mainly due to decreased plasma calcium and phosphate
6. Production, effects and regulation of calcitonin
a. Production
i. Parafollicular cells (of thyroid) Æ Calcitonin
b. Effects
i. Calcitonin inhibits bone reabsorption by directly inhibiting osteoclasts Æ
decreases calcium serum levels
ii. Calcitonin decreases calcium reabsorption in the kidney Æ decreases
calcium serum levels
iii. Treats hypercalcemia, has other hypothesized protective roles
1. may protect against post-feeding hypercalcemia
2. may be important for bone development
iv. No symptoms are associated with dysregulation of calcitonin
c. Regulation
i. Increased serum calcium stimulates calcitonin release
ii. Plasma levels of calcium <9.5mg/dL Æ no secretion of calcitonin
iii. Plasma levels of calcium >9.5mg/dL Æ linear increase of calcitonin
1. Normal plasma calcium is 5 mg/dL
iv. β-adrenergic agonists, dopamine, estrogens, gastrin, secretin, CCK,
glucagon Æ all lead to increased calcitonin secretion
Brown, Eriksen, Jones, Heffernan, Kanjanavaikoon, Leo, Mishkin, Stern, Verlander &Wasserman
Ed. Mishkin 2006
Page 167 of 168

7. Causes, signs & symptoms, and treatment options of osteoporosis


a. Etiology
i. caused by relative excess of osteoclastic fucntion (sic)
b. Sx/Signs:
i. loss of bone matrix Æ increased incidence of fractures in the absence of
trauma
ii. Trabecular bone is lost more rapidly because it has higher metabolic rate
1. trabecular bone is primarily in the bone in the heads of the bones,
not in the shafts, and it is this bone closer to joints that is most at
risk of fracture due to osteoporosis
2. most commonly seen in forearm, hip and in vertebral bodies – all
areas rich in trabecular bone
c. Most common form of osteoporosis is involutional (age-related) –
i. Women are at higher risk due to lower peak bone mass
ii. Postmenopausal estrogen drop decreases bone mass further
d. Bone fracture risks
i. Aging, menopause, other risk factors Æ increased bone loss Æ low bone
density Æ fractures
ii. Men have a higher peak, in 20s and 30s than women (phase 1), men and
women have similar bone loss with age (phase 3) but women have an
additional phase (2) during which we have menopausal bone loss
iii. Low Peak bone mass Æ low bone density Æ fractures
iv. Propensity to fall, poor bone quality Æ fractures
v. Prolonged bed rest Æ accelerated bone reabsorption and hypercalcemia
(reversible)
e. Tx
Brown, Eriksen, Jones, Heffernan, Kanjanavaikoon, Leo, Mishkin, Stern, Verlander &Wasserman
Ed. Mishkin 2006
Page 168 of 168
i. Increased dietary calcium/Vit D
ii. Increased exercise
iii. Estrogens (many side effects make this a non-ideal Tx)
1. Side effects: Increased risk of cancer and CV disease
2. Raloxifene (Evista) is the only SERM we’ve learned about that is
indicated for osteoporosis because it doesn’t cause cancer, and
according to JAMA 2002, it may even protect against CV disease in
women with propensity for CV problems
iv. Drugs
1. Bisphosphonates osteoclasts
a. Fosemax, Boniva
2. Calcitonin (injection or nasal spray)
a. Salmon calcitonin is the calcitonin of choice because its 20
times as effective as human calcitonin (works even better
with Stefan’s dill sauce).
3. PTH – low injected doses Æ anabolic
a. Forteo
f. Other hormones affecting calcium metabolism (and therefore osteoporosis)
i. Glucocorticoids
1. short term Æ osteoclasts Æ ↓ Ca2+
2. chronic Æ ↑ osteoporosis
3. ↓ bone protein synthesis, ↓ Ca2+ & PO43- intestinal absorption, &
↑ Ca2+ & PO43 renal excretion
4. Used as Tx in autoimmune diseases – which are more common in
women than in men, and since women are also more at risk for
osteoporosis, these tx can be harmful
ii. GH
1. ↑ intestinal Ca2+ absorption
iii. IGF-1
1. ↑ bone protein synthesis
iv. Thyroid hormone
1. promotes hypercalcemia, hypercalciuria
2. can cause osteoporosis
v. Estrogens
1. inhibit [cytokine Æ stimulation of osteoclasts] Æ protective
2. Stimulate calcitonin bone reabsorption Æ protective
vi. Insulin
1. ↑ bone formation
2. untreated diabetes Æ ↑ bone loss

Y|Ç

S-ar putea să vă placă și